Praxis II: Professional School Counselor (Questions)

Réussis tes devoirs et examens dès maintenant avec Quizwiz!

Loco Parentis

"In place of parent" --Protecting the rights of children during the school day

A parent calls the counselor and is very upset. The parent claims their child was administered a standardized IQ test by a private practitioner, who determined the student has an intellectual disability. What question should the counselor ask to best help the parent?

"Was the student administered any measures of adaptive functioning?" An IQ test alone cannot determine whether a student has an intellectual disability. The IQ score is used in combination with an assessment of adaptive functioning.

Response services

) student referrals, 2) Remediation planning, 3) crisis counseling

What is referral the most appropriate school counselor response to a student's needs?

*"When in doubt, refer out"

Language Development Late Adolescence 14-18 years

*-Acquisition of academic terms. -Subtle refinements in grammar. -Mastery of connectives (although, however). -General ability to understand figurative language (metaphors, proverbs, hyperbole).

Name the 10 Leadership Responsibilities

*-Communication -Active listening -Questioning -Interpreting -Modeling appropriate behavior -Problem solving -Linking -Assigning homework -Reviewing and summarizing

Name the 6 Leadership Tasks

*-Establish and maintain necessary discipline. -Keep the group on task -Enforce group rules -Protect group members -Encourage full participation -Moving the group in the direction of the stated objectives.

Language Development Early Adolescence 10-14 years

*-Increased awareness of academic terminology. -Ability to understand complex sentences. - Emerging ability to look beyond literal interpretations. -Emerging ability to carry lengthy conversations about abstract topics. -Growth in metalinguistic awareness.

Language Development Middle Childhood 6-10 years

*-Increased understanding of temporal (before/after) and comparative (bigger/big as) words. -Literal interpretation of messages. -Incomplete knowledge of irregular word forms. -Pronunciation mastered at age 8. -Consideration of listeners knowledge and perspective. -Sustained conversations about concrete topics. -Narratives have plots and cause-effect relationships. -Linguistic creativity and wordplay (rhymes, word games).

How old must a child be to refer themselves to receive counseling services?

*12

Criterion (predictive) Validity is...

*The degree to which the score on a test predicts the individual's score on a test or performance in some other areas. -Ex: test of scholastic achievement can be used to predict job success.

Whose role is it to encourage group development of expression, exploration, and self disclosure, and to discourage dynamics that hinder expression and to model healthy, appropriate interpersonal communication?

*The facilitator.

What are standard scores?

*Transforms raw score to common measure and allows for comparison.

Describe the first of Erikson's 8 periods of life (Infancy)

*Trust vs Mistrust Birth - 2 years Important Events: Feeding Outcome: -Children develop trust when caregivers provide reliability, care, and affection. -Lack of this will lead to mistrust.

According to Holland's career typology model, what term described students who do not yet have a clear sense of what career type they are and report enjoying activities of all kinds?

*Undifferentiated

Define useful and nonuseful counselor responses.

*Useful: facilitate clients moving to deeper level of exploration, understanding, and feeling about their experience. Nonuseful: detract or distract from client's experience, remaining superficial, and limiting or discouraging exploration, understanding, and feelings about their experiences.

Nominal

*Uses frequencies to quantify a group of numbers -Ex: Gender; males, females

ASCA Model: Delivery - Direct Student Services

- School Counseling core curriculum - lesson plans designed to help students reach competencies - Individual student planning - help students establish personal goals & develop future plans -Responsive services - meeting students immediate needs/concerns, individual/crisis/small group

ASCA Model: Management - Assessment & Tools

- School counselor competency and school counseling program assessments -Use of time assessment (80% on direct/indirect student services, 20% other) -Annual agreements -Advisory councils -Use of data -Curriculum, small group & closing the gap action plans -Annual and weekly calendars

School counselors role for working with students of exceptionalities

- develop and implement IEP -help sped teacher assess students' needs and develop effective strategies for differently able students -help parents and families of students adjust to challenges of nurturing/caring for children -counseling students to help them develop their own full potential

Behavior counseling

- developed from Skinner's operant learning + conditioning - can help to make incremental changes - behavior needs to be focused upon and understood (not Frued; no hidden problem) - focuses on what can be observed/measured objectively - operant conditioning to shape behavior/influence feelings and attitudes goals/objective clearly defined

Reliability (coefficient range)

-1.0 to +1.0; An index of the consistency of measurement often based on the correlation between scores obtained on the initial test and a retest or between scores on two similar forms of the same test.

ASCA Model: Foundation - Professional Competencies

-ASCA sc competencies outline knowledge, attitudes, and skills -follow ethical standards -protect both student and counselor`

ASCA Model: Accountability

-Analyze have students are diff. as a result of counseling program -use data to show the impact of the sc program on student achievement, attendance, and behavior -guide future action and improve future results

Career choices after HS?

-Apprenticeship -College (2yr or 4 yr) -technical /trade school -employment -military service

Person-centered therapy

-Carl Rogers -clients will grow if right conditions were established (emphasis on good/positive) -ppl are essentially good -from birth, each person is aware, inner-directed, and moving toward self-actualization -what is important is person's perception of reality rather than an event itself -ppl need positive regard: love, warmth, care, respect, and acceptance -

ASCA Model: Foundation - Program Focus

-Counselors ID personal beliefs that address how all students benefit from program -counselors create a vision statement defining student outcomes -counselors create a misssion statement aligning with the school's mission -develop program goals of how to measure vision/mission

Examples of enrichment programs:

-IB program -Governer's school for Arts, Sciences, Technology Academy -Honors classses -AP classes

Examples of primary prevention for substance abuse and addiction

-Just say "no" day -DARE -Parent and teacher education of signs and symptoms

School counselors relationships with students?

-Okay, if student no longer attends the counselors school of employment and NO harm will come to student -Never appropriate if student currently attends the counselors school of employment

ASCA Model: Indirect student services

-SC interacts w/ others on behalf of the student including referrals, consultation/collaboration w/ parent, teachers, other educators, and community orgs

ASCA Model: Foundation - Student Competencies

-Using ASCA student standards to assess 3 domains

Examples of Tertiary prevention for substance abuse and addiction (identifying those already involved)

-Work with parents/community to ensure proper counseling -student assistance counselors

Transition activities for students include...

-articulation agreements with other schools -orientation pograms across grade/building levels -buddy systems for new students -placement activities for incoming students -teaching students job-seeking skills

What is the typical role of a counselor in creating the IEP and implementing it for a student with disabilities?

-being present in the IEP meeting working with team regarding accommodations -being present in CARE team/SST meetings -communication w/ student and parent if necessary -advocating for the student and ensuring services are provided

Ways that school counselor competencies may be used:

-benchmarks for training -checklist for self-evaluation -a guide for hiring -a guide for developing counselor performance evaluations

Examples of Career development at elementary school level

-career day -guided activities -guest speakers -books -dress up days (promoting career awareness)

Solution-Focused Therapy

-concentrates on finding solutions instead of dealing with problems -individuals want to change and only small amt needed

Reality therapy

-emphasize choices taht ppl can make to change their lives -focus on environment necessary for conducting counseling & procedures leading to change -emphasize the fulfillment of psychological needs, the resolution of personal difficulties, and the prevention of future problems -pysch needs: belonging, power, freedom, fun -problems arise from failure to take responsibility for behavior

How does NCLB legislation address the needs of students with disabilities?

-ensured educational accountability -emphasized maximized learning for all children

Behavioral therapy

-focus on how to reinforce, extinguish, or modify a broad range of behaviors -all behavior is learned -no human personality traits -learn thru respondent learning, operant conditioning, social modeling

Cognitive/Behavioral Therapy

-focus on mental processes and their influence on mental health -how ppl think largely determines how they feel and behave -if individuals change their ways of thinking, feelings & behaviors will modify as a result -CBT approach emerge later, using behavioral task to modify faulty perceptions and interpretations of important life events

Existential Therapy

-focus on the pursuit of becoming an indivdual -the importance of anxiety, values, freedom, and responsibility in human life -an emphasis on finding meaning in one's actions -ppl form their lives by the choices they make

Circumstances when school counselors may break confidentiality

-harm to self or others -abuse -court order -with written permission from client

Social skills training is helpful for students who:

-have high levels of introversion -fear of class participation -withdrawn

Examples of secondary prevention for substance abuse and addiction (targeting at as-risk students)

-in school drug testing (with consent) -small groups on healthy living

Adlerian theory

-people have a specific concern and need for social interest -Emphasizes the future, holism, collaboration, and choice -Focuses on the importance of childhood, birth order, and behavioral goals -Stresses definable stages and techniques/practice in school and institutional settings

Rational Emotive Behavior Therapy

-ppl are inherently rational and irrational, sensible and crazy -children more vulnerable to outside influences and irrational thinking than adults -ppl have means to control their thoughts, feelings, actions, -dont use "to be" - individuals should speak & think of their beahviors as separete form their personhood - gives freedom to change

Cognitive theories are intended to:

-restructure schemas -change maladaptive thought patterns

Counselors role in helping students of divorce:

-small group counseling -encourage student to participate in school and community events -connect family with resources

Gestalt therapy

-stresses the perception of completeness and wholeness -self actualization - being who one is (not becoming) -antideterministic: ppl able to change and become responsible thru action -need to take care of whole self and feel emotions (if don't create problems)

Ways to share data results with stakeholders?

-websites -one-page handouts -part of larger report to admin and school board members -presentation to faculty -add to school district's data material

Principles of measurement

...

Reliability (internal consistency(

...

Reliability (parallel forms)

...

Reliability (stability)

...

What are behavioral anchors (in rating scales)?

...

What is the relationship between standard erros and true scores?

...

Kohlberg Staes

1) Amoral stage: 0-7, personal needs need to be satisfied 2) Pre-conventional stage: 7-10, learn rules to stay out of trouble 3) Conventional stage: by age 10, people should be moving to where morality means to follow norms, values 4) Post conventional stage: people begin to reflect not he abstract of what is right or wrong.

Name the 4 group stages?

1) Initial - forming, exploring, orientation 2) Transition - power and control, storming 3) Working - norming stage, cohesion, negotiation 4) Termination - closure, seperation

School counselor's planning components

1) Procedures to evaluate school goals 2) to assess the needs of all constituents 3) to select program goals and objectives

What do the National Career Development guidelines include (3 things)?

1) Self-Knowledge 2) Educational and occupational exploration 3) Career-knowledge

What are Piaget's 4 stages?

1) Sensorimotor 2) Preparations 3) Concrete Operations 4) Formal operations

ASCA ethical standards on the use of computer technology in working with students

1) ensuring the applications are appropriate to the needs of the students, 2) ensuring that students know how to use the applications, 3) ensuring that the applications are nondiscriminatory, and 4) ensuring that follow up counseling with the counselor to students are all required.

What are the 3 domains in multicultural counseling competencies?

1. Awareness of own assumptions, values, and biases. 2. Understanding the worldview of the culturally different client. 3. Developing appropriate intervention strategies and techniques.

Reflecting involves...

1. Communicating empathy 2. Neither adding to nor subtracting from the clients' messages 3. Communicating back to the clients the main meaning of their messages 4. Reflecting clients' feelings/emotions - directly expressed in words or implied thru nonverbal aspects 5. Checking with the client to see if the counselor's reflection is accurate

What are the 3 parties involved in the consultation process?

1. Consultant 2. Consultee 3. Client or situation to be addresssed

4 Components of Counseling program evaluation and improvement

1. Counselor self-analysis (competences assessment) 2. Self-analysis of program using program assessment 3. Admin. evaluation of counselor performance (appraisal) 4. Review of program goals created at the beginning of the school year

What are the 3 types of results reports?

1. Curriculum-results reports 2. Small-group results reports 3. Closing-the-gap results reports

ASCA's 9 Steps of Ethical Decision making:

1. Define the problem 2. Apply ASCA ethical standards & the law 3.Consider student's developmental level 4. consider the setting (parent rights/minor rights) 5. Apply moral principals 6. Determine potential actions & consequences 7. Evaluate selected plan 8. Consult 9. Implement

What is ASCA's 9 Step model to ethical decision making?

1. Define the problem emotionally & intellectually 2. Apply the ASCA ethical standards and law 3. Consider student developmental level 4. Consider the setting (parent rights & minor rights) 5.Apply moral principals 6. Determine potential course of action and consequences 7. Evaluate the selected plan 8. consult 9. implement the course of action

What should an annual agreement between a counselor and principal contain?

1. Explanation of the school counselor's use of time based on school data. 2. Indication that the school counseling program's mission and goals align with the school's mission and goals. 3. Counselor's responsibilities within the school counseling program, including caseload and program components.

Attending involves..

1. Focused listening 2. Perceiving verbal and non-verbal messages 3. Cognitive and affective pats of the client's experience

5 stages of group therapy

1. Forming 2. storming 3. Norming 4. performing (synergy) 5. termination

What are the basic elements of comprehensive guidance programs?

1. Guidance curriculum 2. Individual planning 3. Responsive services 4. System support

Jean Piaget's stages of MORAL development

1. Heteronomy (children don't understand rules, they rely on authority to reinforce 2. Autonomy (9+ years, Begin to understand the behaviors of right and wrong

4 stages of needs assessment

1. Identify guiding questions 2. Identify populations to be assessed 3. Collect needs assessment data 4. Analysis of data

What are Gardner's 8 intelligences?

1. Linguistic 2. Logical/mathematical 3. musical 4. spatial 5. bodily-kinesthetic 6. naturalistic 7. interpersonal 8. Intrapersonal

6 stages of systemic change:

1. Maintenance of the old system 2. Awareness 3.exploration 4. transition 5. emergence of new infrastructure 6. predominance of new system

Benefits of small-group counseling for students

1. Natural interpesonal context for students 2. Creates safe context w/n which students can practice interpersonal skills and get feedback 3. Allow students to hear form others with similar experiences 4. Allow many more student to be served by counselor then in one-on-one activities

Scales of measurement

1. Nominal 2. Ordinal 3. Interval 4. Ratio

Intrepreting involves...

1. Offering clients new and facilitative ways to understand their experiences 2. Basing interpretation on counselor's perceptions of the client's experiences

The development of a comprehensive guidance plan has the following stages:

1. Planning 2. Organizing 3. Implementing 4. Evaluating

Stages of a group

1. Pregroup stage - forming a group 2. Initial stage - orientation & exploration 3. Transition stage - leader helps group begin to work on concerns 4. Working stage - productiveness 5. Final stage - id learned & apply

5 main modes of consultation?

1. Prescription 2. Provision 3. Initiation 4. Collaboration 5. Mediation

What are the 3 levels of prevention?

1. Primary Prevention 2. Secondary Prevention 3. Tertiary Prevention

What are the 4 forces of human behaviors regarding counseling?

1. Psychodynamic 2. Behavioral 3. Humanistic 4. Multicultural

"10 Diversity dimensions"

1. Race 2. Ethnicity 3. Culture 4. Language 5. Gender 6. Age 7. Sexual Orientation 8. Religion 9. Giftedness 10. Disabilities

What are some cultural issues in assessment?

1. Representativeness on norm groups in terms of gender and culture. 2. Culture biases in language use 3. English as a second language 4. Gender bias

3 most used career inventories:

1. SDS 2. SII 3. KOIS * all are based on Holland's codes

What are the 3 components of a school counselor performance appraisal?

1. Self-evaluation 2. Administrative evaluation 3. Assessment of goal setting

According to NCDA what competencies are appropriate for all students?

1. Self-knowledge 2. Educational and occupational exploration 3. Career planning

adlerian theory (Adler)

1. The one dynamic force behind people's behavior is the striving for success or superiority. 2. People's subjective perceptions shape their behavior and personality 3. Personality is unified and self-consistent. 4. The value of all human activity must be seen from the viewpoint of social interest. 5. The self-consistent personality structure develops into a person's style of life. 6. Style of life is molded by people's creative power.

What are the 3 tiers of RTI?

1. Tier I -universal core instruction interventions (action plan) 2. Tier II- supplemental, strategic intervention, students that are at-risk (closing-the-gap) 3. Tier III- Intensive, individual interventions, for students at high-risk (closing-the-gap)

Erik Erikson's 8 stages of psychosocial Development

1. Trust v. Mistrust 2.Autonomy v. Shame and doubt 3. initiative v. guilt 4. industry v. inferiority 5. identity v. role confusion 6. intimacy v. isolation 7. generativity v. stagnation (recreation) 8.Ego Integrity v. despair

Questioning involves...

1. Using open-ended and closed-ended questions 2. Asking for clarification and meaning 3. Asking to understand the client's experiences better

Confronting involves...

1. Verballying holding apparently discrepant or incongruent aspects of the clients' messages and behaviors "in front of" clients for them to see 2. Helping clients clarify, resolve, or accept the discrepancy

5 areas of assessment

1. achievement 2. aptitude 3. Intelligence 4. Interests 5. Personality

3 areas of motivation:

1. behavioral (Skinner) 2. Cognitive (Piaget) 3. Humanistic (Maslow)

Classroom guidance: skill acquisition...

1. listening skills 2. self assessment skills 3. study skills 4. interpersonal skills

An organized guidance curriculum matches instruction with...

1. students developmental status 2. uses varied learning strategies for different students 3. promotes interactive learning processes

Savickas 4 areas of development

1. vocational personality 2. developmental tasks 3. adaptability 4. life themes

Average IQ score?

100

Carl Roger's Theory of Personality is based on how many propositions?

19 propositions. His theory was based on the understanding that all people can achieve their goals. When this achievement happens, he believed self actualization takes place

When does the preoperational (Jean Piaget) stage occur?

2-7 years of age; During this stage, children are capable of developing an internal representation of the world. During this stage they can describe their feelings, events, and other people.

When school counselors are helping to create an assembly, they should divide the responsibilities into how many basic categories?

3: Tasks to be performed before the assembly Tasks to be performed during the assembly Activities to do immediately after the assembly

How many levels of support does the RTI (Response to Intervention) method have?

3: Tier 1: High quality, classroom instruction, screening and group interventions Tier 2: Targeted Interventions Tier 3: Intensive Intervention and comprehensive evaluation

Relational aggression comes in how many forms?

3; Excluding others from social activities Damaging another student's reputation Withdrawing attention and friendship from another student

The ASCA model is based on how many sub elements?

4 quadrants; Foundation Delivery Management Accountability

How many primary symptoms of stress are there?

4: Environment Physiological Cognitive Social

How many types of questions are used in SFBC?

4: Scaling Coping Exception Miracle questions

How many stages does Jean Piaget outline in his cognitive development in children?

4: Sensorimotor: birth to toddlerhood 18-24 months Preoperational: toddlerhood to early child hood 7 Concrete Operational: 7-12 Formal Operational: adolescence to adulthood

Students are protected from "unreasonable searches" under the

4th Amendment to the U.S. Constitution.

Maslow Hierarchy of Needs:

5. Physiological Needs 4. Safety Needs 3. Social Needs 4. Esteem Needs 5. Self-actualization (morality)

What is a common task of a guidance advisory committee (GAC)?

A GAC is commonly charged with the task of reviewing program data and making recommendations based on its evaluations.

What is a sociogram?

A chart of the social interrelationships in a group of students. It can help a teacher identify friendships, isolated students, and personal conflicts.

behavior checklist

A checklist that provides descriptions of specific skills (usually hierarchical order) and the conditions under which each skill should be observed. Some checklists are designed to assess one particular behavior or skill area. Others address multiple behaviors or skill areas. Most use a Likert scale to rate responses.

Mr. Zhao, the school counselor, always writes two questions about the lesson topic on the board when he is coming into a classroom to conduct a classroom guidance lesson. Students know from past experience and Mr. Zhao reinforces to each class that students should write simple answers to Mr. Zhao's questions in their notebooks while everyone is finding their seat. What type of strategy is Mr. Zhao using?

A classroom management strategy. Writing interesting questions about a lesson topic or writing brain teasers on the board before a lesson is a common classroom management strategy. It helps students get focused and interested in the lesson.

What is "Etic" ?

A cultural counseling approach that considers characteristics presumed to be shared by all cultural groups, such as identity development, acculturation and social class differences

What is a sociogram and when would you use one?

A graphic representation of social links a person has. It can be drawn on the basis of many different criteria: social relations, channels of influence, lines of communication etc. Another definition: It represents the patterns of relationships between individuals in a group, usually expressed in terms of which persons they prefer to associate with.

What warrants gifted testing?

A score in the 90th percentile or above in composite, reading, or math

What is required for confidential information about a student's counseling to be released to 3rd parties?

A signed release

Exceptionality

A significant difference in functioning from an established norm. Ex: a student who scores below 80 or above 120 on an IQ test is said to have an exceptionality. Also applies to emotional and physical development.

What is the meaning of a negative correlation coefficient?

A statistic that gives a measure of how closely two variables are related. A statistical measure of the extent to which variations in one variable are related to variation in another. Negative correlation means that the two values vary in opposite directions. So, as parameter x increases, parameter y decreases.

What is the meaning of the term exceptionality, as used by educators?

A student has some area of functioning in which he or she is significantly different from an established norm. This definition includes both students with disabilities and those with special gifts or talents.

What is the best place to store student grade data?

A student information system, where security needs can be met and the filed can be backed up.

A system of behavior modification based on the systematic reinforcement of target behavior. The reinforcers are symbols or "tokens" that can be exchanged for other reinforcers.

A token Economy *Operant conditioning "this for that" *Behavioral therapy *Skinner

Discriminant Validity is ...

A type of validity that is determined by hypothesizing and examining differential relations between a test and measures of similar or different constructs. It is the opposite of convergent validity. Evidence that a measure of a construct is indeed measuring that construct.

behavioral counseling is often used with students that have been diagnosed with __________________

ADHD *B.F. Skinner

I. School counseling programs II. Foundations III. Management IV. Delivery V. Accountability These are components of ?

ASCA School Counselor Competencies

_____________________ identify and prioritize the specific knowledge, attitudes, and skills that students should be able to demonstrate as a results of the school counseling program.

ASCA Student Standards (student competencies)

Program evaluation, planning and advertisement are all part of of ASCA ______________ component

ASCA's Management component

This component of the ASCA national model is used to determine how students are positively affected/different as a result of the counseling program.

Accountability Component

The purpose of this ASCA component is to analyze data that has been collected and make program decisions based on the analysis.

Accountability component

4 Components of the ASCA national model?

Accountability, Management, Foundation, Delivery

_______________ Measures students academic progress. Field Included: promotion & retention rates Drop out rates grad rates standardized test data grade point averages completion of academic programs

Achievement Data

AYP

Adequate Yearly Progess

"Individual Psychology"

Adlerian (Alfred Adler)

Who should conduct the counselor's performance appraisal?

Administrator

When working with adolescents in small groups, what is important to remember?

Adolescent girls mature more quickly than adolescent boys do, which may affect interactions in mixed-sex groups.

Advantages and Disadvantages of Online-Internet Counseling

Advantages: easily accessible, convenient, and disinhibition. Disadvantages: loss of personal touch, harder to read emotions online, breach of confidentiality risk, and unethical internet counselors.

Advantages and Disadvantages of Classroom Observations

Advantages: observers see progress as it is happening, counseling staff has easy access to projects/activities, the observers skill can increase based on observation, and observations can provide both quantitative and qualitative data. Disadvantages: staff members do not always have time for observations, confidentiality must be considered, and the evaluator is semi intrusive.

Advantages and Disadvantages to Surveys

Advantages: they are quick, can be performed a number of ways, close ended surveys provide quantitative data, can be anonymous. Disadvantages: questions may be misunderstood, closed ended questions provide partial depth, must understand the survey design to use it, quantitative surveys are less useful when the surveyed group is small.

Yalom's curative factors in group counseling

Altruism Group cohesiveness Universality Yalom's approach relies on sharing feelings and experiences as a way to cope with problems.

Triadic consultation

Among the counselor, teacher, and student

Kohlberg

Amoral stage -- 0-7 - personal needs need to be satisfied. Pre-conventional stage - 7-10 - learn rules to stay out of trouble. Conventional Stage - -By age 10, people should be moving to where morality means to follow the norms, values. In the Post Conventional Stage, people begin to reflect on the abstract of what is right or wrong. Most don't reach this stage College appears to nurture this stage.

What is Internal Consistency?

An estimation based on the correlation among the variables comprising the set.

What teaching style or learning activity is most effective for helping students understand and appreciate differences? Why?

An experiential (interactive) style tends to be an effective approach - although others are also effective.

According to Piaget, what is a schema?

An individual's representation of something.

What is an interpretation response?

An interpretation response provides some NEW information or insight related to a client's statement.

During an observation of an elementary school student, a school counselor records a high rate of off-task behavior that is interfering with the student's completion of class work and causing a classroom distraction. To increase on-task behavior, the school counselor should initially recommend that the teacher

Analyze antecedents and consequences of the student's behavior

The counselors at a high school are reorganizing the counseling program to model a comprehensive, developmental model. They need to develop priorities that will guide them in the process of putting together programming to meet the needs of various groups in the school and determine primary goals and objectives. Which tools or methods will most effectively assist them?

Analyzing a community-wide, data-driven needs assessment - this is necessary to determine which areas and delivery mechanisms will best meet the needs of their population.

Behavior is made up of how many components?

Antecedents Behaviors Consequences

Evaluates potential or ability to learn, assessing in a variety of areas

Aptitude

A negative correlation coefficient means?

As one variable increases, another variable decreases, and vice versa

Annie is a fourth-grade student in Ms. Garcia's homeroom class. For the past few days, Ms. Garcia has noticed that Annie has not been as engaged with her classmates as usual and seems withdrawn. Ms. Garcia asks the counselor for help with Annie. What should be the counselor's first step?

Ask Ms. Garcia to ask Annie to come see the counselor the next day. Creating an initial contact with Annie directly is the best first step to learn more about how she is doing and what her concerns might be.

A counseling department evaluates data from a needs assessment that was distributed to teachers, parents, and students. The data show that the programs and services provided for students have not been effective. To best meet the needs of the school community, what would be the next step for the counseling team?

Assemble an advisory council to discuss the results of the needs assessment. Advisory councils consist of stakeholders who can assist in program evaluation. In this instance, using an advisory council would be the most effective way of meeting the needs of the school community.

Independence; which encourages the client to exercise freedom of choice and behavior when actions do not infringe on the rights and beliefs of others Definition of?

Autonomy (one of the five ethical principals)

dual relationship with teacher

Avoid entering into a personal counseling relationship with staff. From a consulting perspective it is appropriate and recommended, that the school counselor be a resource for other staff. Offering referral to potential care providers that the teacher might see out for counseling is the best approach.

In addressing the needs of minority students, counselors should do what?

Avoid placing blame for academic failure on the values or way of life of student's families. Counselors should avoid the Deficit Model of education which denigrates certain cultural backgrounds and unfairly assumes that children from certain backgrounds are less capable than others.

What is a good way to discourage inappropriate behavior in a group? Why?

Avoiding a response and looking away from an individual enacting inappropriate behavior is a good way to extinguish or discourage that behavior in a group. Because it doesn't positively reinforce behavior.

A counselor encounters mildly inappropriate behavior by a member of a student group. What should the counselor do to discourage the behavior?

Avoiding a response and looking away is a good way to extinguish or discourage inappropriate behavior in a group.

Nonmaleficence

Avoiding actions that cause harm.

Behavioral theorist

B.F. Skinner

Temperament

Behavioral or emotional traits that are relatively constant over a child's development

Foundations

Beliefs and Philosophy Mission Statement CCGP Student Outcomes History and Role of a S.C. Knowledge of Human Development Ethical and Legal Principles

Foundation

Beliefs and Philosophy, Mission Statement: ASCA content standards for student academic, career and personal/ social develpoment

Areas of "program focus"?

Beliefs, vision statement, mission statement, program goals

Alfred Adler

Believed in mastering 3 universal life tasks: Building friendships Establishing intimacy Contributing to society Emphasis on interpersonal relationships; striving toward superiority because of feelings of inferiority

Erikson

Believed there were cultural differences in behavioral development

The counselors responsibility to do good. Counselors should positively contribute to the well-being of their students Definition of?

Beneficence (1 of 5 ethical principals)

Who introduced 6 classifications of learning levels? 1. knowledge 2. Comprehension 3.Application 4. Analysis 5. Synthesis 6. Evaluation

Benjamin Bloom (Bloom's Taxonomy)

What is the sensorimotor stage of Jean Piaget's cognitive development theory?

Birth to 2 years of age. During this stage, the child has a low level of awareness.

In regards to physical development, what time period does childhood include?

Birth to puberty (3 stages: early childhood, middle childhood, and late childhood)

Mattering *Savickas

Brings purpose and meaning to work

What is most the most important aspect in working with adolescents in small group setting?

Building trust

Name this therapy: thoughts and feelings affect behavior problem solving approach cognitive restructuring goal driven direct socratic dialogue homework cognitive modeling

CBT / Aaron Beck

In regards to educational software, what does CMS stand for?

Campus Management System; software application used to manage student data

3 Main Levels of Counseling

Career Professional Residency

Piaget's Theory of Cognitive Development

Children progress thru 4 stages of cognitive development, 2 processes underlie development: assimilation and accommodation. Sensorimotor, pre operational, concrete operational, formal operational

Eliminating alternatives perceived as inappropriate due to societal expectations, prestige, and gender *Gottfredson

Circumscription

Writing questions about the lesson topic or brain teasers on the white board before the lesson begins is an example of ?

Classroom management strategies

What are the purpose of Cognitive therapies and why does using cognitive theories w/ a student work?

Cognitive therapies focus on changing maladaptive thought patterns. The purpose of using cognitive theories is to change thought patterns through reconstructing of schemas

What therapeutic approaches are best used to help adolescents who are experiencing depression?

Cognitive-behavioral therapy - identified as being effective in the treatment of mood disorders and, in severe cases, is often used in conjunction with medication.

Group characteristics that promote success

Cohesiveness, caring, level of trust among members, freedom to experiment, commitment to change, Yalom's curative factors of group (1995)

What is a collaborative-interdependent consultation?

Collaborative mean the process is collaborative among other colleagues, interdependent means that other people share expertise and that the solution isn't dependent on one person.

A middle school counselor has been asked to conduct a follow-up study to determine the effectiveness of the school's career information program. Which of the following steps should the counselor take first?

Collecting relevant data from stakeholders

Standard Deviation

Commonly used tool in statistics. It is computed by using a mathematical formula that shows the spread of a set of data.

What is a norm-referenced assessment?

Compares student performance with that of other students.

What is the most appropriate use of the Wechsler Intelligence Scale for Children?

Comparing students' scores with those of the general population

A high school counselor is working with a group of students who are uncertain about what to do after graduation. The most effective first step for the school counselor to take in promoting a successful post secondary transition is to have these students

Complete an interest inventory

A student is throwing tantrums in class. What is the best action for the counselor to take to determine the antecedents and consequences of the student's tantrums?

Completing a functional behavior analysis since it is used determine the antecedents and consequences of a problematic behavior.

Giving up designed alternatives for those that are more accessible *Gottfredson

Compromise

What does CAC stand for?

Computer Assisted Counseling. CAC software offers assistance in helping counselors with their work. Ex: assistance in administration of career development inventory, creating an activity sheet, and going through a virtual learning experience.

What does CMC stand for?

Computer Managed Counselor; help counselors effectively manage their school related work

According to Piaget's theory of development at what stage can children begin to think logically for example understand mathematical concepts?

Concrete operational

Eric's teacher shows him two glasses. One glass is tall and thin and the other glass is short and wide. His teacher pours 8 ounces of juice into the tall, thin glass. The teacher then pours the juice from the tall, thin glass into the short, wide glass and asks Eric which glass had more juice in it. Eric laughs and says that they both had the same amount of juice in them. Eric is most likely in which stage of development?

Concrete operational - children are able to recognize that even though a liquid can change shape, the amount of liquid does not change

What happens during the continuously improving stage of RTI?

Conduct an ongoing evaluation after a plan has been implemented.

Primary intervention

Conducted at the beginning of the school year, intended to prevent a problem before it begins

The ability to be authentic and genuine when working with a client

Congruence *Carl Rogers

Often used to diagnose ADHD?

Connors scale

Dysfunctional Families

Considered an at-risk population because they present serious challenges to children. However, a family is not considered to be at-risk because it has a single parent or a minority background or a gifted or disabled child.

What is the best way to describe a reliable test score?

Consistent - reliability refers to how consistent or stable scores are from one test administration to the next.

roles of a school counselor

Consultation, appraisal, counseling and guidance curriculum

____________ is recognized for his or her expertise on the client and shares this expertise with the consultant

Consultee (the person requesting the assistance of the of the consultant)

An elementary teacher shares a concern about a female student. The teacher states that the student, normally a bright and active participant in class, has become increasingly withdrawn over the past few weeks. After meeting with the student, the counselor suspects the student is being sexually abused by her mother's new boyfriend. In this situation, a school counselor's first responsibility is to do what?

Contact CPS. Sexual abuse of a minor is, by law, a reportable event.

What is the first thing you should do when you suspect sexual abuse of a minor?

Contact Child Protective Services. Sexual abuse of a minor, by law, is a reportable event so the first response is to notify them.

A middle school principal asks the counselor to establish a social-skills group to resolve a dispute that two groups of sixth-grade students have been having for several months. What is the first step the counselor should take?

Contact each student's parent or guardian to discuss the purpose of the group and to request permission for the student to attend.

A counselor is selecting a standardized test to help teachers determine the extent to which their students' performance meets learning outcomes defined as specific goals and objectives. Which type of assessment is the most appropriate?

Criterion-referenced - enables teachers to determine whether their students are performing up to the standard measured in the assessment.

___________________ assessment enables teachers to determine whether their students are performing up to standard measures in the assessment

Criterion-referenced assessment

Comparing an individual's score to a predetermined criteria or learning standard

Criterion-referenced assessments

Common names for online-internet counseling

Cyber counseling Web counseling Online counseling

Veracity

Dealing truthfully with individuals with whom counselors come into professional contact.

Every Student Succeeds Act (ESSA)

Dec 10th, 2015: Replaces NCLB Title IV of ESSA: Provide access to a well-rounded education Improve school conditions for learning Improve use of technology to increase academic achievement and digital literacy of all students Formula funded: 20% of $ must by spent on creating well rounded programs 20% must be spent on student learning conditions programs

McKinney -Vento Act of 1987 What is it?

Defines homelessness as lacking a fixed regular & adequate nighttime residence & provides federal money for homeless shelters.

Macrosystem

Describes the cultures in which a child lives within *Ecological Theory

Life themes *Savickas

Determine what matters to an individual

___________ services are provided WITH student

Direct student services

When counselors separate data variables to determine if there are any groups of students that are not performing as well as others. What type of data?

Disaggregating Data

Explain an ordinal scale? Exp?

Distinctiveness and magnitude (larger numbers mean more) Example: class rank

Explain a nominal scale? Exp?

Distinctiveness only (each # is a unique entity). Example: zip code

3 stages of adulthood physical development

Early adulthood, middle adulthood, and late adulthood. Adulthood stage begins when height stops increasing (20 for males, 17 for females) Middle adulthood begins at the age of 40 and ends at age 60. Late adulthood 60+

__________________________ was developed by Urie Bronfenbrenner. Bronfenbrenner believed that a person's development was affected by everything in their surrounding environment.

Ecological Systems theory

Name this theory: Individual Microsystem Mesosystem Exosystem Macrosystem Chronosystem

Ecological Systems theory *Bronfenbrenner

Types of groups

Educational, task, discussion, experiential, support, self-help, counseling/therapy

When do counselors implement career education?

Elementary school

Creator(s) of REBT?

Ellis and Dryden

What is "O-NET" used for?

Enables students to link career assessments to interests, abilities, and values

A middle school counselor decides to consult with the parents and guardians of a group of adolescents identified as at-risk based on their behavior. Which of the following should be the school counselor's primary goal for these interactions?

Encouraging the parents and guardians to be part of an ongoing support system for changing their children's behavior

What is Etic. ?

Eric. approach considers characteristic presumed to be shared by all cultural groups such as identity development, acculturation, and social class differences

Erik Erikson

Erikson believed that social factors also played a vital role.His model of psychosocial development consists of eight developmental stages, occuring in a fixed order: 1. Trust vs. Mistrust (between birth and 1 yr); 2. Autonomy vs Shame & Doubt (ages 2 to 3); 3. Initiative vs Guilt (ages 4 to 5); 4. Industry vs. Inferiority (ages 6 to puberty); 5. Identity vs. Role Confusion (adolescence); 6. Intimacy vs. Isolation (young adulthood) 7. Generativity vs. Stagnation (middle adulthood) 8. Ego Integrity vs. Despair (late adulthood) When the outcome of a crisis is favorable, the person achieves a certain virtue or strength; when it is unsuccessful, the person develops a maladaptive character and continues to struggle with this conflict later in life.

What would be the most effective strategy for a school counselor who wants to develop mutual, reciprocal partnerships with community agencies?

Establish a school-community advisory council with members from all area agencies that might be working with children and youth, and then hold regular meetings to create a network partnership. Engaging external agencies by creating an advisory council or committee would establish an ongoing collaboration that may benefit all parties.

In Kohlberg's theory of moral development, an individual must ________________________________

Experience each level, there is no level skipping.

When choosing career development materials for use at the middle school level, a school counselor should focus on materials that facilitate students'

Exploration of career clusters

A five year old boy continues to attempt to play with children who openly ridicule him and leave him our of games. His persistence with this group of children has recently resulted in a number of fights on the playground. After being referred to your office, you learn that he has an unsupportive home life. His parents were divorced last year. He lives with his mother who works nights, and he does not see his father. He is often left at home alone; however, a neighbor watches him from time to time. According to Pavlov's theory what should the counselor do to help this child?

Explore social development delays he has and propose some type of intervention.

The four stages for the implementation of RTI

Exploring and adopting Planning Implementing Continuously improving

_______________ motivation rests outside the individual, and is driven by reward or threat of punishment

Extrinsic Motivation *Skinner

Anima?

Feminine side

Trusting the therapeutic relationship built on reliability and authenticity. Counselors should honor their commitments to clients.

Fidelity (1 of 5 ethical principals)

Which of the following organization provides students with a free Web site to help plan and track their school progress?

Florida Center for Advising and Academic Support

Kohlberg's "conventional stage" of moral development focuses on what?

Focuses on societal definitions right and wrong

Person Centered Theory (carl rogers)

Focuses on the uniqueness of the individual. Used term client-centered to emphasize inherent goodness of people. Strive for congruences between the ideal self and the real self. Believed psychological disorders are result of overly demanding and critical parents. If you didn't have acceptance and love when younger, you want/expect it later in life.

In Piaget's theory of cognitive development, what term is used to describe the final stage of development?

Formal Operations; coincides with adolescence, a child is capable of hypothetical and deductive reasoning. At this stage, a child is capable of understanding a topic from multiple perspectives.

______________ assessments are normally objective, norm-referenced, and quantitative

Formal assessments

According to Piaget's theory of development at what stage can children master abstract thinking?

Formal operational - at this stage concepts such as death become easier to understand.

Monitoring student learning and providing ongoing feedback that can be used to improve instruction, curriculum, or teaching methods

Formative Assessment

1st stage of group work, members use first impressions, develop trust, low productivity and high anxiety during this stage

Forming

Pregroup stage description

Forming a group - designing the proposal, attracting members, screening & selecting, and orientation

Autonomy

Fostering the right to control the direction of one's life.

The following are characteristics of what ASCA component? program focus, student competencies, professional competencies

Foundation

Known as the "father of vocational guidance"?

Frank Parsons (trait & factor theory)

Developmental theorists

Freud, Erikson, Piaget - Continuous process move through stages

What is most important when working with adolescents?

Gaining their trust

theory that is very focused on the here and now

Gestalt (Fritz Perls)

Being in the present (here and now) and the empty-chair technique are related to which therapy?

Gestalt therapy - which was founded by Fritz and Laura Perls in the 1940s.

Benefits of Memberships in Professional Organizations

Grow professionally as well as personally Benefit from others' experiences and research Journals, magazines, and publications are free or discounted Professional development opportunities: conferences, workshops, webinars Liability insurance: free with ASCA membership Free resources: lesson plans, handouts, online community

Behavioral counseling is based on the theory that human development is primarily a function of an individual's

Growing mastery over unconscious drives..

What is GAC? What do they do? Who do they make recommendations to?

Guidance Advisory Committee; they are charged with the task of reviewing program data and making recommendations directly to the counselor?

What are the 4 components of an effective counseling delivery system

Guidance Curriculum Individual Student Planning Responsive Services System Support

What is the best way for a counselor to address a monopolizing student in group?

Have the group examine the student's behavior

What is the HPSO?

Healthcare Providers Service Organization. The HPSO is an insurance company for professionals to protect themselves.

What two assessments are used to assess for an intellectual disability?

IQ test and an adaptive functioning assessment

Ability-Achievement Discrepancy Model

Identifies children who exhibit a severe discrepancy between their IQ and academic achievement as measured by standardized tests. The model is one way to identify students with learning disabilities.

What skills should a peer mediator have?

Identifying common interests Active listening Plan development

When one is confused, explores roles, question authority, struggles w/ social relationship and moral issues *Erikson

Identity Crisis

Concurrent Validity

If the results of a new test instrument correlate closely with the results of previous test, the new test has concurrent validity. In determining concurrent validity, teachers and counselors usually compare a new test to a previous test that is widely used and accepted.

Giving feedback to another person about how you are experiencing them, your perceptions, and feeling in the moment. You are using the immediate situation

Immediacy response

An elementary school counselor is completing an 8-week, unstructured small group counseling program for students who are experiencing emotional hardships. Which of the following would be the most appropriate procedure for the school counselor to use next?

Implementing a plan to monitor the students' progress for positive outcomes

What is a significant difference between a support group and a counseling group?

In a counseling group, the counselor guides the group toward achieving therapeutic objectives. In a support group, the agenda is set more by the members of the group. Support groups are intended mainly to offer support to the members of the group. They are less likely to have therapeutic goals

In assisting students of diverse cultures, counselors in schools

In assisting students of diverse cultures, counselors in schools

Which of the following roles would be appropriate for a peer helper in a high school peer helper program?

Increase students' awareness of available resources

______________ services are provided for students

Indirect student services

What is the IDEA?

Individuals with Disabilities Education Act: requires schools to place students with disabilities in the least restrictive environment possible. IDEA requires that a school notify parents about a change in placement for a student with a disability. Change in placement does not require parental consent.

An advanced sense of inability and inequality in meetings lifes demands

Inferiority complex *Adlerian

A child who has a strong send of industry will be more likely to cope with a low grade on a test. He will understand that he may need to study harder for the next test or seek assistance according to Erickson's theory. On the other hand, a child with a poor school, family, or peer support system may experience feelings of

Inferiority. In this case he is likely to have insufficient mechanisms and not perform will in school.

The following are characteristics of what type of assessment? subjective Not norm referenced Generally Qualitative

Informal Assessment

Sole Possession Records

Informal notes about a student that a teacher or counselors has made for his or her own use.

Action Research

Inquiry or Research in the context of focused effort to improve the quality of an organization and its performance. Typically designed and conducted by practitioners who analyze the data to improve their own practice.

When an individual is motivated by completing the task itself, without additional reward

Instrinsic Motivation *Piaget

SDS SII KOIS are examples of?

Interests assessments

Why would a counselor limit engagement in individual counseling?

It addresses the needs of only one student at a time; therefore, it is the least efficient form of direct intervention.

What is the purpose of a "Self-Directed Search"?

It directs individuals to possible connections based on personality

Americans with Disabilities Act

It is a law that prohibits discrimination based on physical and mental handicaps. Accommodations for individuals taking tests from employment must be made.

What is a school counseling program?

It is comprehensive in scope, preventative in design, and developmental in nature

What does it mean when a consultation approach is "dependent"?

It is when the consulate (teacher) relies on the SC to collect and interpret the behavioral data. Teacher is relying (depending) on school counselor for information

Why should counselors avoid judgmental questions?

It pressures the student to agree with the counselor's view.

Nationally recognized program directed by the Dept. of Labor -At risk teens -provided housing & allowance -vocational training -8months to 2 years -Option to earn GED or Diploma

JobCorps

The following are examples of: -Design strategies for growth of program -implementing effective counseling programs -Sharing beliefs -building linkages w/important stakeholders

Leadership activities

4 themes of the ASCA national model?

Leadership, Advocacy, Collaboration, Systemic Change

School counseling, which began with a focus on vocational guidance, has become more comprehensive in that it now includes what?

Leadership, advocacy and collaboration

School counselors are school and community ___________________

Liaisons

_______________ is the career theorist that emphasized self-concept

Linda Gottfredson

Lesson plans, calendars, and action plans are all part of which ASCA component?

Management

What is true about variability among cultural groups?

Members of the same cultural group may be more different from each other than from members of other cultural groups. It is important to remember when working in the educational system that there may be more differences between people of the same culture than there are between people of two different cultures.

What is the best example of a mentoring program?

Mentoring programs typically include tutoring and recreational activities provided by adults in the community.

The _______ system's setting is the direct environment we have in our lives. Your family, friends, classmates, teachers, neighbors and other people who have a direct contact with you are included.

Microsystem

What does MEASURE stand for in relation to accountability?

Mission (determine mission) Elements (which elements are relevant) Analyze (analyze the data) Stakeholders (unite towards desired goals) Unite Reflect (upon data results) Educate (educate stakeholders on what was found)

_________________ provides the focus & direction of the comprehensive program & aligns with the schools mission

Mission Statement

A student, Beth, is diagnosed with ADHD. Beth struggles with assignments and rarely completes classwork on time. What is the best accommodation for the counselor to recommend first?

Modifying Beth's classroom assignments so that they are more manageable and allowing extended time to complete her classroom assignments. These are appropriate accommodations.

Which of the following pairs of symptoms is associated with steroid abuse?

Mood swings and heightened aggressiveness.

Kohlberg's cognitive theory

Moral thought as important as moral behavior. Young children pay more attention to consequences older children pay more attention to intentions.

Collaborating with parents and community member may result in ?

More funding and additional materials

Simple Random Sample

Most basic type of random sampling; a subset is chosen purely by chance, with no method to ensure that it is representative of the larger set.

Useful counselor responses...

Move student to deeper levels of exploration, understanding and feeling about their experiences

Most significant and controversial educational reform

NCLB (No child left behind)

Who establishes standards for testing under the NCLB?

NCLB requires that states set standards for student achievement in math and reading-not the federal government

In regards to school counseling, NCSC stands for what?

National Certified School Counselor

NOSCA

National Office for School Counselor Advocacy Promotes value of school counselors as leaders in everything from school reform, student achievement, and college preparedness.

What does NOSCA stand for?

National Office for School Counselor Advocacy; NOSCA promotes the value of schools counselors as leaders in everything from school reform, student achievement, and college preparedness.

To create culturally sensitive school counseling programs, which of the following factors should a school counselor consider first?

Needs of diverse populations

If the left-tail is longer the Skewness is?

Negative

What is a pro-bono service?

No charge for person's w/ financial hardships

Toys are used in play therapy to represent what?

Nuturing (express relationships) Aggressive Scary Expressive Pretend

What best provides career assessments that are linked to students' interests, abilities, and values?

Occupational Information Network (O*NET)

2 examples of Career Information Delivery Systems (CIDS)

Occupational Outlook handbook (OOH) Occupational info network (O*Net)

nontraditional occupations are?

Occupations that are held by the opposite sex than which has historically held that role (ex. Woman as a doctor)

Predictive Validity

Offers a good prediction of how well a student will do in a course. The most widely used predictive tests are those that students take when applying to colleges

Abraham Maslow

Often called the father of humanistic psychology. Theory focuses on each individual's potential and stresses the importance of growth and self-actualization.

Dyadic Counseling

One-on-one counseling; as opposed to group counseling

O*NET

Online replacement of the Dictionary of Occupational Titles; uses Holland's codes, values inventory, measures 9 job relevant activities.

Open vs closed groups

Open - new members at any time. Closed - no new members; more cohesiveness

Symptoms of? temper tantrums argumentative purposefully annoying and hurtful mean

Oppositional Defiant Disorder

What are the components for creating a comprehensive guidance program in the school?

Organizing, planning, implementing, and evaluating

Positive Behavior Support

PBS is school wide based, and promotes positive discipline.

Rephrasing what a student says in an example of?

Paraphrasing

Candice visits the school counselor to talk about her plans after graduation. She shares her fears about her family being able to afford college and about moving away from home. She is also unsure of which schools she should apply to. While she is speaking, the school counselor does not interrupt, and when she finishes, the school counselor rephrases what Candice shared. What basic skill is the counselor using?

Paraphrasing - enables the counselor to rephrase a client's comments to verify the client's intended meaning.

Ms. Johnson, a third-grade teacher, is frustrated because one of her students, Jonathan, frequently misbehaves in class. She has approached the counselor for assistance in managing Jonathan's behavior. The counselor decides to use a behavior checklist to make an initial assessment of the scope and frequency of Jonathan's problem behavior. Which people are appropriate to fill out the checklist?

Parent, teacher and counselor.

Bandura's Social Cognitive Theory

People can learn just by observing without receiving any type of reinforcement (bobo doll).

Examples of what kind of assessment? MBTI Vineland

Personality

___________ assessments identify attitudinal, emotional, interpersonal, and motivational characteristics

Personality Assessments

Brofenbrenner's Bioecological Theory

Personality and cognitive change attributed to proximal process- reciprocal interactions between and individual and immediate environment

John Holland

Personality typology theory: six personality types (RIASEC) Realistice, Investigative, Artistic, Social, Enterprising, Conventional. Also known for Self-Directed Search (SDS), My Vocational Situation and the Vocational Preference Inventory.

CBT is best used for?

Phobias addiction depression anxiety

What are the 4 phases of creating a counseling program?

Planning Designing Implementing Evaluation (These phases are all continuous)

If the right-tail is longer the Skewness is?

Positive

Ratio measurement

Possible to compare 2 or more items. These variables can be meaningfully added, subtracted, multiplied, and divided. Has a true Zero. Ex: a graduation rate of 60% is twice as high as a graduation rate of 30%

According to Piaget's theory of development what stage stage can children use representative language, for example they begin to understand that a round toy is a ball and the concept of conversation is understood.

Pre operational stage

Working stage of group description

Productive - mutuality and self-exploration incrase, group focused on making behavioral changes (can rever to initial stage feelings/actions)

assessment of school counseling program that's purpose is to guide future action within the program and improve future results

Program Assessment

Most small groups in schools are

Psychoeducational (both educational and therapeutic)

Name this therapy: Irrational thinking is cause of dysfunction therapeutic counseling relationship ABC Model counselor will challenge assumptions disputing

REBT (Rational Emotive Behavior Therapy) -Ellis

Racial identity is defined as..

Racial identity can be defined as a process or series of stages through which a person passes as the person's attitudes toward his/her own racial/ethnic group are shaped

What does RAG stand for in regard to accountability?

Random Acts of Guidance. Strategic planning works to eliminate RAG, making guidance planned and measurable.

Statistics & Testing - Sampling theory is ...

Random sampling of a population that we'd like to study. If population is large, researchers often turn to sampling. Considerations include: - is sample representative? - what % of error is acceptable?

The relative position, standing, or degree of a specific grouping

Rank ex. GPA Ranking

Original scores that have not yet been converted into a meaningful score

Raw scores

Holland's personality types

Realistic Investigative Artistic Social Enterprising Conventional

Hollands 6 occupational environments

Realistic (physically demanding) Investigative (scientific, mathematics) Artistic (music, art, theatre) Social (interacting with society) Enterprising (lead, persuade, sell) Conventional (office work, clerical, accounting)

Holland test

Realistic, Investigative, Artistic, Social, Enterprising, Conventional

Name this therapy: control or choice therapy responsible living self-defeating behaviors behaviors are attempts to control our environment self-evaluation external locus of control you have control of your life

Reality Therapy / Glasser

William Glasser

Reality Therapy; all behavior is purposeful and to fulfill one of 5 basic needs: Belonging Power Freedom Fun Survival Wants, Direction, Evaluation, Planning (WDEP)

Rebecca has been referred for counseling after numerous reports of spreading rumors about other students and isolating students from their peers. What are her behaviors best characterized as?

Relational aggression, which is aimed at hurting someone emotionally or damaging their relationships.

How can you involve parent's in meetings about their students if they aren't saying anything?

Remind them that they are the experts when it comes to their child

A counselor meets with a student's parents and teachers to address concerns that the teachers have about some of the student's classroom behaviors. During the meeting, as the team plans strategies to help the student be more successful, the school counselor notices that the parents do not provide any feedback. What best demonstrates an effective strategy for involving the parents in the problem-solving process?

Reminding the parents that they are the experts with their child - this helps them understand that they have knowledge and skills that are important to the problem-solving process; this can help them to be less reluctant to provide feedback.

Cognitive Restructuring

Replacing maladaptive thoughts with more functional thoughts *CBT

A 12th grade student is taking a college preparatory curriculum and currently has a 3.4 GPA. The student is interested in biology and writing, wants to attend an in-state college, and will need financial assistance. In identifying which college to apply to, the most appropriate first step for the student to take would be to

Research in-state colleges using the counseling program's online resources.

Research on group guidance indicates that it

Research on group guidance indicates that it

Responsive language

Respectful towards students; uses positive verbal guidance to give reasons and explanations to students about the rules.

Activites involving prevention and intervention fall under ________________ according to ASCA

Responsive Services

When an adult adjusts to a child's level of development

Scaffolding *Vygotsky

Use of multiple reinforcements over time to modify/shape behavior *B.F. Skinner

Schedules of reinforcements ex. behavior contract

What is the most appropriate and effective way for school counselors to prevent burnout?

Scheduling regular downtime each week.

According to the National Career Development Association's guidelines, competencies appropriate for students at all levels include ________.

Self-knowledge, educational and occupational exploration, and career planning. When integrating career development guidelines across various levels, counselors should identify developmentally appropriate goals and interventions that will serve a person throughout the life span.

Cognitive Modeling

Self-talk *CBT

Beginning around the age of 11, children can think in logical, abstract terms. According to Piaget's theory of development, what stage does this represent?

Sensory motor stage. During this time a child acts and learns based on senses and reflexes.

What is "pro bono"

Service provided at no charge for persons with financial hardships

Pro-Bono

Services provided at no charge for person's with financial hardships.

What are the four domains of high stakes testing?

Setting Timing Presentation Response

1997 ASCA published

Sharing the vision: The national standard for school counseling Programs

a measure of the asymmetry of the probability distribution of a real-valued random variable about its mean. The skewness value can be positive or negative, or even undefined.

Skewness

What does SCCT stand for?

Social Cognition Career Theory; Focuses on self-efficacy. It also includes culture, genetic, gender, and social factors that can influence career decisions.

What does SEL stand for?

Social and Emotional Learning; Counselors base curriculum on SEL for all grade levels

Staff at a school distribute a questionnaire to the families of all students at the end of the school year. In evaluating the school's counseling program, it would be most important for program staff to identify the percentage of families who indicate that the

Social and academic advisement needs of their children are being met.

Lev Vygotsky

Social development theory. 1) social learning procedes development, 2) More knowlegeable other- higher ability learner, 3) zone of proximal development - the distance between the students ability to preform by an adult

A fourth-grade student is not participating in class and has declining academic scores. As part of a comprehensive evaluation, the student was administered a personality test, on which the student scored high on introversion. What would most likely help this student?

Social skills training - the student's lack of class participation and introversion indicate the student may have a lack of social skills and a fear of participation that are detrimental to learning.

Samantha is a third-grade student with autism spectrum disorder. During an IEP meeting, Samantha's parents and teacher share concerns about her struggles with organization and social interactions. What strategies will be the most effective for the school counselor to use to support Samantha in these areas?

Social skills training and tactile support since the student has issues of organization and social interactions.

Lev Vygotsky is responsible for creating?

Sociocultural approach / social development theory

SMART goal format (used for writing program goals)

Specific Measurable Attainable Results-orientated Time-bound

Behavioral Objective

Specific and measurable; Describes a behavior that demonstrates a student's mastery of a subject. Taking a course, going to college, ad taking part in some form of community service do not demonstrate mastery of a subject. Having a greater appreciation of classical music is not measurable; therefore, it is not a behavioral objective.

Behavioral anchors

Specific examples of behavior that demonstrate a student's competence in a certain area. For example, a behavioral anchor might be, "demonstrates awareness of other students' feelings"

Counselors can best contribute to the growth and development of fellow school stakeholders by providing what service?

Staff in-service training - this action supports the entire school.

Which group counseling stage takes the least amount of time to complete?

Stage 1 (orientation, forming) is the shortest stage since it is more or less an introduction phase.

ASCA Academic Development Guidance Curriculum Standards

Standard A: Students will acquire the attitudes, knowledge and skills that contribute to effective learning in school and across the lifespan. Standard B: Students will complete school with the academic preparation essential to choose from a wide range of substantial post-secondary options, including college. Standard C: Students will understand the relationship of academics to the world of work and to life at home and in the community.

ASCA Personal/Social Guidance Curriculum Standards

Standard A: Students will acquire the knowledge, attitudes, and interpersonal skills to help them understand and respect self and others. Standard B: Students will make decisions, set goals and take necessary action to achieve goals. Standard C: Students will understand safety and survival skills.

What is a shortcoming of standardized testing?

Standardized testing reflects a student's performance on just one occasion.

Descriptive Statistics

Statistics for describing, organizing, and summarizing a given set of numerical data/data set. These include measures of central tendency and dispersion. Ex: average intelligence of students in a class

Stage of group work when members challenge each other and experience conflict, dominance struggles, and rebellion

Storming

A counselor is facilitating a group and has noticed that there is tension between a few members. One group member monopolizes the meeting and a few other members repeatedly point this out. One member is visibly upset by the negative comments of another group member. Based on the behaviors of the group members, what formative stage is the group going through?

Storming - the stage of group work when members begin to challenge each other and experience conflict, dominance struggles, and rebellion.

What is "storming" in group?

Storming is the stage of group work when members begin to challenge e/o and experience conflict, dominance struggles, and rebellion

Name this therapy: positive psychology enhances resilience Nuturing our best self Combating risk factors identifying past and current successes build on assests personal strengths allows you to grow and achieve focus on inner thoughts turning negative into positives

Strength-based counseling / Seligman

SSPARC

Student Services Personnel Accountability Report Card; Used as a continuous improvement tool for counselors. Used to show a strong commitment to getting strong results with student improvement.

A counselor assumes the role of consultant in relation to all school-related parties except?

Students; A consulting relationships implies a peer relationship that would not be appropriate for counselors and students.

Meta-analysis

Study of studies; it combines the results of several studies that address a set of related research hypotheses. For example, a meta-analysis of studies on standardized test results might reveal strengths and weaknesses that are common over a wide range of student populations.

Romantic relationships between a school counselor and student

Such a relationship is permissible only if the student NO LONGER attends where the school counselor works and NO HARM will come to the student. Such a relationship is NEVER appropriate if the student currently attends the school where the counselor works.

What is the first step in designing a comprehensive guidance program for a new high school?

Survey the needs of students and teachers

Examples of needs assessment tools

Surveys Interviews Data analysis

Most frequently used appraisal method in schools?

Surveys (surveys tend to emphasize the availability of activities, staff, facilitates, and programs.

What is the final stage of therapy?

Termination - when goals have been met, counseling sessions end. The client relationship is terminated and the client is sent to live independent of the therapist.

________________ prevention target a specific population to reduce or eliminate a problem or behavior that is already occurring

Tertiary Prevention

A school counselor wants to examine test results for a test that demonstrates how well students have achieved instructional objectives. What type of test should the counselor use? a. norm-referenced b. criterion-referenced c. content-referenced d. interpretative e. standardized

The Correct answer is: criterion-referenced The counselor wants to find a test that demonstrates how well students have achieved instructional objectives. In other words, the counselor is concerned with the validity of a test. A criterion-referenced test would provide the counselor with the best indication of student performance in relation to instructional objectives.

Internalization is what?

The acquisition of specific and shared knowledge of a culture *Vygotsky

Redirection

The act of transferring the student's attention elsewhere. The attention should be moved to a more positive activity or direction

Mean

The average of scores collected

What range of ages is addressed by Erikson's theory of personality development? a. birth to adolescence b. birth to age 18 c. birth to age 21 d. age 5 to age 21 e. birth to death

The correct answer is: birth to death

When is referral the most appropriate school counselor response to a student's needs?

The counselor cannot meet the student's need b/c of: -time -student's issue is beyond the scope of school counseling setting -addiction -counselor bias

Through a needs-assessment survey, a counselor has learned that juniors and seniors as well as their parents wish they had more information available about the college financial aid process. What would have been the most effective response?

The counselor could develop a slide presentation, placing it on the school's counseling Web page, and notifying all students and parents of its location so they can access the information at their convenience. This is the most effective way to distribute the information.

Content Validity is ..

The degree to which an achievement test contains a representative and appropriate sample of the subject matter (content) contained in the instructional objective whose attainment the test is intended to measure.

Criterion (predictive) Validity is

The degree to which the score on a test predicts the individuals score on a test or performance in some other area. (i.e. if correlated, a test of scholastic achievement can be used to predict job success)

Validity

The extent to which an instrument measures what it is intended to measure

What occurs in the performing stage of group counseling?

The group is working to reach their goals.

What does an interpersonally oriented group focus on?

The here-and-now experiences of all members and the ongoing process among them in the development of their mutual relationships

Median

The middle score

Attribution theory of motivation

The need to feel good about oneself can be a powerful motivator.

What does an intrapersonally oriented group focus on?

The needs and concerns of the individual members and engages the group in interactions that focus on those needs as well, sometimes an entire session devoted to one member's needs

What is countertransference?

The process by which a counselor experiences feelings similar to those of a student because of similar past experiences

What is the range of reliability coefficients?

The range is from 0 to 1. The reliability coefficient cannot be a negative number.

If a school declines to change a student's record that a parent thinks is inaccurate, and a formal hearing upholds the school's position, what further right does a parent have?

The right to place a statement with the record; in this statement, a parent can explain his or her view about the contested information.

In regard to careers; young male students who are entering middle school will most likely be interested in?

The same career as their fathers

What type of careers would young male students who are entering middle school most likely be interested in?

The same career as their fathers - when boys of middle school age are asked about what careers they might like to have, the majority of them will name the career in which their fathers are engaged.

What is Pedagogy?

The science and art of education, it mainly refers to instructional theory.

The establishment of the National Vocational Guidance Association was the beginning of

The unification of the counseling profession.

Nativist Theory (Noam Chomsky)

Theory of language development, argues that all children have an inborn understanding of how language works.

Constructivism

Theory of learning which argues that humans generate knowledge and meaning from their experiences and build constructs.This theory has led to teaching methods that stress a hands-on approach to learning.

A young child may not understand consequences, according to Cognitive Behavior Theory because

Thought at this point is concrete, in order to fully understand consequences, one must be able to understand fight from wrong, as well as be able to visualize what future consequences will come about by present actions. In most ashes, an adolescent will understand these consequences because of their ability to think abstractly.

What tier of RTI's can "Check in, check out plans" be found?

Tier 2; through this plan, the student uses not only their school counselors for support but also people at home

A middle school counseling department is working with administrators to evaluate the program and make improvements. What accountability tools would be the most effective for them to use?

Time assessments and a school data profile analysis are the most effective accountability tools for program evaluations and program improvement.

Intended to increase academic achievement by improving teacher and principal quality

Title II

No person in the United States shall, on the basis of sex, be excluded from participation in, be denied the benefits of, or be subjected to discrimination under any education program or activity receiving federal financial assistance.

Title IX Education Amendment of 1972

Students are protected from discrimination on the basis of gender according to

Title IX of the Educational Amendments of 1972.

Role of school counselor

To design and implement a comprehensive program of services with specific goals and objectives that complement the broader mission of the entire school.

Strategic Planning in regards to accountability

To get measurable results of school improvement. 6 steps: Data Goals Planning Implementation Results Sustain

behavioral consultation is an example of?

Triadic-dependent

T or F: A score below 80 on IQ test indicates a developmental disability?

True

T or F: Analyzing trends in GPA is an appropriate activity for School Counselors?

True

T or F: IQ tests and measures of adaptive functioning go hand-in-hand?

True

T or F: Scores of 2 standard deviations above the mean are classified as "very superior"?

True

Erik Erikson's 8 Stages of psychosocial development

Trust vs. Mistrust Autonomy vs. Shame and Doubt Initiative vs. Guilt Industry vs. Inferiority Identity vs. Role Confusion Intimacy vs. Isolation Generativity vs. Stagnation Integrity vs. Despair

Relational aggression

Type of aggression that causes harm to another student through bullying. The bully will cause damage to another student's reputation, social status and/or relationships. Consequences may be: low academic performance, lack of close relationships, depression, and low school engagement. Also known as Covert Aggression. Relational aggression is always deliberate and hurtful.

Grade/age equivalences

Type of developmental score that compares a student's raw score with those of a certain age/grade ex. 5.3 --> 5th grade, 3rd month

stanine

Understandability of percentages with the properties of the normal curve of probability. Stanines divide the bell curve into nine sections with largest at the 40-60th percentile.

What is Convenience Sampling?

Using a sample that is readily available.

Empathy means...

Using active listening to sense what a client feels without saying so, or reading clients' emotions even when they are unspoken (not interpreting - which is putting what the counselor feels/thinks onto the client)

Tim has been smoking marijuana daily before he comes to school. He told his peers this helps him come to school and focus on his assignments for the day. What are his actions an example of?

Using drugs or alcohol to get through the day or to function normally is a sign of substance abuse.

What are the 3 types of bullying?

Verbal Physical Relational

Confronting involves..

Verbally holding apparently discrepant or incongruent aspects of clients' messages and behaviors "in front of" clients for them to see. Helping clients clarify, resolve, or accept the discrepancy

On an IQ test with a mean of 100 and a standard deviation of 15, a score of 130 and above is placed in which classification?

Very superior Scores of two standard deviations or more above the mean are classified as very superior.

In Erikson's 8 stages of psychosocial development, each stages is connected to a _______________

Virtue

What are the three different client therapy relationships in SFBT?

Visitor Complainant Customer

Zone of Proximal Development (difference between what a child can do w/assistance and w/o assistance)

Vygotsky

Most commonly used IQ test?

WISC

Questions used to guide program goal review?

Was the goal a SMART goal? Was the goal a closing-the-gap goal? Was the goal met? How? Why not? Implications for goal setting next year?

Learning Theorists

Watson, Skinner, Bandura, Pavlov - development is observable behavior that can be learned through experience with the environment

Reliability

Way of measuring whether a test produces consistent results. If a test produces similar results every time it is given, it is said to be reliable.

Goals for counseling programs should clearly state what?

What is needed from students, counselors, parents, teachers, and staff. If these groups know what is expected of them, the goals will have a higher likelihood of being fulfilled.

When should counselors use the ignore strategy?

When a behavior is not dangerous, disrespectful, or hurtful. Whining or efforts from a student to try and make an adult angry should be ignored.

What is a token economy example?

When a teacher uses a secondary reinforcement that the students can collect to purchase another reinforcement

In regards to physical development, what time period does toddlerhood include?

When child begins walking to age four; Childhood is from birth to puberty Early childhood is from birth to toddlerhood

When is the "Wechslers Intelligence Scale for children" acceptable to use?

When comparing student's scores w/ those of the general population

When is consultation essential to success?

When creating a comprehensive school counseling program - it would be impossible to create and maintain a comprehensive school counseling program without consultation with and input from multiple stakeholders, such as teachers and other school staff, parents, and community stakeholders.

As data becomes more symmetrical, its skewness value approaches zero.

When data is not heavily to one side or the other

When is an assessment considered to be standardized?

When it is administered with consistent, set procedures.

When is an assessment is considered standardized?

When it is administered with consistent, set procedures. Can be created for group or individual use, but does not have different scoring systems for different setting and can be scored with various scales.

Collaboration with outside providers

While it is recommended that school counselors collaborate with other service providers for the benefit of the student, ethical behavior requires that consent first be obtained (need to get written informed consent from parent or student over 18)

When addressing the issue of counselor evaluation, a critical question for the school counseling profession is:

Who will evaluate counselors?

Creator of reality therapy?

William Glasser

Beneficence

Working for the good of the individual and society by promoting mental health and well-being.

According to Lev Vygotsky, what is ZPD?

Zone of Proximal Development; the ZPD is the area where the most sensitive instruction should be given to a child.

Ambivalence (feeling)

a : continual fluctuation (as between one thing and its opposite) b : uncertainty as to which approach to follow

Clifford Beers was instrumental in the mental health movement as

a hospitalized patient who wrote the book A Mind That Found Itself.

In cases of students from divorced parents, both the custodial and non-custodial parent have legal right to see their child's school records unless

a judge rules otherwise.

Define variance

a measure o the spread of the values in a distribution. The larger the variance, the larger the distance of the individual cases from the group mean.

Statistics & Testing - Central tendency is ...

a measure of the point about which a group of values are clustered.

Statistics & Testing - Correlation coefficients mean ...

a numerical value that identifies the strength of relationship between variables

"Hold one individual conference with each student during the year" is an example of

a service-related goal.

norms

a standardized sample against which all other scores are assessed

Licensure/certification for school counselors is

a state's responsibility

T scores are ...

a test score converted to an equivalent standard score in a normal distribution with a mean of 50 and a standard deviation of 10

What is operant conditioning? What are the types?

a type of learning where behavior are controlled by consequences? a) Positive reinforcement - giving something pleasant after a behavior i.e. compliment b) negative reinforcement - taking something away unpleasant i.e. exempt from final if you have perfect attendance c) positive punishment - presenting something unpleasant after a behavior to decrease behavior i.e. misbehaving = get a time out d) negative punishment - removing something pleasant after a behavior i.e. habitually late for work = losing music privileges

IDEA

academic institutions are financially responsible only for testing that they request. Parent requests are evaluated on case-by-case basis, and typically the financing is the parents' responsibility

What are the three domains in a comperehensive counseling program

academic, career, personal/social

Title II of the Education Amendments

addresses teacher and principal training, and Enhancing Education through Technology

School counselors should establish/organize ________________ councils

advisory councils

Benefits of professional organizations

advocacy, professional development, lobbying efforts, liability insurance, and networking, accessibility to resources and current research, and leadership development.

Negative punishments are removed _________________

after the response *B.F. Skinner

In cases of suspected child abuse,

all school personnel are required by law to report.

Empathy responses

always reflect something the client is experiencing and would recognize

Ensuring adequate and appropriate interpretation of test results to students, parents, and teachers is

an essential responsibility of the school's testing coordinator.

School leaderships teams do what?

analyze academic data establish school-wide goals & objectives

A primary competency for school counselors who wish to be effective in counseling diverse students is to:

become aware of their own cultural values and biases

In assessing children's potential to be successful in counseling relationships, counselors assess students'

behavioral and cognitive development

Individual planning

best described as the broad collection of counseling and guidance activities that are designed to help all learners develop their own educational plans and career objectives.

Program assessment that attempts to measure the results of counseling services is

called outcome evaluation.

External evaluations of school counseling programs

can complement internal evaluations and avoid parochial perspectives.

Assessment of school climate

can produce findings that help counselors and schools select appropriate services.

Classroom observations by school counselors

can provide information to evaluate student learning and behavior

Career development at the middle school level is?

career exploration

Career development at the high school level is?

career planning

Experiential group (description)

centered perhaps on a topic, such as grief, does not have an objective or lesson, but is solely process orientated.

Ways to score criterion-referenced assessments

checklists rubrics grades

preoperational stage

children begin to engage in symbolic play and learn to manipulate symbols

Wechsler Intelligence Scale for Children - 4th Edition (WISC - IV)

comparing students' scores with those of the general population and identifying learning disabilities and giftedness. Grade retention decisions should be based on academic achievement, not intelligence test scores.

Title IX of the Education Amendments

comprehensive federal law that prohibits discrimination on the basis of sex in any federally funded education program or activity (applies to all schools and colleges that are funded) also protects from sexual harrassment and employment discrimination

Used to measure new tests or procedure against a proven benchmark

concurrent validity

etic approach

considers characteristics presumed to be shared by all cultural groups, such as identity development, acculturation, and social class differences

Developmental counseling

considers important life tasks in the counseling process.

Didactic in terms of group process is defined as _____

consisting of reading assignments, lectures, and group discussions.

Together the ___________ & ______________ define the problem, establish goals, and create and implement. Then evaluate the plan of action

consultee and consultant

Concurrent validity

correlating test scores with scores on a criteriation measure obtained at the same time, important for personality tests

Virtual reality technology

could be a possible tool to assist in counseling and guidance.

Future technology

could have professional and practical implications for school counseling programs.

____________________ informs decisions about the counseling program. Informs counselors of student needs and community trends

data analysis

Behavior therapy interventions

desensitization, aversive conditioning, behavioral chaining, and classical conditioning.

What is "Kuder Career Planning System" used for?

developing portfolios, making educational plans, and taking career assessments?

In-person interactions with students

direct student services

Used to compare scores on tests of unrelated subjects

discriminant validity

appraisal and Advisement are strategies that are used when?

during individual student planning

Empathic (feeling) v. Sympathetic

empathetic - id affective part of client's message not stated, shows awareness of the client from part of counselor/listening sympathetic

Facilitator's role in groups

encourage group development of expression, exploration, and self-disclosure, to discourage dynamics that hinder expression and openness and to model healthy, appropriate interpersonal communication

Family Systems Theory

encourages people to think of issues, like marital conflict, addiction, acting out teenagers, difficult relationships, loneliness in terms of a multigenerational family or a system. The approach enchoranges people to move away from blaming others and towards individual responsibility

Reliability of test results is

essential for a test to have validity.

A first step in designing a comprehensive program of services is to

examine the current program that is in place.

Classroom guidance, parent education programs, and teacher workshops are

examples of instructional consultation.

Survey instruments that assess the satisfaction of students, parents, and teachers with a school counseling program are

examples of perceptual measures.

Authoritarian style group leader

expert leader promotes his/her own agenda (teacher role)

Incremental validity

extent to which a test increases information above existing forms of assessment

504 of ADA

federal law designed to protect the rights of individuals with disabiites in programs and activites that recieve Federal finiancial assistance from the U.S. Department of Ed. - no exclusion from the participation in, be denied the benefits of, or be subjected to discrimination in any program or activity. Must recieve a free and appropriate education

reflection of feeling

feeling must include a feeling or affective word and must be something the client is feeling, not the counselor

What is the "termination" stage?

final stage of therapy

Informational services can include information about community resources, career and educational opportunities, and

financial assistance sought by students for college.

preconventional stage

first level of Kohlberg's moral development taxonomy where individuals' ethical principles are guided by the consequences of the action and the benefit awarded to him or herself

Immediacy responses

focused on the present, right at the time and place and context of the counseling session. They may be statements or questions, but must be about the present moment, not the past or future.

ASCA National Model four pillars

foundation, delivery, management and accountabilty

Planning, Organizing, Implementing, and Evaluating are

four phases of a comprehensive program

Inclusion classroom

general ed classroom

Establishing a relationship, exploring concerns, taking action, and ending the relationship are

general phases of a counseling relationship.

Interpretations

generated within the counselor so they are new to the client

what should be set before choosing members of an advisory council?

goals and objectives

One difference between group counseling and group guidance is that

group guidance activities can be presented to large audiences.

A "control group" research format attempts to demonstrate

group management skills of a counselor.

Homogenous group counseling

group members share a variety of common characteristics

inter-organizational

groups of diverse organizations

classroom guidance

guidance has its own curriculum and is not a way to substitute for the regular classroom teacher. Classroom guidance is a form of developmental preventive mental health and will help reduce the need for individual and other remedial interventions and is no less valuable then other forms of counseling intervention. Content for classroom guidance is intended for delivery by school counselors, although they may team teach with the classroom teacher. Within comprehensive models, classroom guidance is considered to be an integral part of the overall school curriculum, with identified competencies and learning outcomes for all students.

Parental involvement in schools

has been related to higher student success in school.

sensorimotor stage

having or involving both sensory and motor functions or pathways

A school counseling program is to be established in a new public school that has just opened in an expanding school district. The first step in developing the program is to (A) survey the guidance and counseling needs of the student body (B) devise behavioral objectives for classroom management (C) decide on minimum competency levels for student progression to higher grade levels (D) gather appropriate counseling and guidance materials, such as tests and occupational information

he best answer is (A). Survey approaches used to determine counseling needs in the school are valuable for the completeness of the data collected and the coverage provided. Surveys are probably the most frequently used appraisal method in the school. Survey approaches tend to emphasize the availability of activities, staff, facilities, and programs.

Sociograms

help teachers and counselors assess student relationships.

A school counselor's portfolio

includes many different methods of data collection.

One factor contributing to the school counseling profession's difficulty in establishing a clear identity has been

inconsistent use of terms to identify who school counselors are and what they do

Referrals, consultation, and collaboration are examples of _______________ services

indirect student services

Services provided on behalf of the student as a result of the school counselors interactions with others

indirect student services

Consulting relationships for counselors in schools include problem-solving, informational, and

instructional services.

Methods of appraisal

interviews, rating scales, behavioral observations, formal published inventoies, sociograms, test batteries

unconditional positive regard (carl rogers)

involves showing complete support and acceptance of a person no matter what that person says or does.

The DSM-IV-TR

is a diagnostic manual for classifying mental disorders.

An advisory committee for a school counseling program

is a vehicle through which counselors obtain input about the program.

Guidance Advisory Committee (GAC)

is commonly charged with the task of reviewing program data and making recommendations based on their evaluations. GACs typically do not evaluate counselors' professional activities, report directly to administrators, or disclose counseling work to the media.

internal consistency (reliability coefficients)

is important for concurrent measurements

primary value of needs assessment

is providing the counselor with an empirical basis for establishing goals and objectives

management system

is the when( calender and actin plan) why ( use the data) and on what authority( management agreement and advisory council)

Apathy (feeling)

lack of feeling or emotion

A major historical event that created an atmosphere for propelling the school counseling profession was the

launching of the Soviet Sputnik I in 1957.

FERPA

law stipulates that parents and students over 18 have the right to view all of the official records at any time. It also stipulates that access by others be limited to those in an official capacity requiring access to this information for the benefit of the student.

Americans with Disabilities Act

laws that prohibit discrimination based on physical and mental handicaps. Accomidations for individuals taking tests from employment must be made.

Laissez faire style group leader

leader is more of a member

Local school board policies are

legal guidelines for school personnel to follow.

School counselors have a _______________ obligation to parents and _______________ obligation to students

legal obligation to parents and ethical obligation to students

Content validity

looking at the tests content, mostly acheivement tests

The process of socialization by accepting the cultural values of the larger society is:

marginality

Animus

masculine side

validity

means the test measures what it is designed to measure

What is a test battery?

measure of a wide range of aptitudes and is used in such areas as occupational selection, rehabilitation, and vocational counseling,

predictive validity

measures of how well a test predicts future performance

Which measure of central tendency is most resistan to extreme scores?

median

1990 ASCA

moved to call the profession school counseling and the program a school counseling program

Collaborative models are

mutual, shared, and reciprocal

Collaborative models are usually...(3 things)

mutual, shared, and reciprocal

_______________ assessments should include input from students, parents, teachers, and community stakeholders

needs assessments

**How an the counselor intervene with parents who discipline of their children is ineffective?

negotiate carefully based on your judgment of parents' ability to accept information. You can offer more information on what might be effective working with a kiddo ... particularly if it is working at a school. Remember that some parents are doing all that they know / the best they can. Be careful not to exercise judgment.

Tests that report a student's relative standing in a sample population are referred to as

norm-referenced tests.

Because individual counseling relies largely on the verbal skills of both the counselor and client, it is

not an appropriate service for every student.

Ways to collect data...

observation, interviews, performance based assessments, and behavior assessments

A first step that counselors take in accepting a referral from teachers to counsel a student is to

observe the student in the classroom.

Ethical standards for the school counseling profession

offer a general framework for professional practice and responsible behavior.

A process group is ...

one that has a trained counselor or facilitator who can lead the group in accomplishing personal growth

Two types of group structures are

open groups and closed groups.

One procedure counselors use in selecting tests and inventories is to

order and examine specimen sets.

In-house collaboration teams may include?

peer-helping programs student office aids

Intra-organizational collaboration

people from the same organization

Erich Fromm was a ____________ theorist

personality theorist

comprehensive programs provide

placement, follow-up, and follow-through activities to assist students with their next steps. Direct delivery of counseling and other guidance activities on a demand basis is included because there may be a need for direct immediate services to students while they are still in the school building

Aptitude tests

predict future performance or ability to learn

Educational group (description)

prescribed course for lessons and process as a goal

Classroom guidance lessons are often used as a ________________ measure

preventative measure

The term used to indicate that a person is protected from having confidential information revealed in a public hearing or court of law is

privileged communication.

# of participants, # of times an intervention took place what type of data?

process data

____________ data describes the way activities are conducted and how many students were affected by the activity "what?" "for who?"

process data

applications of Vygotsky

promotes learning contexts in which students play an active role in learning. Teacher should collaborate with his or her students in order to help facilitant meaning construction in students. Learning is a reciprocal experience

Achievement test

provide information about an individual had learned or acquired

Late 1950's and early 1960's National Defense Education Act

provided funds to train school cousnelors

An important effort in transforming the school counseling profession in the twenty-first century is the

publication and promotion of the ASCA National Model®

Correlation coefficients

range from -1.00 to 0.00 to +1.00

Projective techniques in student appraisal are

rarely, if ever, used by school counselors.

Individual counseling is often ____________________, not __________________

reactive, not preventative

Competent multicultural counselors

recognize their biases and personal readiness to engage in multicultural counseling

The National Board of Certified Counselors (NBCC)

recommends eight core areas of knowledge as the base of study for all specialties in the counseling profession.

parallel forms (reliability coefficients)

reducing practice effects and for predictions

Experiential in terms of group process is ...

related to personal experience and an individual's reality within the group process.

ASCA accountability

responsibility for ones actions, particularly for ojectives, procedures and results for ones work and program; involves an explanation of what has been done. Responsibilites for counselor performance, program implementation and results.

The ethical code for school counselors suggests that counselors are

responsible to both their clients and the institutions that employ them.

A major barrier to public recognition and acceptance of school counseling as a profession is

school counselors' avoidance of program evaluation and professional accountability.

Evaluation in a school counseling program

serves the purposes of generating valid measures of how a program meets goals and objectives, helping others understand the role of the counselor, and gathering data for research purposes. It is not used to plan and develop procedures when the program is unable to change

A difficulty in organizing group counseling in schools is

setting a schedule for group sessions.

While the research does not support any particular approach to counseling, there is some indication that a crucial aspect of successful counseling is the

skillfulness of the counselor.

Genuineness, empathy, positive regard, and concreteness are

some of the "core conditions" of effective counseling

Some studies have indicated that high school counselors

spend much time in clerical and administrative tasks.

Represent the distance between a given score and the mean and are used in norm referenced assessments to compare a student's score to their peers

standardized score

Merging specific core academic standards with counseling standards

standards blending

Websites, brochures, and speaking engagements are

strategies for marketing counseling services.

abroad term for student achievement

student success

Advantages of using small groups in counseling setting

students experience groups as a familiar and natural setting for interacting, groups also provide opportunities for students to hear from and learn about others with similar experiences, groups allow more students to be served at one time by the counselor, and allow less active students opportunities to benefit by watching and listening to others

The intent of due process, as provided by the 14th Amendment to the US Constitution is to protect

students from actions and regulations that are inherently unfair.

Transition plans are required for _________________

students with IEP's

What individuals should provide input through a needs assessment?

students, parents, teachers and community stakeholders.

Construct validity

systematic study of the adequacy of the test in appraising a specified psychological construct, personality tests

Krumboltz theory is known for teaching?

techniques for career making decisions

stanines

test scores converted to an equivalent standard score in a normal distribution with values 1, 2, 3, ...9 a mean of 5 and a standard deviation of 1, 96.

Pavlov's Theory of conditioning responses states

that when an action is repeatedly followed by a response, the one performing the action will either continue or stop doing do depending on the response.

Z Scores are ...

the # of standard deviations above or below the mean

% ranks

the % of people in the norming sample who had scores less than or equal to the students score

Today the American School Counselor Association is an independent division of this national organization:

the American Counseling Association (ACA

The federal bill that provided funds to train counselors for secondary schools during the late 1950s and early 1960s was

the National Defense Education Act (NDEA).

Explain Carl Rogers "congruence"?

the ability to be authentic and genuine w/ a client. When a counselor's thoughts, feelings, and behaviors are not at odds w/ e/o.

Define mean.

the average value in a data set it is determined by adding all the values & dividing the sum by the # of values in the set.

One factor that guides a school counselor's decision to refer a student to another professional for counseling is

the counselor's knowledge and training in the specific area of concern.

Define deviation

the difference between an observed value and the expected value of a variable or function

Statistic & Testing - Variability is defined as ...

the dispersion, spread, or scatter of scores or values in a distribution, usually about the mean

Statistics & Testing - Standard error of measurement is ...

the estimate of the 'error' associated with the test-takers obtained score when compared to their hypothetical 'true' scores.

Cultural validity

the extent to which instruments consider worldviews that differ from the dominate culture

Racism is defined as ....

the inherent belief in the superiority of one race over all others & thereby the right to dominance

Democratic style group leader

the leader collaborates w/ members

Define median.

the measure in central tendency that occupies the middle position in a rank order of values. Generally has the same # of items above it as below it.

resilient to extreme scores in a distribution

the median

Individualism is a philosophy that believes

the person is discrete from other beings.

Define mode.

the single class/value that occurs most frequently in a series of numbers

The spread of students' scores across the group is measured by

the standard deviation.

Ethnocentrism is defined as ...

the tendency of most people to use their own way of life as a standard for judging; now also indicates the belief, on the part of most individuals that their race, culture, society, etc. are superior to all others.

In developmental school counseling programs, the goals of a guidance curriculum link to

the therapeutic goals of counseling services.

What consultation model should you use when doing behavior consultation?

the triadic-dependent model (counselor, teacher, student)

The term, worldview refers to:

the way students and others give meaning to life.

foundation is

the what of the program

__________ & _______________ are the most effect accountability tools for program evaluations and improvement

time assessments & school data profile analysis

The decision of whether of not to use individual counseling in a school counseling program is related, in part, to the

time available to provide this service.

Discounting (feeling)

to minimize the importance of

Learning-related goals are selected

to target specific learning objectives for students.

Redirection of a clients feelings to the therapist

transference

true or false: adolescent girls mature more quickly than adolescent boys

true

true or false: self mutilation, anorexia, and bulimia are treated as imminent risk

true

When counseling students and families of diverse cultures, school counselors

understand the importance of language in the helping process

The school counseling profession has its origin in the

vocational guidance movement in the early 1900s

What is "convenience sampling"

when a sample is chosen based on availability or convenience.

test-retest (reliability coefficients)

when the test will be used to make a prediction

When setting objective you should be able to answer

who? how? what?

In regards to collaboration, viewing youth as experts and partners who share responsibility and accountability for results is an example of?

youth-centered collaboration

ecological family theory

Argues that you cannot understand or predict behavior by looking at the individual in isolation. -->the social up-brining of the child is very important but you can't just focus on the immediate environment you must focus on the whole environment -->The relationship between family members is what is important. There is a basic family structure and when this is disrupted then behavior problems occur.

Aversive Conditioning

Behavior therapy technique; client is exposed to an unpleasant stimulus while engaging in an undesirable behavior. Goal is to create an aversion to the behavior.

What is an example of a management activity?

Designing a comprehensive developmental guidance program - there is a difference between managing a school counseling program and performing those duties.

Indicators of physical abuse

Destructive, aggressive, or disruptive behavior Fear of going home and being withdrawn

IDEA

Federal legislation related to the education of students with disabilities This law includes the requirement of school counselors to participate in IEP planning process and meetings.

Interval measurement

Items are ranked, and the interval between items is significant. "Interval" itself means "space in between," interval scales not only tell us about order, but also about the value between each item. Interval scales are numeric scales in which we know not only the order, but also the exact differences between the values. The classic example of an interval scale is Celsius temperature because the difference between each value is the same. For example, the difference between 60 and 50 degrees is a measurable 10 degrees, as is the difference between 80 and 70 degrees. Time is another good example of an interval scale in which the increments are known, consistent, and measurable. Interval measurement does not have a "true zero." For example, there is no such thing as "no temperature."

Theorist that blended social learning with career development

John Krumboltz

What is IDEA?

Law that ensures the equality of students identified with a disability 1. physical or neurological 3. Sensory 4. Developmental

Daniel Goleman

Leading exponent of emotional intelligence theory. Describes emotional intelligence as the ability to understand and manage emotions in self and others.

Applications of Vygotsky

Learning is a reciprocal experience; students play an active role in learning and teachers should collaborate with their students in order to facilitate meaning in students.

What does LRE mean?

Least Restrictive Environment; (IDEA) This often means that students are placed in a regular classroom and receive assistance from an aide.

IEP students should be mixed with general ed students as much as possible and be placed in the _______________________________

Least restrictive environment

In their role as consultants, counselors should not be involved in which type of intervention?

Legal intervention

What theorist believes that thoughts, language, and reasoning development are contextual and driven by cultural, environmental, and social interactions

Lev Vygotsky

Lawrence Kolhberg's LEVEL's of moral development

Level 1 - Preconventional (obey rules to avoid punishment & gain reward) Level 2 - Conventional (Actions are intended to gain approval of others) Level 3 - Post Conventional (Rules are important, but decisions are made based on individual values/beliefs)

Recognizing when student problems must be addressed at a policy or legislative level is an example of micro or macro advocacy?

Macro advocacy

School Counseling Advisory Committee

Made up of school counselors, parents, teachers, staff, and community members. The committee reviews and makes recommendations about the school's counseling program

NCLB requires schools to disaggregate test results by?

Major racial and ethnic groups Students with disabilities Economically disadvantaged students Students with limited English proficiency Students from migrant families

A teacher reports to the counselor that she overheard a student tell a friend that he is going to kill himself. What should the counselor do first to ensure the student's safety?

Make sure the student is supervised and not alone. The first priority is to ensure the student's safety by ensuring supervision. After that, the counselor can assess whether the student is indeed suicidal and initiate preventive measures.

______________ suggests that the most basic human needs must be met before an individual will actively seek out ot be motivated towards higher levels of need

Maslow

What are three measures of central tendency?

Mean, median and mode.

Measures of central tendency

Mean, mode, median

Statistics Testing - Testing norms ...

Norm referenced tests compare a person's score against the scores of a group of people who have already taken the same exam, called the "norming group"

Types of Standardized Tests

Norm-Referenced Tests (GRE) Criterion-Referenced Tests (ACT)

Standardized Testing

Norm-referenced and criterion-referenced. An advantage to standardized testing is that is gives quantitative information. Disadvantages are that he results do not provide information for making instructional change, test questions are not related to what is taught, and they are influenced by non-cognitive factors.

A school counselor tell the parents of a 5th grader that their child's test scores reflect the child's performance relative to that of other 5th graders across the United States. This type of score is referred to as a

Norm-referenced score

What does NCE stand for?

Normal Curve Equivalents NCE are uniform measurements

stage of group work when the group begins to be effective. Focus shifts to "How can I help the group?"

Norming

What limitation in testing occurs because of state test guidelines having too lengthy of a list of items to test?

Not all state requirements are actually tested. State test guidelines have a lengthy list of rules and testing materials to follow and use. Since the amount of information is extensive, students are not fully tested on all of the information set by state guidelines.

What type of data would document time devoted to the delivery component of an effective counseling program?

Number of consultative services provided to colleagues and parents. This delivery component is directly out of thr ASCA model.

"Above all do no harm" Definition of?

Nonmaleficence (1 of 5 ethical principals)

___________ data shows the impact of an activity or program and answers the question, "so what?

Outcome Data ex. Discipline referrals decreased by 30% for student with 4 or more referrals

Impact of interventions, improvement in achievement, attendance, behavior what type of data?

Outcome data

_________________ prevention is aimed at mediating a specific behavior or problem identified as potential threat for a specific population that is deemed at-risk

Secondary Prevention

13 performance standards school counselors are accountable for

Plans, organizes and delivers the program, implements guidance curriculum through instruction and careful planning, implements individual planning components, provides responsive services, provides system support, discusses counseling department management system, is responsible for establishing an advisory council, collects and analyzes data, mentors students, uses time and calendars to implement an effective program, develops a results evaluation under the counseling program, conducts a yearly program audit, and advocates for students.

________________ is used to identify concerns, but also show how goals have been attained and how counseling makes a difference

Program results data types of results data: process perception outcome

SOAP notes - often used to organize case notes

S- subjective observations O- Objective observations A- Assessment P-Plan (treatment strategies)

Accountability

S.C. Performance Standards Action Plans Results Reports CCGP Performance Review Research Assessment

This career assessment helps identify preferences for activities, matching personality type with 6 workplace environments

SDS

Holland created which career interest inventory?

SDS (Self-Directed Search)

This career assessment provides info about careers and matches codes

SII

_________________ is a representative group of stakeholders selected to review and advise on the implementation of the school counseling program.

Advisory Council -Should meet at least twice a year -maintain agenda and minutes

Efforts to identify external barriers and developing strategies in response to barriers, is an example of?

Advocacy

What are some of the benefits of professional organizations?

Advocacy, professional development, lobbying efforts, liability insurance, networking, accessibility to resources and current research, and leadership development

Howard Gardner

Theory of multiple intelligences; according to Gardner, the concept of IQ describes only a limited range of intelligence. He proposed that other types of intelligence, such as musical intelligence or interpersonal intelligence, are also important and can bee exhibited in many ways (e.g. linguistically, mathematically, and spatially). Explains the diversity of human achievements. An individual can possess more than one type of intelligence at exceptional levels, and no one type of intelligence is superior to others. Believes that standardized tests can have value, but they do not address the full range of a student's intelligence.

How long should a counselor keep records of counseling with students?

There are 3 classes of data. Records of counseling falls under "class C" data - this type of data should be removed or destroyed as soon as usefulness has expired - maximum of 5 years is standard.

The ACA code of Ethics created how many main purposes?

There are 6 main purposes and 9 sections (counseling relationship, confidentiality, professional responsibility, relationships with other professionals, evaluation assessment and interpretation, supervision training and teaching, research and publication, distance counseling, technology and social media) that address different areas of ethics.

Example of when in consultation "dependent"?

When the consultee (teacher) relies on the consultant (counselor) to collect and interpret behavioral data

What is a "prescriptive" mode of consultation?

When the counselor acts as an expert to help teacher

Oppression is defined as...

an unjust & systemic excessive exercise of power against an unidentified group of people, such as disable people where laws, attitudes towards & treatment (including portrayal) of this group all reinforce this discriminatory situation.

________________ is at the heart of having a data-driven school counseling program

analyzing results

Indignation (feeling)

anger aroused by something unjust, unworthy, or mean

504 with assesment

any instrument used to measure appropriateness for a program or service must measure the individual's ability, not a reflection of their disability

By using monthly report forms that indicate how many students received counseling, how many consultations with parents were held, and how many group sessions were presented, school counselors

are able to quantify how they spend their time.

Needs assessments

are an essential part of program planning.

The text in this course takes the position that comprehensive counseling programs at the elementary, middle, and high school levels

are similar because the overarching major functions of school counselors are similar,

The differences between counseling and therapy

are still debated among helping professionals.

In devising a schedule of services, school counselors

are wise to seek suggestions from their teaching colleagues.

Holland's theory of career development:

maintains that in choosing a career, people prefer jobs where they can be around others who are like them. They search for environments that will let them use their skills and abilities, and express their attitudes and values, while taking on enjoyable problems and roles. Behaviour is determined by an interaction between personality and environment. There are six basic types of work environments: Realistic, Investigative, Artistic, Social, Enterprising, Conventional. People search for environments where they can use their skills and abilities and express their values and attitudes. For example, Investigative types search for Investigative environments; Artistic types look for Artistic environments, and so forth. People who choose to work in an environment similar to their personality type are more likely to be successful and satisfied.

School counselors should create methods of accountability in their programs

to demonstrate the value of their services and professional credibility.

School counselors who provide counseling services to parents and teachers

use their best professional judgment and ethical standards to determine if they are the appropriate person to provide these services.

Collaborative approaches to consultation are the most _____________________________

used methods in schools

One advantage of group counseling is that it

uses controlled peer pressure to encourage students.

Interactive style group leader

uses experiential learning

The term ethnicity:

usually refers to commonality of shared cultural views

According to the National Career Development Association's guidelines, competencies appropriate for students at all levels include...

Self Knowledge Educational and occupational exploration Career planning

What are the three components of self concept?

Self worth Self image Ideal Self

Which assessment uses Holland's codes to assist students in identifying possible occupations and majors based on personality?

Self-Directed Search Form R: 4th Edition (SDS R)

What would a student with Down syndrome most likely exhibit?

Students with Down syndrome most commonly exhibit impairment in cognitive function.

Cognitive Behavior Theory

Suggests that as children grow into adolescent's, they begin to form abstract thoughts. This is the ability to think about things that cannot be seen. This is an important part of youth development because as abstract thought develops, oral thought increases.

change affecting the entire system; the focus of change is upon the dynamic of the environment, not the individual

Systemic Change

Behavioral technique commonly used to treat fear, anxiety disorders and phobias. Using this method, the person is engaged in some type of relaxation exercise and gradually exposed to an anxiety producing stimulus, like an object or place.

Systemic Desensitization

Leadership, advocacy, and collaboration are key strategies that are needed to create _________________

Systemic change

Supportive Response

Acknowledges the student's problem and offers to find help.

A plan that details how the counselor intends to achieve desired results

Action Plan

What are the 6 techniques used in group therapy?

Active listening Cutting off/blocking Eye contact Dyads Leadership role End of group comments

Four elementary school students have been meeting regularly with a school counselor. The counselor uses an activities based approach to foster the students' development of social skills. Which of the following actions by the counselor would most likely have a disruptive effect on current group processes?

Adding a new member to the sessions.

A counselor in a culturally diverse school makes every effort to increase her awareness of traditional family relationships in different students' home cultures. How is this practice useful for the counselor?

Adjust counseling strategies in ways that apply to each student's experiences - it is important for the counselor to consider a student's culture in counseling.

Locus of Control: internal vs external

Internal: that person attributes success to his or her own efforts and abilities External: attributes his or her success to luck or fate (higher anxiety)

What does the ICN stand for?

International Counselor Network; the ICN actively sends out ideas and opinions of counselors throughout the world.

collaboration between School counselors, teachers, admin, social workers, psychologists, nurses and other helping professionals is an example of?

Interpersonal collaboration

What is Test - Retest Reliability?

*An estimation based on the correlation between two or more administrations of the same instrument for different times, locales, or populations.

What is Split-Half Reliability?

*An estimation based on the correlation of two equivalent forms of the scale.

A reliability coefficient is defined as...

*An index of the consistency of measurement often based on the correlation between scores obtained on the initial test and a retest, or between scores on two similar forms of the same test.

Ordinal

*An order or rank of items, interpreted as a difference in rankings -Ex: Height; tall -> short

What are some cultural issues to consider in assessment?

*Representativeness of norm groups in terms of gender and culture. Cultural Biases English as a second language Gender Biases

How would a school counselor best respond to a teacher concerned about bullying by some students in the classroom?

*Ask for Specifics -Names -Who, what, when, where, who Call in the Victim Go from there and assess

Define "Questioning"

*Asking open and close ended questions. Asking for clarification of meaning. Asking for info known only to the client. Asking to understand the client's experiences better.

What are the principles in the Ethical Standards for School Counselors?

*Responsibility to: -students -parents -colleagues/associates -community -school -profession Maintenance of Standards Resources

Implications of language development between 2 and 6 years

*Read age appropriate story books. Give tactful/corrective feedback. Work on simple listening. Ask follow up questions to check for understanding. Ask children to construct narratives about recent events.

Describe Erikson's 5th Period of Life (Adolescence)

*Identity vs Role Confusion 12-18 years Important Events: Social Relationships Outcomes: -Teens need to develop sense of self and personal identity. -Success = able to stay true to self, Failure = role confusion/weak sense of self.

What are self care activities and resources?

*Help to maintain balance, personal centeredness, well being, and avoid burnout. Balance work activities. Balance work and personal life. Maintaining cognitive, emotional, and physical health. Developing and maintaining healthy professional relationships, networking and mentoring.

What is an example of an immediacy response?

"What are you feeling right now as you're telling me what happened last night?" Immediacy responses are focused on the present, right at the time, place, and context of the counseling session. They may be statements or questions, but must be about the present moment, not the past or the future.

Which of the following is a responsibility of a Guidance Advisory Committee (GAC)? (A) Reviewing counseling program data and activity outcomes to make recommendations to the counselor (B) Evaluating the counselor's professional activities (C) Reporting recommendations about the counseling plans to administrators (D) Sharing the work of the committee with the local media

(A). A Guidance Advisory Committee (GAC) is commonly charged with the task of reviewing program data and making recommendations based on their evaluations. GACs typically do not evaluate counselors' professional activities, report directly to administrators, or disclose counseling work to the media.

School systems often utilize standardized tests that assess what students have learned in many different areas. These tests assess which of the following? (A) Achievement (B) Aptitude (C) Behavior (D) Intelligence

(A). Achievement tests measure what an individual has learned, not his or her ability to learn.

According to Carl Rogers, a congruent counselor is one (A) whose actions are not at odds with his or her feelings and thoughts (B) whose expression reflects what the client is presenting (C) whose choice of techniques and interventions remains constant throughout the counseling relationship (D) who gets a client to do what the counselor thinks he or she should be doing

(A). Carl Rogers defined "congruence" as an ability to be authentic and genuine when working with a client. When a counselor's thoughts, feelings, and behaviors are not at odds with each other, the counselor is better able to be with the client more fully, so (A) is the correct answe

When selecting an appropriate assessment, a counselor will most need key information about which of the following? (A) The purpose of the assessment (B) The cost of the assessment (C) The length of time needed to administer the assessment (D) The type of scaling used to report the results

(A). The most important criterion for selecting an appropriate assessment tool is the purpose of the assessment. The professional school counselor needs to determine what information is needed from the assessment and how that information will be used. The cost of the administration, the length of time required, and the type of scaling used in reporting are issues that may be considered, but the decision should not be based on any of these without full consideration of the purpose.

What factors contribute to counselor burnout?

*High level of stress Role ambiguity Environment Individual factors Over identification with clients Therapeutic locus of control.

A local school district decides to implement an anti-bullying program. At the beginning of the school year, the district holds an assembly to talk about the new policies in place to prevent bullying from occurring on campus. The assembly represents which of the following types of intervention? (A) Primary (B) Secondary (C) Tertiary (D) Behavioral

(A). This is a primary intervention because it occurs at the beginning of the school year and is designed to stop a problem before it even begins. Secondary and tertiary prevention occur after a problem has already developed.

Which of the following is a disadvantage of asking clients open-ended questions during counseling? (A) Clients will not be able to express their true feelings (B) The therapeutic process may get off track if a client is verbose (C) Clients will not feel comfortable during therapy (D) Counselors will not be able to gather information on several issues

(B). An open-ended question allows the client to choose how to respond. This freedom may be positive if it allows the client to move on to topics that are a concern, but it may also allow the client to move away from the present focus of therapy, so (B) is the correct response

Which of the following is NOT a distinguishing feature of a collaborative style of interaction? (A) It is voluntary (B) It involves one party as an expert and the other party as a learner (C) It includes sharing resources (D) It is based on mutual goals

(B). Collaborative models are mutual, shared, and reciprocal. (A), (C), and (D) all reflect those elements, while (B) does not, so (B) is the correct answer.

Ms. Reger has 10 students from the same class referred to her anger-management group. She decides to use the opportunity to evaluate the effects of her favorite small- group anger-management program. She plans to use all 10 students in her experiment. Given the information provided, her sampling method is best described as (A) cluster (B) convenience (C) quota (D) simple random

(B). Convenience sampling is when a sample is chosen based on availability or convenience. The students in this study were already referred, so they were an easy group to use

Which of the following is NOT considered an example of a responsive service? (A) Student referrals (B) Classroom guidance (C) Remediation planning (D) Crisis counseling

(B). Of the options presented, only (B) represents an activity that is proactive and focused on prevention. The other activities are in response to an event and are thus reactive and focused on intervention or remediation.

When working with students who have disabilities, a professional school counselor should communicate with outside doctors and therapists to enhance school-based services. By doing so, the counselor is taking on the role of (A) investigator (B) collaborator (C) enabler (D) coordinator

(B). Other professionals, school-based or not, can provide valuable tools and even key phrases that can help to develop treatment goals and plans to help students with disabilities succeed socially and emotionally.

The best way for a counselor to address the problem of a student who is monopolizing group counseling time is to (A) tell the student that group participation will be terminated if the behavior does not change (B) have the group examine the student's behavior in a productive and nonthreatening manner (C) ask the student politely to allow the other group members to speak more frequently (D) discuss the student's monopolizing behavior with the student outside of group time

(B). The best way for the student and his or her peers to learn from the experience of the inappropriate interaction is for them to examine the behavior and its implications in a nonthreatening manner. The other choices either do not take advantage of the group setting to help all students involved or are offensive and nonproductive to the treatment of the student who is monopolizing group time.

Ashley, a high school sophomore, tells the school counselor that she is depressed about her recent breakup with her boyfriend. During the conversation, Ashley says she wishes she "could go to sleep and never wake up." In this situation, the counselor should (A) immediately call the local mental health facility for a psychiatric evaluation (B) assess whether Ashley is suicidal and intervene if she is (C) recognize that Ashley's statement is only a cry for help and should not be taken seriously (D) see if there is any chance of reconciliation for Ashley and her boyfriend

(B). The student's comment clearly indicates some suicidal ideation, but she needs to be assessed further to determine the level of suicide risk and the appropriate intervention. (A) is premature without a suicide risk assessment and (C) and (D) inappropriately ignore the risk of suicide entirely.

Which of the following examples best demonstrates the professional school counselor acting as an advocate for a student? (A) The school counselor advises a teacher to create a behavior plan for a disruptive student. (B) The school counselor conducts a classroom guidance lesson on bullying. (C) The school counselor addresses a concern with a teacher about a student feeling degraded. (D) The school counselor conducts a meeting with the principal and parent to address absences.

(C). In this situation, the professional school counselor must work to advocate for the student. (A) is incorrect because in this situation the school counselor is taking on the role of consultant. (B) is incorrect because conducting a classroom guidance lesson addresses a broad range of student needs and does not involve advocating for a specific student. (D) is incorrect because the school counselor is acting as a collaborator.

Which of the following is the best example of a mentoring program? (A) A professional school counselor and a special education teacher cooperate in developing a plan to integrate students with disabilities into extracurricular activities. (B) A professional school counselor arranges a large-group assembly in which community members share their work experiences with students. (C) Students receive tutoring from and participate in recreational activities with adults from the community during free periods or after school. (D) Students spend part of the day in a resource room getting individualized attention from a special education teacher.

(C). Mentor programs commonly include tutoring and recreational activities provided by adults in the community. The other choices are examples of consultation, collaboration, and teaching.

All of the following are acceptable uses of the Wechsler Intelligence Scale for Children®— Fourth Edition (WISC®-IV) EXCEPT (A) seeing how students' abilities compare with those of the general population (B) identifying learning disabilities (C) identifying gifted children (D) determining whether students need to be retained at grade level

(D). Acceptable uses of the Wechsler Intelligence Scale for Children®-Fourth Edition (WISC®-IV) include comparing students' scores with those of the general population and identifying learning disabilities and giftedness. Grade retention decisions should be based on academic achievement, not intelligence test scores.

According to the American School Counselor Association (ASCA) National Model®, how often must a professional school counselor program be reviewed and evaluated to appraise the progress of the program development and implementation? (A) Twice during the school year (B) Three times during the school year (C) Every other school year (D) Annually

(D). According to the American School Counselor Association (ASCA) standards, the school counselor program should be reviewed once a year.

Which of the following is a management activity? (A) Delivering guidance programs (B) Individual planning (C) Offering responsive services (D) Providing system support

(D). According to the American School Counselor Association's ASCA National Model®, providing system support is an administration and management activity, whereas delivering guidance programs, individual planning, and offering responsive services are not.

A counselor encounters inappropriate behavior by a student group member. The counselor should do which of the following to discourage the behavior? (A)Nod or smile at the student (B)Address the student warmly (C)Shift posture to lean toward the student (D) Avoid responding or close the eyes

(D). Avoiding a response or closing the eyes is a good way to extinguish or discourage inappropriate behavior in a group. The other options all positively reinforce the inappropriate behavior, making it more likely to occur.

A student who cares more about how his peers view him than about how his family views him is said to be functioning at which one of Erik Erikson's stages of psychosocial development? (A) Basic trust versus mistrust (B) Industry versus inferiority (C) Intimacy versus isolation (D) Ego identity versus role confusion

(D). Ego identity versus role confusion occurs from the time a child is 12-19 years old. During this stage, a child learns how to develop a personal identity.

All of the following are important purposes of evaluation in a school counseling program EXCEPT (A) generating valid measures of how well the school counselor meets program goals and objectives (B) helping other stakeholders clearly understand the unique role of the school counselor in the school (C) gathering data for research that will advance the school counseling profession (D) planning and developing accountability procedures even though the program cannot be changed at this time

(D). Evaluation in a school counseling program serves the purposes of generating valid measures of how a program meets goals and objectives, helping others understand the role of the counselor, and gathering data for research purposes. It is not used to plan and develop procedures when the program is unable to change.

In consulting with a teacher about disciplinary problems in the teacher's classroom, a high school counselor demonstrates keen interest in the teacher's various concerns by listening and empathizing with her. By doing this the counselor is assuming the role of (A) a supervisor (B) an evaluator (C) a collaborator (D) a helper

(D). In the school, counselors are those specialists who are expected to assist in the management of social-emotional dimensions of problem situations. They help clients manage problems. Good helpers listen intelligently to clients and respond within the client's frame of reference.

Several tenth-grade students have requested information on possible career paths from the professional school counselor. Which of the following is the first step the school counselor must take to address the needs of the students? (A) Creating a presentation that outlines a variety of career resources (B) Advising the teachers that it is their responsibility to address these concerns during class (C) Setting up several business and college trips that present a variety of career options (D) Surveying students interests and aptitudes to assist them in choosing potential career paths

(D). It is important that the professional school counselor first gain knowledge about the interests and aptitudes of the students to best assist them in the process of exploring career paths.

An elementary teacher shares a concern about one of the students in her class. She states that the student, normally a bright and active participant in class, has become more and more withdrawn over the past few weeks. After meeting with the student, the counselor suspects the student is being sexually abused by her mother's new boyfriend. In this situation, a school counselor's first responsibility is to contact a) the student's mother b) the school principal c) a local counseling center to make a referral d) the state child protective services

(D). Sexual abuse of a minor is by law a reportable event, so the first response is to notify the state child protective services, which will take further appropriate action.

A student and her family lost all their possessions in a recent tornado. The student reports difficulty sleeping, recurrent nightmares, and loss of appetite. The student is most likely suffering from (A) schizophrenia (B) obsessive-compulsive disorder (C) oppositional defiant disorder (D) posttraumatic stress disorder

(D). The student was exposed to a traumatic event and exhibits sleep disturbance, distressing dreams, and diminished interest in a significant activity (eating). These are all symptoms of posttraumatic stress disorder but not of the other disorders listed.

All of the following are fundamental guidelines included in the Individuals with Disabilities Education Improvement Act (IDEA 2004) EXCEPT (A) children with disabilities must be given the opportunity to receive a free, appropriate education (B) assessments must include the use of multiple evaluative procedures (C) parental consent is required before any assessment activities can take place (D) institutions are financially responsible for all testing that occurs

(D). Under IDEA, academic institutions are financially responsible only for testing that they request. Parent requests are evaluated on a case-by-case basis and, typically, the financing is the parents' responsibility.

How would the counselor's behavior be different in facilitating an experiential group as compared to an educational group?

*An educational group has a prescribed course lesson and process as a goal. An experiential group does not have an objective or lesson, but is solely process oriented.

A high school counselor meets with a student who is having difficulty deciding what college to attend in the fall. The counselor discloses his own personal reflections on decisions he made relating to college choice and describes regrets. The counselor's comments are (A) appropriate because they help the student understand that he is not alone in his dilemma about choosing a college (B) appropriate because they allow the student to feel close to his high school counselor (C) inappropriate because the counselor does not understand contemporary issues related to choosing a college (D) inappropriate because they focus more on the counselor's experience than on the student's current dilemma

(D). Unless the counselor can identify a direct therapeutic connection between his own experiences and the client's current situation, self-disclosure focuses the session on the counselor and is, therefore, not appropriate, so (D) is the correct answer.

Language Development Infancy Birth - 2 years

*-Interest in listening to human voice. -Exchange vocalizations with adults. -Repetition of vowel sounds (cooing) at 1-2 months. -Repetition of consonant-vowel (babbling) at 6 months. -Understand common words at 8 months. -Use single words at 12 months. -Use two word combos at 18 months. Rapid increase in vocab in 2nd year.

Language Development Early Childhood 2-6 years

*-Rapid advance in vocab/syntax. -Incomplete understanding of simple words (underextension, overextension, confusion). -Overregulation (ex: foots, gooder) -Overdependence on word order and context. -Superficial understanding of "good listening". -Difficulty pronouncing phonemes and blends (r, th, spl) -Increasing ability to construct narratives.

What areas of development are appropriate for classroom guidance from the counselor? (5)

*-Skill Acquisition (listening, self assessment, student, interpersonal) -Prevention Programming (enhancing academic performance, counter cultural oppression, conflict resolution, coping skills) -Information Dissemination (drugs/alcohol, sexuality, careers, personal safety, relationships) -Affective Development (self esteem, learning about feelings, expressing feelings). -Group Dynamics (value of team work).

What are Yalom's curative factors of effective groups?

*-Universality (we are not alone in our problems) -Installation of hope that treatment will work. -Imparting of Information (teaching) -Altruism (helping others) -Corrective recapitulation of primary family group. -Development of socializing techniques. -Imitative behavior of effective members. -Catharsis (expression of strong effects). -Existential factors. -Direct advice -Interpersonal learning.

What are the 5 types of standard scores?

*-Z scores: number of standard deviations above or below the mean. -T scores: equivalent standard score in a normal distribution (mean of 50, SD of 10). -Stanines: used to understand student's relative range of performance. -Percentile Ranks: percentage of people who had scores less than or equal to the student's score. -Centiles

What are the 5 stages of Super's Life Span Theory?

*1. Growth (birth - 15) 2. Exploratory (15-24) 3. Establishment (25-44) 4. Maintenance (45-64) 5. Decline (65+)

What are the 7 steps in the ethical decision making process?

*1. Identify the problem. 2. Apply ASCA code of ethics. 3. Determine nature and dimensions of the dilemma. 4. Generate potential courses of action. 5. Consider potential consequences of all actions. Choose course of action. 6. Evaluate selected course of action. 7. Implement course of action.

How long should the counselor keep records of counseling with students?

*7 years

Explain Correlation Coefficients

*A numerical value that identifies the strength of the relationship between variables. -1 to 1 linear relationship -1 = negative, no meaning +1 = positive, things go in the same directions. +1 is the strongest relationship

Explain Holland's Theory of Vocational Choice/Typology Model

*A person can be typed into 1 of 6 categories based on interests. -Realistic -Social -Investigative -Enterprising -Artistic -Conventional Assessment type = 3 letter code

Explain Racial Identity Theory

*A person's racial self concept as well as his or her belief's, attitudes, and values. Composed of 3 components: 1. Personal ("Who am I?") 2. Affiliative (What happens to other members of racial group happens to them). 3. Reference Group (level of conformance to the norms of the racial group. Identify to a group by age 3).

What is a Likert Response Scale?

*A psychometric response scale often used in questionnaires. Most widely used in survey research. 5 pt item.

What is personal professional development?

*Activities and resources designed to enhance the professional school counselor role. -attending professional workshops/seminars. -completing advanced grad studies. -keeping current in professional literature. -joining/participating in state, regional, and national associations.

What is a raw score?

*Actual score before manipulation.

What is internal consistency?

*An estimation based on the correlation among variables comprising the set.

What is the role of the school counselor in assessment?

*Assessing students for a variety of purposes -aptitudes and abilities -inter/intrapersonal characteristics and behaviors. Provides understanding by appropriately selecting, administering, and interpreting assessment devices and techniques. -Interviews -Ratings -Observations -Inventories

Describe Erikson's 2nd Period of Life (Early Childhood)

*Autonomy vs Shame and Doubt 2-3 years Important Events: toilet training Outcome: -Children need to develop a sense of personal control over physical skills and sense of independence. -Success = autonomy, Failure = Shame and Doubt.

Name the multicultural counseling competencies (3)

*Awareness: understanding of personal biases. Knowledge: reinforced importance of understanding the world views of culturally different clients Skills: deals with the process of actively developing/practicing appropriate intervention strategies needed to work with culturally different clients.

Describe the Cognitive Behavioral Theory

*Based on the idea that our thoughts determine our feelings and behaviors. Feelings and behaviors are not based on external things like people, situations, and events.

Describe Piaget's Theory of Cognitive Development

*Based on the idea that the developing child builds cognitive structures... -Mental Maps -Schemes -Networked Concepts Used to understand and respond to physical experienced within his or her environment.

Implications of language development between 10 and 14 years.

*Begin to use terminology used by experts. Use classroom debates to explore controversial issues. Present proverbs and have children consider underlying meanings. Explore the nature of words and language as entities in and of themselves.

What are some effective transitional activities? (5)

*Buddy/Mentoring system. New student orientation. Articulation agreements with other districts. Placement activities for incoming students. Teaching job seeking skills.

What are the implications of Kohlbergs stages of moral development for school counselors?

*Can use understanding to help children learn self control. Help parents with discipline issues. Understand boys and girls make moral judgements in different ways.

Explain Super's Life Span/Life Space Theory

*Career development as a life long process. Self concept is the driving force that establishes a career pattern.

What are some of the materials a counselor is responsible for getting out to students and families?

*Career resource materials. Self help information. Community resource information. Computer based interactive guidance systems. World wide web resources.

What are group characteristics that promote success? (5)

*Caring Cohesive Commitment to change Level of trust among members Freedom to experiment.

Theories of Intelligence Explain Cattell-Horn-Carroll's Theory of Cognitive Abilities

*Children differ in fluid intelligence; execution of tasks requiring rapid decision making. Children also differ in crystalized intelligence; knowledge/skills acquired from experience, school and culture. Intelligence has three layers or strata: Stratum III: General Intelligence Stratum II: 10 broad abilities Stratum I: 70-100 very specific abilities are differentiated.

Define "Reflecting"

*Communicating empathy. Neither adding to nor subtracting from clients messages. Communicating back to the clients the main meaning of their message. Reflecting client's feelings (verbally and nonverbally). Checking with the client to see if the counselor's reflection is accurate.

What are some effective strategies for school to work transitions?

*Community building activities Training opportunities Vocational Trainer Voc-Ed at high school level Internships Career guidance for all high school students Working with businesses in the community to develop partnerships. Self awareness Occupational awareness Decision making skills

Name and describe Piaget's third level of development

*Concrete (7 - 11 years)* Accommodation increases. Develops ability to think abstractly and to make rational judgements about concrete or observable phenomena.

Describe the Planning Stage

*Conducting Needs Assessments -surveys -interviews -observations -existing data Inventorying current resources already available for use.

Implications of language development between 10 and 14 years.

*Consistently use academic terminology. Distinguish between similar abstract words (ex: weather and climate). Explore complex syntactic structures. Consider underlying meanings in poetry and fiction. Encourage English language learners to use English in informal conversations and creative writing.

What are the 5 types of validity?

*Content Criterion Concurrent Predictive Discriminant

Describe the Implementing Stage

*Coordinating counseling services and activities with academic and other school programs. Advertising the ability and scope of services. Scheduling counselor time and tasks. Establishing record keeping and information systems.

System support, as part of a comprehensive guidance program, includes:

*Coordinating data collection and sharing. Community and advisory boards. Researching and evaluating services. Program management and administration.

Describe the Evaluating Stage

*Coordinating, designing, and interpreting program development plans. Conducting program evaluation on an ongoing and regular basis. -student outcome studies -satisfaction surveys -counselor self assessment -performance evaluation of counselor -repeated needs assessment

What are the implications of Piaget's levels of development for school counselors?

*Counselors can adjust their approach and select interventions to match the child's level of cognitive functioning.

What are the six types of student transitions?

*Home - School School - School School - Work Referral from one specialist/agency to another. Seeking placement in a different program Return to school from residential treatment or special program.

What is the purpose of counseling classroom guidance?

*Design and implement comprehensive services. Set goals that complement the broader mission of the school. Opportunity to engage in preventative mental health. Organized curriculum matches instruction to student's developmental status, uses varied learning strategies. Promotes interactive learning process.

Discriminant Validity is...

*Determined by hypothesizing and examining differential relations between a test and measures of similar or different constructs. Evidence that a measure of a construct is indeed measuring that construct.

Describe the Organizing Stage

*Developing a comprehensive guidance curriculum. Explicit student competencies and outcomes. Developing preventative and responsive services.

What are the goals of a counseling program in an elementary school?

*Developing knowledge, attitudes, and skills necessary for children to be confident leaders. Work as a team with school staff, parents, and community to create a caring school climate. Provide education, prevention, and early identification/intervention to reach academic success.

What does "least restrictive environment mean?

*Education with students who do not have disabilities to the furthest extent possible. Inclusion principle

Name the types of small counseling groups. (7)

*Educational Task Discussion Experiential Self help Support Counseling

Theories of Intelligence Explain Gardner's Multiple Intelligences

*Eight Intelligences: -Logical/Mathematical -Musical -Kinesthetic -Spatial -Interpersonal -Intrapersonal -Naturalist -Existential Gardner's perspective offers the possibility that the majority of children are intelligent in one way or another.

How do the roles of elementary and secondary school counselors differ?

*Elementary: more opportunity to focus on the self. Function in transition to community. More independent and group counseling. More psycho education. High School: Scheduling, more focused on career and transition to work.

What are some consulting activities with agencies and organizations?

*Empathy Establish collaborative relationships. Establish community advisory committees. Help agencies understand the school system. Develop services to benefit students. Collaborate with agencies to share info in ways that will benefit students.

What are some consulting activities with parents?

*Empathy Helping parents understand child's behavior Helping parents concerned about children and their experience in school Using family counseling skills with students in their family contexts. Teaching parenting skills Managing boundaries and confidentiality. Helping parents learn new responses to behaviors. Helping parents make decisions about options for their children.

What are considered to be the core conditions of effective counseling?

*Empathy Unconditional positive regard Congruence

What are some consulting activities/skills with staff?

*Empathy Educating the staff on various resources available for helping students achieve. Helping staff understand student behavior. Generating alternative solutions. Supporting staff efforts. Making recommendations for change.

Implications of language development between birth and two years.

*Engage infants in simplified/animated conversations. Label/describe objects. Teach hand signs. Ask simple questions. Repeat and expand on children's early sentences.

What are the goals of a counseling program in a secondary school?

*Enhance learning process/promote academic achievement. Acquire positive social skills. Set appropriate career goals. Reach full potential.

Interval

*Equal intervals, show rank order and tell us how far apart intervals are. -Ex: difference between 5 and 6 is the same as 34 and 35.

Define "Ethnocentrism"

*Focus on one's culture as superior.

Group Leadership Style Interpersonally Oriented

*Focus on the here and now experiences of all group members. Ongoing process among them in the development of mutual relationships.

Group Leadership Style Intrapersonally Oriented.

*Focus on the needs and concerns of individual members. Engages the group in interactions that focus on those needs. An entire session could be devoted to one member's needs.

Define "Attending"

*Focused listening. Perceiving verbal and nonverbal messages. Cognitive and affective parts of client's experience.

Name and describe Piaget's fourth level of development

*Formal Operations (11+, Adolescents)* Final Stage No longer requires concrete objects to make rational judgements. Capable of hypothetical and deductive reasoning.

What are the stages of group development? (5)

*Forming: group comes together Storming: group process, storming ideas. Norming: agreement on how group operates. Performing: group practices craft and meets objectives. Adjourning: letting go of the group structure and moving on.

What are the primary components of the National Standards for School Counseling Program?

*Foundation (the "what" of the program). Delivery System (how) Management (when, why, by whom, and the authority of the system) Accountability (evaluation of the program)

What are the four elements of comprehensive guidance programs?

*Guidance curriculum Individual planning Responsive services System support

What is the counselor's role serving students with exceptionalities under IDEA?

*Help develop and implement IEP *Be part of collaborative team *Provide transition services. Help special education teachers assess student needs and develop effective strategies. Help parents adjust to the challenges of nurturing and caring for children. Counsel students with exceptionalities to help them develop to their full potential. Know/understand state and federal laws

Define "Empathy"

*Identify with another's feelings. Emotionally put yourself in the place of another. Ability to empathize is directly dependent on your ability to feel your own feelings and identify them.

Erikson's 8 Personality Stages of Man

*Individual faces different "crisis" or development task 1. Trust v. Mistrust 2. Autonomy v. Shame/Doub 3. Initiative v. Guilt 4. Industry v. Inferiority 5. Identity v. Role Confusion 6. Intimacy v. Isolation 7. Generativity v. Stagnation 8. Ego Integrity v. Despair

What does IDEA stand for?

*Individuals with Disabilities Education Act

Describe Erikson's 4th Period of Life (School Age)

*Industry vs Inferiority 6-11 years Important Events: School Outcomes: -Children must cope with new social and academic demands. -Success = sense of competence, Failure = feelings of inferiority.

Describe Erikson's 3rd Period of Life (Preschool)

*Initiative vs Guilt 3-5 years Important Events: Exploration Outcomes: -Children must assert control/power over their environment. -Success = sense of purpose, Failure = children try to exert too much power and experience disapproval, resulting in a sense of guilt.

Theories of Intelligence Explain Spearman's G

*Intelligence = General Factor: single pervasive reasoning ability used on a wide variety of tasks. Specific Factors: number of narrow abilities involved in executing particular tasks. Children's performance on any given task depends on the general factor and specific factors the task involves.

Theories of Intelligence Explain Sternberg's Triarchic Theory

*Intelligence in 3 Domains -Analytic: making sense of and evaluating info and problems often seen in academic settings. -Creative: imagination, invention, and synthesis of ideas within the context of new situations. -Practical: applying knowledge/skills effectively to manage and respond to everyday problems and situations. Intelligent behavior involves the interplay of the 3 factors which vary from one occasion to the next.

Theories of Intelligence Explain Distributed Intelligence

*Intelligent behaviors depend on the following support mechanisms: -physical -social -cultural Children distribute thinking in 3 ways. -Use physical objects to manipulate large amounts of information (physical). -Work with others to explore ideas and solve problems (social). -Represent and think about situations they encounter using various symbolic tolls (cultural).

Name 3 types of reliability coefficients.

*Internal Consistency Split-Half Reliability Test-Retest Reliability

How does empathy differ from interpretation?

*Interpretations are generated within the counselor, so they are new to the client. Empathy responses reflect something the client is experiencing and would recognize.

Name the 6 methods of appraisal.

*Interviews Rating Scales Behavioral Observations Formal Published Inventories Sociograms Test Batteries.

Describe Erikson's 6th Period of Life (Young Adulthood)

*Intimacy vs Isolation 19 - 40 years Important Events: Relationships Outcomes: -Young adults need to form intimate, loving relationships with others. -Success = strong relationships, Failure = loneliness and isolation.

Name some community organizations and agencies that school counselors consult with.

*Legal Businesses Mental Health Clinics Medical Industrial Cultural Recreational Religious

What are the rights of parents to info about their children's counseling?

*Legal right to access information under FERPA. Counselor should review confidentiality with the parents and stress the importance of the counseling relationship and trust.

Name the levels and stages of Kohlbergs Moral Development

*Level 1: Preconventional Morality -Stage 1: Obedience and Punishment -Stage 2: Individualism and Exchange Level 2: Conventional Morality -Stage 3: Interpersonal Relationships -Stage 4: Maintaining Social Order Level 3 Postconventional Morality -Stage 5: Social Contract and Individual Rights -Stage 6: Universal Principles

What is the role of the school counselor when a teacher discloses his or her own personal struggles and asks for help?

*Listen Be a helper Refer Out

Define "Prejudice"

*Maintaining incorrect conscious or unconscious attitudes, feelings, and beliefs about a cultural group as inferior or unacceptable.

What are the 4 scales of measurement in appraisal?

*Nominal Ordinal Interval Ratio

Define "Interpreting"

*Offering clients new and facilitative ways to understand their experience. Basing interpretation on counselor's perceptions of the client's experience.

What are the mandatory elements of an IEP?

*Performance Assessment Goals and Objectives Service and Participation Transition Services Timeline of Services Criteria for Measuring Success

What types of student development are the focus of a comprehensive guidance program?

*Personal/Social Academic Career

Name 4 different types of tests

*Personality -Objective, Projective -Attitudes and Beliefs -Psychological Constructs Ability and Aptitude Career Interest Achievement

What are the major components of a comprehensive program?!

*Planning Organizing Implementing Evaluating

Name and describe Piaget's second level of development

*Preoperational (talking - 7 years) Applies knowledge of language. Use symbols to represent objects/personifies objects. Better able to think about things and events not immediately present. Oriented to present - difficulty conceptualizing time. Thinking influenced by fantasy - assumes others see situations from his viewpoints.

What are the goals of a counseling program in a middle school?

*Programs are essential for students to achieve optimal personal growth, acquire social skills and values, set appropriate career goals, and reach full potential to become contributing members of society. Enhance learning process to promote academic achievement. Work as a team with staff, parents, and community.

Name the 10 diversity dimensions

*Race Ethnicity Culture Language Religion Gender Age Sexual Orientation Giftedness Disabilities

Explain Consulting

*Roles involve helping others to be more effective in their work with students. Counselor + Consultee collaborate to develop preventative and responsive interventions. Can be formal or informal. Can be occasional or extended over time.

What is the effect of the Family Educational Rights and Privacy Act (FERPA) on the work of the school counselor? (aka: Buckley Amendment)

*Schools must annually send a notice to parents regarding rights to review files and the right to file a complaint against anything they disagree with. Law limits who may access records "with a legitimate educational interest".

Explain Information Acquisition and Dissemination

*Seek appropriate and effective info resources. Review information for quality and relevance. Develop and implement plans for acquiring, maintaining, and disseminating info to students. -career resources -community resources -university apps -internet resources -self help info -computer guidance systems Help students process the info they receive

How can a counselor become more aware of his or her own values and biases?

*Self Reflection Personal Growth Work Personal Counseling Sessions Supervision and Consultation

Name and describe Piaget's first level of development.

*Sensorimotor (birth - 2 years) Child learns about himself through motor and reflex actions. Thought derives from sensation and movement. Child learns he is separate from his environment and aspects of the environment continue to exist outside of his senses.

What are the 3 types of internal consistency?

*Split Half: compare two halves of the test for response consistency. Kuder-Richardson: mean of all split-havles, multiple choice. Cronbrach Alpha: multiple response formats.

What is Standard Error of Measurement? (SEM)

*Standard deviation of error scores. Average amount of error in our measurement around our scores. Use SEM to create confidence intervals (range of upper or lower statistical values that are consistent with observed data and are likely to contain the actual population mean).

What are the legal and ethical considerations that govern the counseling, guidance, consulting, and coordination functions of the school counselor roles?

*State regulations School board policy and regulations

Define "Exploitation"

*Steady transfer of the results of the labor of one social group to the benefits of another.

What effects do testing conditions have on students taking assessments?

*Students can be impacted both positively and negatively by the testing environment. -Room temp -Arrangement -Mood of the facilitator -Mode of the test (computer/pen and paper)

How is a support group different from a therapy group?

*Support Group: Self help. Focus on a particular shared symptom or situation. Usually not led by a trained therapist. Therapy Group: helps people cope with problems and provides for change and growth. Underlying roots of the problem and changing them.

Implications of language development between 6 and 10 years.

*Teach irregular word forms. Use group discussion to explore subject matter. Have child develop and present short stories. Encourage jokes and rhymes. Consult speech and language pathologist if articulation problems are evident.

Ratio

*Tells us proportions Has an absolute zero -Ex: Speed

Explain Standard Deviation Scores

*The average amount of variation around the mean in original terms. Score allows us to measure the difference in distribution of numbers. 1 SD = 68% of population 2 SD = 95% of population 3 SD = 99% of population

Concurrent Validity is...

*The comparison of two measures done at the same time. Degree to which two tests are in agreement.

Content Validity is...

*The degree to which an achievement test's content contains a representative and appropriate sample of subject matter.

Define "Confronting"

*Verbally holding apparently discrepant or incongruent aspects of the client's message and behaviors "in front of" the client for them to see. Helping clients clarify, resolve, or accept the discrepancy.

What are some effective strategies for home to school transitions of very young children?

*Visual Aides Social Stories Schedules in the Classroom Buddies

What is required for confidential info about a student's counseling to be released to third parties?

*Written parental consent to release information.

A group process is...

*one that has a trained counselor or facilitator who can lead the group in accomplishing personal growth.

How does No Child Left Behind address the needs of students with disabilities?

*providing modifications (changing curriculum) and accommodations (equal access)

A didactic group consists of ...

*reading assignments lectures group discussions

appropriate ethical response when there is a professional issue of dealing with one's own stresses

1) inform her colleagues and administrators about the situation, 2) seek ways to reduce her level of responsibility and tasks to a level that she can manage affectively, and 3) do all she can to alleviate the personal stresses

Potential risks of making judgments about a client to a third party

1) lead the client to have less trust in the counselor and perhaps be less open and honest with the counselor in the future, 2) misrepresenting himself/herself to the counselor in an attempt to influence the counselor's judgment, 3) limit the counselor's ability to be empathetic

What are the (5) modes of consultation?

1) prescription 2) provision 3) initiation 4) collaboration 5) mediation

What are Piaget's 4 stages of cognitive development ?

1. Sensorimotor (motor & reflex actions) 2. Preoperational (present orientation, egocentric thinking) 3. Concrete Operational (logical thinking, problem solving, less egocentric) 4. Formal Operational (ability to understand abstract concepts, sociocentric thinking)

What are the 3 domains of the ASCA student standards?

1. Academic 2. Career 3. Personal/Social Development

ASCA National Model

1. Accountability - Data Analysis, Program Results, Evaluation & Improvement 2. Management - Assessment & Tools 3. Delivery - Direct Student Services & Indirect Student Services 4. Foundation - Program Focus, Student Competencies, & Professional Competencies

Crisis Intervention Steps (3)

1. Assess students level of risk 2. Stabilize the situation w/ counseling interventions 3. Follow up to assess well-being & determine the need for further intervention

What are the 5 ethical principals?

1. Autonomy 2. NON-Maleficence 3. Beneficence 4. Justice/Equity 5. Fidelity

5 stages of Grief

1. Denial 2. Anger 3. Bargaining 4. Depression 5. Acceptance

3 core conditions of effective counseling?

1. Empathy 2. Unconditional Positive Regard 3. Congruence

Misuses of questioning include:

1. Engaging client in a Q & A session (too many questions) 2. Why questions (make defensive) 3. Close-ended questions (too brief) 4. Strategic questions (telling the client what to do with them)

4 Stages of consultation

1. Entry stage - physically and psychologically entering the system & establishing rapport -explore needs -desired outcome -negotiate a contract 2. Diagnosis Stage- gathering info & data, analyzing and defining the problem, possible inventions, clarifying goals 3. Intervention stage-selecting and implementing intervention. Developing and implementing evlauation plan 4. Disengagement stage- consultation process is evaluated, relationships are terminated, create a plan for post consultation

What are the steps in ethical decision making process?

1.What are the consequences of your decision? 2. How can the option be implemented? 3. Decide which option is the most ethical. 4. Consider your options. 5. Think through the ethical dilemma and identify all components as objectively as possible.

Paiget's Cognitive Stages of Development

1. Sensorimotor - (0-2) absence of representational thought 2. Preoperational thought - (2-6/7) thought tends to be slow, not yet logical 3. Concrete operational thought - (7-10/11) decenter, can think about more than 1 though at a time, logical relationships 4. Formal operational thought - (12-adulthood) logical thinking extends to "formal" or abstract material

4 main domains of appropriate and permissible accommodations for high stakes testing

1. Setting 2. Timing 3. Presentation 4. Response

What are the four main domains of appropriate and permissible accommodations on high-stakes testing?

1. Setting 2. Timing 3. Presentation 4. Response

Name the 5 at risk populations

1. Sexual/Physical Abuse 2. Gay/Lesbian 3. Dysfunctional Family 4. Physically/Emotionally Disabled 5. Youth Violence

When working with grieving students it is important to be aware of:

1. Student's development level 2. Student's level of maturity 3. Family values, perceptions, rituals 4. Factors surrounding the death

What are the 4 standards addressed by the ASCA national model?

1. content standards 2. program standards 3. performance standards 4. ethical standards

Five stages in group counseling

1. dependency/orientation/forming 2. conflict/power struggle/storming 3. cohesion/cooperation/norming 4. Inter-dependence/synergy/performing 5. termination/closure/adjorning

Classroom guidance: information dissemination about..

1. educational planning 2. drugs and alcohol 3. sexuality 4. careers 5. personal safety 6. interpersonal relationships

Classroom guidance: prevention programming...

1. enhancing academic performance 2. countering cultural oppression 3. Teaching problem-solving conflict resolution 4. Coping skills

Average score on the Weschsler Intelligence Scale for Children

100; Scores between 80 and 120 are said to indicate "normal" intelligence

IQ Scores

130> Very superior 120-129 superior 110-119 high average 90-109 average 80-89 low average 70-79 borderline 69 and below extremely low

For males, the physical development stage of adolescence ends at typically what age?

20; the physical development stage of adolescence ends typically at 20 years of age. This is when they stop increasing in height. For females, the physical development stage of adolescence ends at typically 17.

What is ASCA's recommended student to counselor ratio?

250:1 The national average is actually 451:1

According to ASCA data should be gathered into how many broad categories?

2; School counselor performance Program effectiveness

Audience levels in elementary school assemblies

3 levels: Primary audience (pre-k to 2nd of 3rd grade) Intermediate audience (2nd-6th grade) Middle school audience (4th- young adult) Some levels do overlap depending on the audience

Progress monitoring

3 main reasons: 1. Determine if students are benefiting from the instructional program 2. Create better programs for students who are not benefiting from the curriculum 3. Calculate the rate of student improvement

How many ASCA mindset and behavior standards are there?

35; mindset and behavior standards identify and prioritize the specific attitudes, knowledge, and skills students should be able to demonstrate as a result of a school counseling program. 2 Categories: Mindset Standards (6) Behavior Standards (29) School counselors use the standards to assess student growth/development, guide the development of strategies and activities and create a program that helps students achieve their highest potential.

In evaluating levels of counseling programs, how many maturity levels are there?

3: Developmental Implementation Completion

____________ Protects the rights of children w/disabling conditions that limit one or more major life event (vision, hearing, speaking, walking, learning)

504 Plan

Counselors typically do not coordinate or take a leadership role (facilitate) what type of meetings?

504's and IEP's

An IQ score below _____ may require additional testing for mental retardation

69 or below

Collaborative consultation contains how many elements?

6: System Perspective Approach Prototype Mode Model

Erik Erikson's psychosocial developmental theory

8 stages of man, each stage addresses a crisis or turning point for a developmental task.

Average amount of students in a group counseling session (according to exam edge)

8-10 students

Counselor should spend ___________ % of their time doing direct/indirect student services

80%

In regards to school counseling, STEPS outlines how many steps?

9: STEPS stands for solutions to ethical problems in schools. It is a 9 step process to help determine a positive ethical solution to different situations

Privileged communication

A law protecting the relationship between the counselor and client (law often does not apply to school counseling)

Career development

A lifelong process involving psychological, sociological, educational, economic no physical factors and chance factors that interact to influence the career of an individual (plus culture)

Erickson suggests that as children begin to associate with either industry or inferiority in early adolescence. Industry refers to

A person's ability to succeed and feel worthy. Typically, healthy children who receive the appropriate support and encouragement at home and at school will feel a sense of industry. This identification is important, as it can be the basis of future educational, social and coping skills.

What is the "locus of control?"

A person's belief about what controls his or her life; works on an internal vs. external dichotomy

Mr. Herrera has a large glass bowl on a counter in his third-grade classroom. At the end of each lesson, he places a paper fish in the bowl if the students were not disruptive during the lesson. At the end of the week, if there are at least 10 fish in the bowl, he allows the students to participate in a choice of three preferred activities (private reading, drawing, or doing a puzzle) for the final 20 minutes of class. What method is Mr. Herrera using?

A token economy - when a teacher uses a secondary reinforcement that the students can collect to purchase another reinforcement, a token economy is in effect.

Behavioral consultation is an example of which of the following consultation models? (A) Triadic-dependent (B) Triadic-independent (C) Collaborative-dependent (D) Collaborative-interdependent

A). In the behavioral consultation approach, the school counselor actively obtains and evaluates student data, which is used to advise other staff members (such as teachers) how best to help the student. This approach is a triadic consultation (among counselor, teacher, and student), and it is considered dependent because the consultee (teacher) relies on the school counselor to collect and interpret the behavioral data.

Name this therapy: goal driven /goal setting focused on immediate behavior change feelings of inferiority are the source of difficulty Inferiority complex *focused on the individual

Adlerian

small group activities have been shown to have a positive effects on _____________ & ________________

Achievement and behavior

Woodcock Johnson and the ACT are examples of what kind of assessment?

Achievement assessment

Normally recognized as "high-stakes testing"

Achievement assessments/tests -evaluating general knowledge in a variety of subject areas

Most direct measure of program effectiveness

Achievement data

3 types of data used by school counselors to monitor student progress

Achievement data Achievement-related data Competency-related data

What provides the most convincing evidence of program effectiveness?

Achievement-related data - it is the most direct measure of program success

Resource Allocation Data

Academic support Extracurricular activities

What types of student development are the focus of a comprehensive guidance program?

Academic, Career, & Personal/Social development

What are the three domains in a comprehensive counseling program?

Academic, career, personal/social

An IEP must include a statement of the student's present level of performance in what areas?

Academics, life skills, physical functioning, and social skills. Test-taking skills are NOT considered a major developmental issue included in an IEP. Teachers and counselors use this statement to guide instruction and form a baseline for evaluating student progress.

Positive punishment are applied _____________

After the response *B.F. Skinner

Management System

Agreements, Advisory Council, Use of Data, Actions Plans, Use of Time, Calendars

_______________ is known as the originator of social learning theory

Albert Bandura

Which theorist emphasized the concept of unconditional positive regard?

Albert Ellis

Which of the following would be a barrier to a child's mental development?

All - a lack of family support, malnutrition, an abusive home.

Collaborative-Interdependent Consultation

All members consult and share their expertise to solve a problem.

Ratio scale of measurement

All of the same in interval, except there is the addition of a true-zero point. Ex. height, weight, time on task, income, age

The Family Educational Rights and Privacy Act (FERPA) requires schools to...

Allow students aged 18 and older to inspect their school records, obtain permission from parents before releasing student records, and maintain the confidentiality of student records. But schools may destroy student records without notifying parents or students. Schools are allowed to release directory information about a student without parents' consent-unless the parent has specifically requested the school otherwise.

_________________ outlines the organization & focus of the program. This is made between the counselors and admin. in charge of the counseling dept.

Annual Agreement -Should be created and signed within first 2 months of school year -should reflect mission and program goals - list counselors specific duties -identify inappropriate duties -uses data from use-of-time assessment -identify areas of professional development

End-user program

Applications software is commonly referred to as end-user programs. End-user programs can only run with a proper operating system.

What does "twice exceptional" mean?

Applied to gifted students who have a disability; Many gifted children have some form of disability that makes it difficult for them to develop their special talents. These disabilities span a broad range, including, dyslexia, OCD, Asperger syndrome, and sensory processing disorder. The combination of giftedness and disability often leads to problems in schools, and twice exceptional children are often misdiagnosed or misunderstood.

Analyze & evaluate a students abilities, interests, skills, achievement, use of data to develop and make immediate and long-term plans

Appraisal

The following are examples of what kind of assessment? ASVAB Kaufman Battery Assessment PSAT SAT

Aptitude Assessments

What is the most appropriate use for an aptitude test?

Aptitude tests are used to estimate abilities and predict future performance.

6 phases of CBT

Assessment Reconceptualization Skills acquisition Skills consolidation/application training Generalization + maintenance Post treatment follow-up CBT addresses dysfunctional emotions and cognitive processes through the use of systematic procedures.

Used with IQ testing to test for special needs services. The assessment assesses cognitive ability

Assessment of adaptive functioning

Norm-Referenced

Assessments that compare a student's score to that of the norm group

Ordinal measurement

Assigns ranks to different items (e.g. 1 through 5). These numbers are ranks, not quantities. The differences between these numbers does not have meaning or is not known. Ordinal scales are typically measures of non-numeric concepts like satisfaction, happiness, discomfort, etc. ex: Likert Scale

What should a counselor do to facilitate the postsecondary school-to-work transition process for students?

Assist students in evaluating employer recruitment information, teach students how to complete job applications and teach students how to write effective résumés. After employment is obtained, the school counselor generally does not get involved in students' employment activities.

A 7th grade student who is failing three subjects makes an appointment to see the school counselor. After discussing the student's academic history, the best initial action for the school counselor to take would be to

Assist the student in developing a time management and study schedule and set up weekly progress checks.

Vicarious Learning

Associated with Bandura's social cognitive theory; this type of learning does not involve direct instruction. For example, a child may learn a skill just by observing an adult, without being "taught"

The mother tells the counselor that she is very upset about how some of the other students have been treating her daughter. They have called her names, left her out of group activities, and said harsh things about her family's ethnicity. What action should the counselor take?

Assure the mother that the situation will be addressed, observe for herself these students' behavior toward Jenna, and then meet with the students who are mistreating Jenna in a group to discuss their behavior. A teaching and learning approach to intervention has more beneficial effects.

Empathy response

Attempts to reflect what a student is feeling. Different from an interpretation response, which goes beyond what a student is feeling and offers new views.

The overall performance of a counseling program is determined by what?

Attendance rates Post-secondary education Graduation rates

Which activities is most appropriate for a professional school counselor to undertake to remain current in and knowledgeable about school counseling services?

Attending a professional development conference on rising trends in the field of counseling. This activity provides an opportunity for the professional school counselor to gain new knowledge or information that is helpful to the counselor in the school counseling field.

BASIC ID

B-behavior A-affect (emotion) S-sensations I-imagery C-cognition I-interpersonal relationships D- drugs/diet

Theorist of behavior modification and learning theory?

B.F. Skinner

_____________ Believed that our behavior is determined by environmental factors

B.F. Skinner

Who founded Operant conditioning?

BF Skinner - can change behavior by the use of reinforcement which is given after a desired response

Believed that individuals learn through modeling & observational learning

Bandura

What is Bandura's main argument concerning behavior development?

Bandura's proposed a social behavioral theory of development. This theory combines behaviorism and social psychology and suggest that each one affects the other. More specifically, a child's environment may dictate what behaviors he displays but the behaviors displays will also affect his environment. The environment that an individual lives in may present certain opportunities that elicit certain behaviors.

T-Scores

Based on standard deviations; Mean=50 Standard Deviation = 10 Approximately 68% of t-scores are in the range from 40-60 (2 standard deviations)

The Developmental Theory

Based on the assumption that as people mature and age, they change and adapt based on their self concepts. This theory is used to define career and life developmental stages.

Differentiated Instruction

Based on the theory that students learn in different ways. Therefore, instruction should be adapted to meet the needs of individual students

Name this therapy: identifying behavioral influences focused on behavior, not feelings eliminating problematic behavior behavior is learned Behaviors can be unlearned and modified operant conditioning

Behavioral / B.F. Skinner

________________ Measures fields that have a correlation with academic achievement -discipline referrals suspension rates attendance drug use course enrollment patterns parent involvement participation in extra curricular activities hw completion rates

Behavioral Data

Ms. Johnson is a middle school professional counselor who has developed a positive working relationship with other staff members at her school. Mr. Green, a sixth-grade teacher, refers a student to Ms. Johnson for counseling services. The next week, Mr. Green approaches Ms. Johnson in the hallway to ask how the student has been progressing and whether Ms. Johnson has been able to get any information from the student. How can Ms. Johnson best respond to the request in a professional and ethical manner? (A) Tell Mr. Green that the information discussed is confidential, and walk away. (B) Have a discussion in the hallway about the information the student shared during the session, but ask Mr. Green to keep the information confidential. (C) Ask Mr. Green to meet with her in her office, confirm that she has been working with the student, and inform him that it is important to maintain confidentiality so that she is able to build a working relationship with the student. (D) Ask Mr. Green to meet with her in her office, confirm that she has been working with the student, and allow him to read the counselor's notes taken during the session.

C), which is the only option that addresses the ethical concern of maintaining confidentiality while also building relationships with other school staff.

Which is not an acceptable use of Wechslers Intelligence Scale? A) comparing student's scores w/ those of the gen population B) identifying learning disabilities and giftedness C) determining grade retention

C). Grade retention decisions should be based on academic achievement, not intelligence test scores

While walking through the cafeteria during her lunch break, the counselor notices Ms. Allen, the eighth-grade language arts teacher, talking very emphatically to a male student whose clothes appear quite dirty and old. Ms. Allen then grabs the boy by the arm and begins pulling him toward the school building, and the boy starts crying. What should the counselor do?

Call Ms. Allen and ask to meet with her as soon as possible to offer her assistance in helping the student. In this instance, the counselor observes and becomes concerned about how a teacher is treating a student. Intervening publicly or reporting the teacher to the principal would risk both alienating the teacher and having other teachers see the counselor in an authoritarian role, damaging the important mutual and reciprocal relationship that works best. Ignoring what she observed would not be in the best interest of the student, or of the teacher.

Jolene, an eighth grader in Ms. Collins' class, has been late to school a number of times in the past two months. Ms. Collins reports that Jolene will say only that she is late because she has to help with things at home. Ms. Collins does not feel comfortable calling Jolene's parents and asks the counselor, Mr. Hamilton, for help. What is the best first step for Mr. Hamilton in this situation?

Calling Jolene's parents and asking them to share whatever they can about Jolene's tardiness. It is likely that there are home circumstances that warrant the counselor's involvement.

Reward Systems

Can be either positive or negative; Positive ex: giving a student extra play time Negative ex: taking away play time

Closed questions

Can usually be answered with yes or no. They are likely to put an individual on the defensive. Closed ended questions can contain judgmental aspects.

What does CIDS stand for?

Career Information Delivery System; describes educational and vocational information, training options on national and state/local level; federal funding.

John Holland

Career counseling; career choice is closely associated with personality type. RIASEC (6 types) associated with types of careers.

Holland, Super, Gottfredson, savickas, and Krumbotlz are what kind of theorist?

Career development theorists

readiness to handle challenges involved in exploring & identifying a career choice. (Constructs: decision-making ability, career exploration, career planning, & understanding of the world of work and of specific occupations.

Career maturity

What are three domains addressed in the ASCA National Model: Framework for School Counseling Programs?

Career, academic and personal-social development

_______________________ aims to increase the quality of technical education within the United States in order to help the economy.

Carl D. Perkins Act

Three changes that are a result of ? 1. Using the term "career and technical education" instead of "vocational education" 2. Maintaining the Tech Prep program as a separate federal funding stream within the legislation 3. Maintaining state administrative funding at 5 percent of a state's allocation

Carl D. Perkins Act of 2006

Creator of Person-centered therapy

Carl Rogers

Who founded unconditional positive regard?

Carl Rogers (person centered theory) - showing complete support and acceptance of a person no matter what that person says or does

Which "self" theorist is considered the father of "client-centered therapy"?

Carl Rogers - he believes that the role of the counselor is to be non-directive.

Life Span Development Approach

Constant evolution, individual development is an amalgamation of different processes at various levels of functioning.

When a counselor uses cognitive theories to work with students, what is the overall goal for them?

Change thought patterns through restructuring of schemas - cognitive therapies focus on changing maladaptive thought patterns.

____________ involved modeling through educators attitudes, expectations, and behaviors. Promoting core values of caring, citizenship, respect, courage, honesty, fairness, trust, empathy, etc.

Character Education

Most commonly used programs for promoting peaceful schools and professional behavior is ?

Character education

What is the Achenbach Behavior Checklist?

Child Behavior Checklist Preschool checklist: 18months- 5 years old with 100 questions School aged checklist: 6-18 years old with 120 questions

Jean Piaget

Child psychologist, best known for his theories on child development. Piaget believed that children pass through certain stages of development, and they are not ready to learn certain skills until they reach the appropriate stage.

The patterning of environmental events and transitions over the life course, as well as sociohistorical circumstances. For example, divorces are one transition. Researchers have found that the negative effects of divorce on children often peak in the first year after the divorce. By two years after the divorce, family interaction is less chaotic and more stable.

Chronosystem *Ecological theory

________________________ addresses discrepancies that exist between student groups

Closing-the-gap action plans

What developmental theory describes an adolescent's ability to think abstractly?

Cognitive Development

Piaget, Skinner, Vygotsky, Bandura are all theorist of _____________________________________________

Cognitive development / Learning theories

The counselor reviewed school data and identified that students with IEPs had a notably lower college attendance rate than general education students. What is the most effective strategy that she can use to increase the number of students who attend college?

Collaborating with transition services staff at a local community college to provide workshops focused on transitioning to college for students with IEPs and their families. This is an example of collaborating with local stakeholders to promote equity and apply effective collaborative relationships.

When a counselor and teachers, family members, and community members meet to share their expertise in addressing the complex problems of students who are at risk, what type of consultation approach are they engaging in?

Collaborative-interdependent consultation - the counselor facilitates the consultation process and shares her expertise, as do the other consultants, in an effort to solve a problem.

When working with students who have disabilities, a counselor communicates with outside doctors and therapists to enhance school-based services. By doing so, what role is the counselor taking on?

Collaborator - other professionals, school-based or not, can provide valuable tools and even key phrases that can help to develop treatment goals and plans to help students with disabilities succeed socially and emotionally.

Miguel is a new ELL student at the school. He is fluent in Spanish and knows very little English. How can his school counselor best ensure equity and access for Miguel in the educational setting?

Consult with the family, ESOL coordinator, and a teacher who serves as a student liaison to develop a Limited English Proficiency (LEP) plan to help meet Miguel's language acquisition needs.

Who is in involved in "Triadic" *Consultation

Consultant>consultee>client or situation

Mr. Baer, the school counselor, receives a subpoena to disclose information regarding his work with a child whose parents are in the middle of a custody dispute. What should the counselor do first?

Contact the school system's attorney to seek guidance. Given the complicated nature of the law, consulting a legal representative is appropriate.

Objective Personality Test

Contains controlled response questions. Its results can be analyzed impartially with no need for subjective interpretation. The best known objective personality test is the Minnesota Multiphasic Personality Inventory

Types of validity

Content Criterion (Predictive) Concurrent Discrimminant

For the purpose of evaluating the school's curriculum, the counselor should be most concerned with which type of validity?

Content - indicates how well the test measures the content it intends to cover.

What does CIT stand for?

Crisis Intervention Team; Each school should have their own crisis intervention team to take control of any incidents. These teams must follow set guidelines for schools in an emergency.

Kevin told the counselor that he had been trying to help a classmate when the teacher punished him for talking. Kevin told the counselor that he asked the teacher, "Why was I punished when I did not do anything wrong?" What Lawrence Kohlberg's level of moral development does Kevin's question reflect?

Conventional - Kohlberg's conventional stage of moral development focuses on societal definitions of right and wrong.

Privileged Communication

Conversation that takes place within the context of a protected relationship, such as that between an attorney and client, a husband and wife, a priest and penitent, a doctor and patient, and in some states, a school counselor and a student.

In which of Erik Erikson's development stages does an individual attempt to integrate many roles into a self-image? a. Preschooler. b. School-age child. c. Adolescent. d. Young adult. e. Middle-age adult.

Correct answer: Adolescent Erikson defined eight stages of personal development. In the adolescent stage, an individual attempts to integrate many roles (such as child, sibling, athlete, worker) into a self-image. This task is complicated by the effects of peer pressure.

A school counselor discusses study habits with a student's parents. The counselor is assuming what role? a. Counselor. b. Consultant c. Instructor. d. Mentor e. Teacher

Correct answer: Consultant A counselor who discusses school issues with parents is assuming the role of consultant. In this role, the counselor collaborates with parents on ways to help students succeed. The counselor should approach parents as a peer, not an authority.

According to the ASCA Ethical Standards for School Counselors, which of the following is an example of a dual relationship? a. Counseling the children of close friends. b. Counseling children with special needs. c. Counseling children on both academic and career issues. d. Counseling more than one child from the same family. e. Counseling a child at different stages of his or her development.

Correct answer: Counseling the children of close friends A dual relationship is one in which a counselor has more than one type of relationship with a client, for example, a relationship as a counselor and a relationship as a family friend. Dual relationships can be confusing to children, and they can impair the counselor's objectivity.

Which of the following types of assessments would be most appropriate to use in evaluating the effectiveness of an academic program? a. Norm-referenced. b. Criterion-referenced. c. Subjective. d. Periodic. e. Unstructured

Correct answer: Criterion-referenced A criterion-referenced assessment measures student performance in relation to some standard or criterion. For the purpose of evaluating an academic program, a criterion-referenced assessment would be more appropriate than a norm-referenced assessment, which compares a student's performance with that of other students.

Which of the following would be LEAST useful in conducting a program evaluation? a. student outcome studies b. student satisfaction surveys c. counselors self-assessments d. teacher satisfaction surveys e. demographic analysis

Correct answer: Demographic analysis A demographic analysis of the student population might be useful in program planning, but it would not be of much use in program evaluation. The other choices would offer much more information about a program's effectiveness.

According to the American School Counselor Association, all of the following are appropriate responsive services for secondary school counselors EXCEPT a. individual counseling b. small group counseling c. peer facilitation d. referrals e. individual student planning

Correct answer: Individual student planning Planning is not a responsive service. A responsive service is a response to an issue or a problem that affects students. Individual student planning does not fit this definition.

All of the following describe the guidance curriculum EXCEPT a. responsive b. sequential c. developmental d. student-centered e. results-based

Correct answer: Responsive Responsive activities are those that respond to events. Crisis counseling and grief counseling are examples. The guidance curriculum is not responsive. It is intended to prevent problems rather than solve them after the fact.

To assist students in transition, a guidance program might offer all of the following services EXCEPT a. Orientation programs. b. Buddy systems for new students. c. Placement activities for incoming students. d. Transition programs with other schools. e. School-wide assemblies.

Correct answer: School-wide assemblies Students undergo a variety of transitions in the course of their careers: from home to school, from one grade to another, and from one school to another. School-wide assemblies would serve little purpose in helping students in transition.

Which of the following is one of the provisions of the No Child Left Behind Act? a. Schools must provide military recruiters access to the names and addresses of high school juniors and seniors. b. Schools must adopt the national curriculum in reading and math. c. Schools must administer tests to all students based on national standards in reading and math. d. Counselors may not release confidential information about students without the consent of parents. e. Schools must provide monthly assessments of student progress to parents.

Correct answer: Schools must provide military recruiters access to the names and addresses of high school juniors and seniors NCLB does not require schools to implement a national curriculum or set of standards for reading and math. It has no provisions for informing parents about students' progress. Privacy issues are addressed by the Family Educational Rights and Privacy Act (FERPA), not NCLB. NCLB does require schools to provide student information to military recruiters, unless parents request that this information be withheld.

A teacher administers the same test on two different occasions. The reliability coefficient for the test results is 1. Which of the following is true? a. Test scores for the two administrations were identical. b. There is no reliability in test scores for the two administrations. c. The reliability of the test is uncertain. d. The test is reliable but not valid. e. The test is valid but not reliable.

Correct answer: Test scores on the two occasions were identical A reliability coefficient of 1 means that test scores for the two administrations were identical. The test is reliable. Reliability is not related to validity, which is a measurement of how well a test measures what it is supposed to measure.

A career development program includes assessments that help students understand all of the following EXCEPT a. their skills b. their interests c. their abilities d. their needs e. their job prospects

Correct answer: Their job prospects The assessment component of a career development program is meant to help students understand themselves: their interests, abilities, skills and needs. Students must take this important first step before they can start thinking about such things as job prospects.

What does CACREP stand for?

Council for Accreditation of Counseling and Related Educational Programs CACREP is used to provide leadership skills and promote excellence in the counseling field.

The primary accrediting body for counselor education in this country is the

Council for Accreditation of Counseling and Related Educational Programs (CACREP).

What are the 3 main responsibilities of elementary school counselors as outlined by ASCA?

Counseling, coordination, consultation

Management

Counselor Agreements Steering Committee Advisory Committee Use of Data Action Plans Use of Time Calendars Developing and Maintaining CCGP

Triadic-Dependent Consultation

Counselor advises other staff members; dependent consultee relies on the school counselor to collect and interpret data.

Anecdotal notes

Counselors should make sure that their anecdotal notes are confidential, factual, and only include relevant facts. These types of notes can be placed in student files

Who are mandated reporters and what must they report?

Counselors, teachers, administrators, nurses, and social workers. They are required to inform authorities about suspected physical and or sexual abuse.

Process by which a counselor experiences feeling similar to those of a student because of a similar past experience

Counter transference

A high school student tells the counselor about a romantic situation with a neighborhood boy, and the counselor begins to have feelings similar to the student's based on an experience that occurred when the counselor was in high school. What is the situation an example of?

Countertransference - this is the process by which a counselor experiences feelings similar to those of a student because of similar past experiences.

Achievement-Related Data

Course enrollment patterns Discipline referrals Suspension rates Attendance rates (part of 3 data components that counselors use to monitor student progress)

A school counselor in a prekindergarten program will be working with a 4 year old child whose parents recently died. For a child at this developmental level, the counselor's best approach would be to use

Creative arts technique to facilitate emotional expression.

What is CISM?

Critical Incident Stress Management: used with students suffering from a traumatic event and always done in a safe environment. There are 7 steps: Introduction Fact phase Thought phase Reaction phase Symptom phase Teaching phase Closure phase.

Interactions between a child and cultural group develops cognitive constructs such as speech patterns, written language, and symbolic knowledge

Cultural Mediation *Vygotsky

_____________________ is an effective way to increase student achievement and improve student behaviors & attitudes

Curriculum Action Plan used for: classroom activities, group activities

Analysis of _____________________ results report demonstrates the effectiveness of program & classroom activities and informs program involvement

Curriculum Results Report

Baseline data is required for?

Data based decision making

Long Range Data

Data can be collected over 3 different time frames: immediate, intermediate, and long range. Long range data is collected year to year.

A middle school counselor wishes to use technology to assist in placing students in their requested elective classes. Which of the following types of software would be most appropriate?

Database

School counseling core curriculum, individual student planning, and responsive services are all part of which ASCA component?

Delivery Component

_____________ ASCA component focuses on the method of implementing the school counseling program to students

Delivery component

A school counselor is working individually with several students fro single-parent homes. It would be most important to the student for the school counselor to

Demonstrate respect for their family structure

A lawyer who is working with a student's parent requests assessment information that is keep in the student's cumulative folder. The lawyer, who does not have parental permission for the access, comes to a school counselor's office and demand to see the assessment information. The school counselor cannot reach the student's parent. The school counselor's legal responsibility in this situation is to

Deny access until the school receives a signed release form

Occupational Outlook Handbook (OOH)

Department of Labor Bureau of Labor Statistics; cross references to Dictionary of Occupational Titles (DOT-Replaced by O*NET). Describes work, working conditions, educational and training requirements, salaries, and job markets.

Scores are drawn from raw scores by comparing raw scores to the nrom group. Sometimes called "norm referenced"

Derived scores

Nominal measurement

Descriptive, does not imply ranking of items measured. Nominal scales are used for labeling variables, without any quantitative value ex: race, sex

In evaluating counseling programs, how many difference types of evaluations are typically used?

Design evaluation Process evaluation Outcome evaluation Impact evaluation Cost effectiveness evaluation

Results-based comprehensive guidance program

Designed to produce measurable results. It is intended to provide students the tools they need to be successful in school and in the transition to higher education or work

A middle school counselor receives an email from the assistant principal asking for specific information related to a particular event that occurred with a student. The counselor notices that several other faculty members are copied on the email. What is the best way for the counselor to respond?

Determine whether the assistant principal is privileged to the requested information, and if so, provide it directly to the assistant principal with the information sealed or encrypted.

Functional Behavior Analysis

Determines antecedents and consequences of problematic behavior.

Nonuseful counselor responses...

Detract or distract from students' expeirence, remaining superficiaal and limiting or discouraging exploration, understanding and feelings about their experiences i.e. "you don't really mean that" minimizing client's feelings

Explain an Interval scale? Exp?

Distinctiveness, magnitude, and equal intervals (one unit change in the numbers represents the same difference in the attribute at any region of the set. Example: Fahrenheit scale of temperature

Explain a Ratio scale? Exp?

Distinctiveness, magnitude, equal intervals, and absolute zero (zero means none of the attribute) Example: Kelvin scale of temperature

Heterogenous group counseling

Diverse members, or a combo of homogenous and heterogenous

accountability

Documentation of time spent in various activities is for the purpose of accountability. Examples include classroom guidance, individual planning, system support, and responsive services.

Life span, life space

Donald Super

life stages, career maturity, life career rainbow, self-concept, archway of career determinants

Donald Super

Impairment of cognitive function is a sign of what?

Down Syndrome

USA cultural worldview beliefs

Dualistic thinking, rugged individualism and mastery over nature

In the termination stage of group counseling, what are members responsible for?

During the termination stage of group counseling, members begin to integrate their group experiences with real-life experiences.

What theorist created 8 basic needs? Relatedness transcendence rootedness sense of identity Frame of orientation Excitation and Stimulation Unity Effectiveness

Erich Fromm *Personality theorist

Authoritarinism

Erich Fromm - technique that others use to ward off anxiety

Destructiveness

Erich Fromm - technique which refers to an attempt to destry those we percieve as having power

Automaton conformity

Erich Fromm's term for escaping the demands of freedom by accepting the personality type preferred by our culture or social group

When dealing with middle school students, it is important for a counselor to understand the developmental relationship between industry and inferiority. This is an example of a theory of development based on the work of what psychologist?

Erickson

A school counselor write all case notes on a school computer. Ethical and technological standards require the counselor to

Establish a time line for purging student files

What is the first thing you should do when holding a group with adolescents?

Establish trust

What is most important for a counselor who is working with adolescents in a small-group counseling format?

Establishing trust with group members - trust is foremost in a student's relationships and interactions with adults who are not part of their social network. Establishing trust between the facilitator and members is most important for this age group.

Multicultural Definitions

Etic: Characteristics assumed to be shared by all cultural groups. ex: identity development, acculturation, social class differences. Emic: Particular culture in terms of its internal elements and their functioning. Ethnic: Relating to a population subgroup (within a larger or dominant national or cultural group) with a common national or cultural tradition.

What replaced the No Child Left Behind act? What does it basically state?

Every Student Succeeds (2015); allows more freedom in setting educational goals and has fewer restrictions on how students are tested

ASCA National Model THEMES

Evidenced based school counseling model with an emphasis on Career, academic and personal social development

A school counselor who will be working with a culturally diverse group of students would like to ensure an ability to accurately understand and effectively respond to the student's. Which of the following would be the most helpful first step for the counselor to take to achieve this goal?

Examining personal attitudes for possible cultural bias toward the students

A school counselor is reviewing participation rates in AP courses by gender and finds that females are heavily represented in English and social studies courses but poorly represented in mathematics and science courses. The most appropriate action for the counselor to take would be

Examining the AP referral process currently used in the school.

FERPA, also known as the Buckley Amendment, grants what to a student and their parents?

FERPA grants students and their parents rights to access to official student records. After students turn 18, they have sole right to access records.

What is the effect of the Family Educational Rights & Privacy Act (FERPA) on the work of the school counselor.

FERPA protects the privacy of a students records. It means that parents many request to see students educational records. The impact on the school counselor tends to be an ethical one when non-custodial parents express an interest in the records. Also the counselor should be savvy about what is put in the permanent record.

____________________ grants students and parents access to official student records. After 18, the student has sole right to access records.

FERPA, also known as the Buckley Amendment

What are fair share responsibilities?

Fair share responsibilities are tasks that do not fall within any specific job description but are necessary for the operation of a school. These duties are shared among all faculty and staff members.

T or F: Achievement tests measure a student's ability to learn?

False; achievement tests measure what an individual has learned NOT their ability to learn

T or F: Adolescent boys mature more quickly than adolescent girls?

False; adolescent girls mature more quickly (2 years or so). Thus, it would be wise to remember this when having a co-ed group of girls and boys as it may affect interactions in mixed-set groups

T or F: under IDEA, academic institutions are financially responsible for all testing?

False; only for testing that they (the school) requests for the child

A major guide to help schools use students' records appropriately is the

Family Educational Rights and Privacy Act of 1974.

The federal law that guides schools in monitoring their collection and use of students' records is the

Family Educational Rights and Privacy Act of 1974.

Frank Parsons

Father of career development. Emphasis on testing and matching clients. Understanding ones self (self knowledge)

Used for solving Problem behavior

Functional Behavior Analysis *Used during an IEP to address other contributors to problematic behavior

Final stage of group description

Further identify what was learned and decide how this new learning can become part of daily life - summarizing, pulling togheter, integrating, and terminating

Frequently used for Disaggregating Data:

Gender, race, free and reduced lunch, course enrollment, language spoken at home, grade level,special education, teacher assignment

What is the most appropriate placement for a student who is moderately physically disabled but has average cognitive skills?

General education classroom - a student with a moderate physical disability and normal cognitive function could excel in a regular classroom with appropriate support.

Name this therapy: future is impacted by present here & now the whole is more than the sum of parts may be used in crisis client needs to take responsibility for actions empty chair homeostasis

Gestalt

When student's help each other (peer tutoring) under the general guidance of a teacher

Guided Participation *Vygotsky

What is the best way for a counselor to address the problem of one student monopolizing group counseling time?

Have the group examine the student's behavior in a productive and nonthreatening manner. This takes advantage of the group setting to help all students involved in a non-offensive way.

HIPAA

Health Insurance Portability and Accountability Act Has no effect on mandated reporters, who are required by law to report suspected cases of abuse or neglect. HIPAA requires medical personnel to maintain the confidentiality of patients' health records.

In consulting with a teacher about disciplinary problems in the classroom, a counselor demonstrates keen interest in the teacher's various concerns by listening and empathizing with her. By doing this, what role is the counselor assuming?

Helper - counselors are specialists who are expected to assist in the management of social-emotional dimensions of problem situations. They help clients manage problems. Good helpers listen intelligently to clients and respond within the client's frame of reference.

What are the role's of teachers in implementing the guidance program?

Helping counseling identify the needs of students. Teachers are not involved in evaluating the effectiveness of the guidance program or designing the guidance curriculum. Counselors, not teachers, have the primary responsibility for delivering the guidance curriculum. While teachers may identify students who have problems, a counselor has responsibility to all students, not just the troubled ones.

RIASEC stands for?

Holland's 6 occupational environments

Fidelity

Honoring commitments and keeping promises, including fulfilling one's responsibilities of trust in professional relationships.

Socratic Dialogue

How Questioning *CBT

Content validity

How well a test measures the test-taker's understanding of the curriculum.

Who created the theory of multiple intelligence's?

Howard Gardner (8 independent intelligences across a variety of disciplines)

Freud's Personality Theory

I'd - born with, ego - develops as our cognitive and physical skills do, superego is last to develop - must find a balance between id's desires and restrictions of superego. Conflicts are a result of a person's experiences with 5 psychosexual stage: oral, anal, phallic, latency and genital

Convergent Validity

In analyzing convergent validity, a counselor attempts to determine if results that are expected to be similar are in fact similar. If test results have high convergent validity then the programs are all having a similar effect on student performance. If test results have low convergent validity, then the counselor would try to determine why there is variation in student performance.

National Assessment of Education Progress (NAEP)

In general, participation in NAEP is voluntary, but states that receive Title I funds must participate in NAEP tests in math and reading at 4th and 8th grades. NAEP does not offer different tests in different states; the basic rationale behind NAEP is to establish a common metric for schools across the country. However NAEP is not intended to establish a national curriculum to be used a a basis for awarding federal funds for education.

WorkKeys Assessment

Includes 3 assessments and if student passes all 3 they are eligible for the national Career Readiness Certificate

Data entry, coordination achievement testing, signing tardy slips, performing disciplinary actions, sending students home for dress code, teaching a class when the teacher is absent, computing grade point averages, supervising students, maintaing student records, coordinating IEP's, SST's, and SARB meeting These are example of appropriate or inappropriate activities for school counselors?

Inappropriate activities for school counselors *Keywords for appropriate activities: Interpreting, providing counseling, collaborating, analyzing, providing suggestions, advocating

A counselor meets with a student who is having difficulty deciding what college to attend in the fall. The counselor discloses reflections on personal decisions she made relating to college choice and describes regrets. Are the counselor's comments appropriate?

Inappropriate because the counselor focuses more on their own experience than on the student's current decision. Unless the counselor can identify a direct therapeutic connection between personal experiences and the client's current situation, self-disclosure focuses the session on the counselor and is, therefore, not appropriate.

In their role as consultants, what type of service do counselors provide to students?

Indirect; but in their role of consultants, counselors provide direct services to teachers, parents, community agencies, and others. In effect, they are helping others to help students.

_________________ designed to help students establish personal goals and develop future plans -Delivered in individually, in small groups, or in a classroom setting

Individual Student Planning

What best defines the individual planning component of comprehensive guidance programs?

Individual, small-group, and whole-classroom activities that help all students to develop unique educational and career plans for themselves. Individual planning is best described as the broad collection of counseling and guidance activities that are designed to help all learners develop their own educational plans and career objectives.

IEP

Individualized Education Plan, for students with IEPs, documentation is mandated by the IDEA.

Family Systems Theory

Individuals are not understood properly in isolation, but rather as part of a family. Under this theory, families are looked at as a system, not by each individual person

What happens during the norming stage of group counseling?

Individuals assume their roles in the group.

In regard to the official school records of children under 18 years of age, federal law guarantees parents or guardians the legal right to

Inspect everything that has been placed in their children's records.

Stanine scores are reported as?

Integers; 9 groups which are assigned an integer from 1 to 9. The drawback of this is that the 2 scores in a single stanine are sometimes further apart than two scores in adjacent stanines.

Examples of what kind of assessment? WISC (Comparing student score with general population) WAIS Stanford-Binnet

Intelligence Assessments

___________ assessment evaluate general cognitive ability

Intelligence Assessments

A five year old boy continues to attempt to play with children who openly ridicule him and leave him our of games. His persistence with this group of children has recently resulted in a number of fights on the playground. After being referred to your office, you learn that he has an unsupportive home life. His parents were divorced last year. He lives with his mother who works nights, and he does not see his father. He is often left at home alone; however, a neighbor watches him from time to time. According to Cognitive Behavior Theory, what developmental skills is this child lacking?

Interpersonal skills

What is the chief drawback of a projective personality test? (ex: Rorschach inkblot)

Interpreting responses is a subjective matter. Sometimes interpretion might say more about the interpreter than the person taking it.

During counseling sessions, a student and counselor decide together that to achieve therapy goals, it would be beneficial for the student to attend group counseling sessions and to continue individual sessions with the counselor. What phase of counseling is this?

Intervention and problem solving - this phase is when the client and counselor together decide on what steps to take to meet the therapy goals they have set.

What is the prescriptive mode in consultation?

Involves the SC acting as an expert and then helping the teacher to implement the appropriate solution

What is a school counseling program?

It is comprehensvie in scope, preventative in design and developmental in nature

Creator of "Typology" / trait and factor theory

Jonn Holland

Legal Mandates

Judicial command or precept issued by a court or magistrate, directing proper behavior to enforce a judgment, sentence or decree.

Treating others equally, but in relation to their individual differences.

Justice/ equity (1 of 5 ethical principals)

This career assessment suggests occupations and college majors based on students interests patterns

KIOS

During a counselor's consultation with foster parents, the parents mention that their child was physically abused in a previous living situation. The foster parents state that they do not want the child's teacher to be informed of the abuse. Meanwhile, the teacher has asked to meet with the counselor regarding concerns about the child's behavior. In this scenario, what ethical responsibility does the counselor have towards the family?

Keep the information confidential - sharing information without the parents' permission is considered a breach of confidentiality.

What would be most effective in assisting high school students so that they can develop career portfolios, make educational plans, and take career assessments?

Kuder Career Planning System

__________________________ is used for HS students to develop portfolios, make educational plans, and take career assessments

Kuder Career Planning System

Children with Asperger's Syndrome appear to lack what?

Lack empathy; they have trouble interpreting non-verbal communications and social cues. They may seem uninterested in the feelings of others because they have trouble interpreting them.

Which is a predictor of a student dropping out of school?

Lack of participation in school activities - leads to disconnection from peers and supportive adults, creating a risk factor for dropping out.

What is a predictor of a student dropping out of school?

Lack of participation in school activities; disconnection from peers and supportive adults.

Role of a Helper

Listen intelligently and respond within the client's frame of reference. Listening and empathizing.

_______________________ refers to the extent to which individuals believe they can control events affecting them.

Locus of Control

What's a good way to find out if an element of counseling is related to program delivery?

Look at any consultative services. So data such as attendance, percentages of tests, or mean test scores are not related to delivery component

Result Reports

Looks to show the actual change in student behavior and learning. These reports are used to determine if a counseling program is effectively doing what it set out to do.

____________________ component of the ASCA National Model provides organizational assessments and tools designed to manage a school counseling program

Management

Ms. Patel, a school counselor, reviewed her school's counseling data from last year to determine which programs to utilize again this school year. She then created a weekly and annual calendar to increase participation and awareness of events sponsored by the school counseling department. Ms. Patel's actions are examples of which component of ASCA?

Management - program evaluation, planning, and advertising are all part of the management component.

What are examples of Computer Managed Counselor?

Management of student records List of counseling activity logs Management of scheduled meetings

A student in ESE recently behaved in a way that warrants suspension from school. According to federal law regarding students with disabilities, if the suspension would last longer than 10 days, the school must convene what type of meeting?

Manifestation determination

What are the 4 relationship patterns that occur in families?

Marital conflict Dysfunction in one spouse Impairment of one or more children Emotional distance

Narrative approach to career development

Mark Savickas

Aptitude tests

Measures a student's ability to master a certain subject

Summative Assessment

Measures completed learning

Formative Assessments

Measures learning in progress; Can help teachers determine if instruction is effective and provides guidance for any changes that might be needed. They can also help teachers determine which students need additional help and improve the chances that all students will be successful. Formative assessments are a dynamic process that take place throughout a course. They do not provide the kind of data that would be useful in ranking teachers.

What is a criterion-referenced assessment?

Measures student performance in relation to some standard or criterion.

Which part of central tendency is the most resistant to the effect of extreme scores?

Median

What does MLE stand for?

Mediated Learning Experience; MLE is one of the primary foundations for all the different dynamic assessment approaches

Which legislative act increased the accountability requirements for education professionals, including counselors?

No Child Left Behind Act of 2001 - it raised the standards of accoubtability for educators, including counselors.

Inclusive classrooms

No child is meant to feel inferior or alien because of his or her cultural background. Inclusive schools celebrate diversity and at the same time help students see themselves as part of a larger society composed of a variety of groups.

Title IX of the Education Amendments of 1972

No person in the U.S. shall, on the basis of sex, be excluded from participation in, be denied the benefits of or be subjected to discrimination under any education program or activity receiving federal financial assistance.

Open group v. Closed group

Open group allows for new members to enroll throughout the running time of the group, closed does not

5 characteristics Carl Rogers identified with a fully functioning person

Open to experience Existential living Trust feelings Creativity Fulfilled life

A focus group uses what type of interview technique?

Open-ended technique; the people in these groups are normally associated in a similar way

Behavior is learned. Behavior is repeated as a result of desirable consequences, and behaviors are stopped as a result of undesirable consequences

Operant Conditioning *B.F. Skinner

BF Skinner

Operant Conditioning founder; can change behavior by the use of reinforcement which is given after a desired response

components for creating a comprehensive guidance program in the school

Organizing, planning, implementing, and evaluating

Initial stage of group description

Orientation & exploration - group members present socially acceptable selves, characterized with anxiety/insecurity, determine if the group is a safe place

Middle school career exploration discussion should focus on:

Overview and exploration of career clusters. Middle school students are only beginning to determine their interests, career strengths, and overall view of career options.

What defines how the vision and mission statement will be accomplished and also guides the development of curriculum, small group & closing-the-gap action plans

Program Goals

Cognitive Counseling

Patients learn to control where their attention is directed or change the content of their thoughts. ex. they learn to replace maladaptive thoughts w/ constructive thoughts and learn to direct their attention away from

The key advantage of using group counseling over individual counseling is:

Peer exploration of issues. Group counseling often encourages students to feel freer to speak openly and offers the advantage of both counselor and peer feedback.

The percentage of scores that are at or below a named score

Percentile rank

Data collected through pre/post tests, needs assessments, program evaluations, surveys, etc. -attainment of competencies, changes in attitudes/beliefs, and perceived gains of knowledge

Perception Data

What participants think they know, believe, or can do. what type of data?

Perception data

A student who recently emigrated from a country where English is not the primary language performed poorly on an end-of-year exam. What step should the professional school counselor take in interpreting the results of the exam?

Performance in class and English proficiency should be considered in interpreting the score. The language barrier must be considered when interpreting the assessment results of students for whom English is a second language.

What type of therapy has the relationship drive the counseling process?

Person-Centered

Disturbance is due to incongruence between the real self and ideal self

Person-centered *Carl rogers

What type of of therapy emphasizes the relationship between the client and the counselor and focuses on respect, empathy, genuineness, and a nonjudgmental attitude?

Person-centered - in this type of therapy, the relationship drives the counseling process.

Name this therapy: Unconditional positive regard active listening reflecting feelings paraphrasing Counselor should not be at odds with self used to gain a greater understanding of self

Person-centered / Carl Rogers

What does PIM stand for?

Personal Information Manager; PIMs are used to organize random information for the user. Counselors can use this to help put together their notes or data.

What is a PPOS?

Personal Plan of Study; Counselors can help students determine a personal plan of study for their college and high school experiences. The PPOS is used to help determine a plan of coursework for students based on their ideal career path

Inteval scale of measurement

Possess all the characteristics of the ordinal scale, with one additional feature: The distances between the points on the scale are equal Ex. the distance between 80 and 90 is the same as the distance between 90 and 100 on an depression evaluation or aptitude test; temperature; in reality there are not very many interval scales

A family with a student in school lost all of their possessions in a severe storm. The student reports difficulty sleeping, recurrent nightmares, and loss of appetite. What is the student most likely suffering from?

Posttraumatic stress disorder - the student was exposed to a traumatic event and exhibits sleep disturbance, distressing dreams, and diminished interest in a significant activity (eating). These are all symptoms of post-traumatic stress disorder.

What are the 3 aspects of evaluation in regard to the 4 phases of a counseling program (continuous planning, designing, implementing, and evaluation)

Program evaluation Personnel evaluation Results evaluation

What are Kohlberg's levels of moral development?

Pre-conventional: Sense of morality is externally controlled. Accept and believe rules of authority figures; judge an action based on its consequence. Conventional: Sense of morality is tied to personal and societal relationships; children continue to accept rules of authority figures because they believe it is necessary to ensure positive relationships and societal order. Post-conventional: Sense of morality is defined in terms of abstract principles and values. people now believe some laws are unjust and should be eliminated or changed.

A preference in the way something is achieved or done

Preferred modalities

Modes of Consultation

Prescription (counselor acting as expert) Provision Invitation Collaboration Mediation

What are modes of consultation?

Prescription, provision, initiation, collaboration and mediation

Time assessment and School Profile Analysis are most effect for?

Program evaluations and program improvement

Culturally Responsive Course

Presents knowledge in a way that is relevant to the cultural background of students. The purpose of this approach is to make the subject matter meaningful to students so that they will be more engaged in the educational process.

Linda Gottfredson

Prestige and sextype ratings of occupations in the different Holland fields of work. Circumscription and compromise.

____________ prevention focuses on programming for the entire student body using education to ward off any potentially at-risk behaviors or situations

Primary Prevention

What is a guidance group?

Primary group that is mainly preventative ex: study groups

A local school district decides to implement an antibullying program. At the beginning of the school year, the district holds an assembly to talk about the new policies in place to prevent bullying from occurring on campus. What type of intervention is represented through an assembly?

Primary intervention - it occurs at the beginning of the school year and is designed to stop a problem before it begins.

IEP meetings do not require which administrative staff to attend?

Principals and Counselors (although counselors should attend if they have previously worked with the student or can provide any helpful input).

Experiential learning

Process of making meaning from direct experience; ex: going to the zoo, observing animals, and then reflects on those observations

The Thematic Apperception Test

Projective personality test. Typically clients are given a series of ambiguous photos and asked to make up a story about them, including what has led up tot he event shown in each picture, what the people in the pictures are feeling, and how the story will end.

Kevin has been referred to his counselor because of behavior problems. He tells the counselor that he has been diagnosed with bipolar disorder and states that he frequently has thoughts of harming himself. Kevin says he is not currently getting counseling services outside of school. What should be the counselor's first course of action?

Provide Kevin's parents with a list of referrals and have them sign a release-of-information form. A student with a chronic, serious mental illness needs more support than the school counselor can provide, so she should provide information about referrals. It is also important for her to ask Kevin's parents to sign a release-of-information form so that the school counselor can collaborate with the outside counselor.

What is classified as a responsive service?

Providing individual or group counseling - according to ASCA, responsive services consist of activities involving prevention and intervention.

Upon completing the 10th grade, a student expresses a desire to study engineering in college. A review of the student's educational record, including classes in Algebra I, geometry, biology and chemistry, reveals an overall academic GPA of 3.68. Which of the following would be the best approach for a school counselor to use in assisting this student?

Providing the student with information on course requirements for the engineering field

A content group is ...

Psychoeducational driven. A content group is driven by content material - didactic in nature.

Cognitive Therapy is what type of therapy?

Psychotherapy; used to help people overcome problems by changing their dysfunctional thinking, behavior, and emotional responses

According to Kohlberg's theory of moral development, a student will follow school rules in order to avoid receiving detention. This is an example of what stage of moral development?

Punishment-Obedience

What is the most important issue for a school counselor to consider when selecting an appropriate assessment?

Purpose - school counselors must be able to select a valid assessment instrument to obtain desired information.

Arthur is an out-of-state transfer student who is having trouble adjusting to his new school setting. Arthur reveals to the counselor that he does not like his classes and that he is bored. Arthur's permanent record indicates his scores from last school year were as follows: Complete Composite 85th percentile; Reading 95th percentile; Language 89th percentile; Math 70th percentile; Science 65th percentile; Social Studies Total percentile. What is the counselor's best next step?

Recommending Arthur for further testing for the gifted program - a score of 90th percentile or above is often used to qualify a student for further testing to see if the gifted program is appropriate.

What are the mandatory elements of an IEP?**

Referral Assessment Modifications Accommodations

Reflective Responses

Reflects, or mirrors, a client's words and feelings. A reflective response shows the client that the counselor is listening and empathizing.

Forms of logical consequences

Rehearsal of a desired behavior Restitution Temporary loss of privilege.

Eric Fromm's 8 basic needs:

Relatedness Transcendence Rootedness Sense of Identity Frame of Orientation Excitation and Stimulation Unity Effectiveness

An elementary school counselor meets weekly with a small group of students to teach social skills. After several weeks, the counselor notices that one especially quiet student is trying to hide unexplained bruises. The counselor suspects that the child is being physically mistreated. In this situation, the counselor is required to follow which of these procedure?

Reporting the observation to the child abuse hotline immediately

What are logical consequences?

Require adult intervention; When disciplining a student, the incorrect action taken by the student should always have logical consequences. The action should match the punishment. It should be not be more or less severe than necessary,

The No Child Left Behind Act

Requires annual testing in math and reading for all students in grades 3rd through 8th. Students are tested once again in these 2 subjects in high school. NCLB does not require schools to implement a national curriculum or set of standards for reading and math. It has no provisions for informing parents about students' progress. Privacy issues are addressed by FERPA not NCLB. However, NCLB does require schools to provide student informaiton to military recruiters, unless parents request that this information by withheld.

What is an early intervention process used to help students with: behavior academic achievement staying in school

Response-to-Intervention (RTI)

Activities that meet students immediate needs and concerns

Responsive Services

_______________ services consist of activities designed to meet students immediate needs and concerns. Designed to help student(s) overcome/resolve academic, career, and personal social issues

Responsive services

While evaluating school data, an elementary school counselor determines that students who participated in study-skills workshops showed a 40 percent increase in attendance. What type of data is the attendance rate?

Results

Accountability

Results Reports, School counselors performance standards , The Program Audit

_____________ provides a written presentation of the outcomes of counseling program activities; contains process, perception, and outcome data

Results report

When one commits to an identity, "role confusion" end, and _______________________ is reached *Erikson

Role Achievement

When one has neither experienced an identity crisis or commitment *Erik Erikson

Role Diffusion

What does SCALE stand for?

School Counseling Analysis Leadership and Evaluation

What does SART stand for?

School Attendance Review Team. SART includes teachers, counselors, administrators and school psychologists. The team helps students with attendance problems.

_________________________ is planned, written, instructional program that is comprehensive, preventative, and developmental. -Delivered to all students -delivered in class or during group activities

School Counseling Core Curriculum

what is SCOPE?

School Counseling Operational Plan for Effectiveness -needs programming -measurable objectives -research supported curriculum -cross walking (standard blending) 0aimed at closing-the-gap

________________ is used to evaluate the school counseling program in comparison with the ASCA national model. it should identify strengths and weaknesses of the program, provide direction for improvement, and should be reviewed annually.

School Counseling program Assessment

An evaluation of the counselor performance

School Counselor Performance Appraisal

______________ is used to monitor student progress data and determine what students need to achieve

School Data Profile

_______________ provides a summary of the school achievement, attendance, behavior, and safety school records over a multiyear (3 year) period and can contribute to a better understanding of trends within the school.

School Data Profile Analysis

Delivery of Services

School Guidance Curriculum Individual Student Planning Responsive Services System Support School Wide Interventions Consultation and Collaboration

Delivery System

School Guidance curriculum indivual student planning responsive service system support

What is "collaborative-independent consultation"?

School counselor facilitates the consultation process and shares his or her expertise, as do others, in an effort to solve a problem

Consultation models are often "ethnocentric". What does ethnocentric mean?

School counselors adjust models and services to accommodate individuals worldviews and bridge cultural differences.

Goals 2000 Educate America Act National Skills Standards Act School to work Opportunities act of 1994 Examples of?

School to work movement

RTI models and curriculum are based off of?

Scientific Support

Mode

Score that appears most frequently

Development of self concept

Shift of understanding thru internal not external, elementary school kids will define themselves in terms of social characteristic, self understanding increase to social comparison w/ others. What they can do compared to others. Self concept/Self esteem are closely related.

Informed Consent

Should be given by everyone participating in the counseling session. Informed consent includes explanations of guidelines, instructions, confidentiality issues, fees, and rights of all group members.

________ theorist believed personality development was driven by sexual interests and pleasures

Sigmond Frued

Which theorist believed religion to be the "universal obsessional neurosis of humanity"?

Sigmund Freud - he is considered the founder of psychoanalysis.

Name this therapy: brief in nature positive psychology Focused on solutions not problems focused on strengths not weaknesses exceptions what are you doing that works? reframing miracle question scaling 1-10 mind mapping

Solution focused brief counseling

ASCA Career Development Guidance Curriculum Standards

Standard A: Students will acquire the skills to investigate the world of work in relation to knowledge of self and to make informed career decisions. Standard B: Students will employ strategies to achieve future career goals with success and satisfaction. Standard C: Students will understand the relationship between personal qualities, education, training and the world of work.

An acceptable measure of variability in a given set of data

Standard Deviation

a method of scaling test scores on a nine-point standard scale with a mean of five and a standard deviation of two.

Stanine *Assessment

What interval in a stanine distribution has the largest percentage of the scores?

Stanine 1 lowest 4% Stanine 2: 7% Stanine 3: 12% Stanine 4: 17% Stanine 5: 20% (Largest %) Stanine 6: 17% Stanine 7: 12% Stanine 8: 7% Stanine 9: Highest 4%

Differential Statistics

Statistical technique for determining the difference between two or more groups. Ex: alcoholic consumption across ethnic groups

According to Holland's theory, what does undifferentiated mean?

Students who have many interests with no clear preference are described as undifferentiated

Structured group v. unstructured group

Structured groups feature a sequence of lessons or activities whereas unstructured groups have no agenda, lessons, or activities planned - unstructured groups tend to be open.

What activities focus on Positive Behavior Systems?

Student Assemblies Staff Development Reward Systems PBS is school wide and promotes positive discipline. Everyone involved in the school should support the implementation of PBS.

In schools include academic support for students with special needs, the promotion of rigorous courses that support future goals, and regular contact with organizations that can provide support for students academic and emotional development

Student Support Service

SST

Student Support Team; used for helping students with technology and with any of their school related needs

Best place to store student data?

Student information system

What is the advantage to placing a disabled student in a regular classroom?

Student's with disabilities learn to function in the kind of environment they will encounter when they leave school; contributes more to overall development.

Where should input come from for a "needs" assessment?

Student's, parents, teachers, and community stakeholders

What is peer mediation?

Students of the same age group help with resolving disputes. Peer mediation is known to help with improved self-esteem and critical thinking skills among students.

Mentoring Programs

Students receive tutoring from and participate in recreational activities with adults from the community during free periods or after school.

Evaluate learning at the end of a unit against some standard or benchmark

Summative Assessment

Believed that children are driven by curiosity, develop self-control, which leads to planful career decision making

Super

Donald Super

Super's Life-Span and Life-Space Theory: "Life Rainbow" Belief that our identities, and by extension our career identities, are a product of how we see ourselves. Our vocational choices put this concept into practice in the real world. Career: course of events that constitute a life Career Awareness: developing an inventory of one's knowledge, values and preferences Career Development: the process of building the inventory of one's own knowledge, values and preferences Career Decision Making: process of choice, entry and adjustment related to one's career

When using the prescriptive model of consultation with a teacher to address behavioral problems in the classroom, what should the counselor do?

Support the teacher when he or she conducts an intervention, provide expertise in defining a problem and determine the most appropriate actions for the teacher to take. The prescriptive model involves the school counselor acting as an expert and then helping the teacher to implement the appropriate solution.

TRUE or FALSE: Email communication is subject to FERPA?

TRUE

True or False: "sole possession" case notes are not consider part of the students academic profile

TRUE

True or False: Sexting is against the law

TRUE

True or False: regularly sharing results with admin, faculty, and community members will likely increase understanding, promote respect and support for counselors

TRUE

True or false: Cyberbullying is considered by law a criminal offense

TRUE

True or false: individual planning is different than individual counseling

TRUE

A student with a specific learning disability is meeting with a school counselor to select courses for the 11th grade year. The student wishes to graduate with a standard diploma. The student has an overall GPA of 3.4 and has already earned 14 credits during high school. In grade 10, the student passed the statewide reading assessment but failed the statewide mathematics assessment with accommodations. Teachers describe the student as hardworking and focused. Which of the following recommendations would be most appropriate for the school counselor to make to the student regarding courses for the upcoming school year?

Taking an intensive remediation course in mathematics

Fair Share Tasks

Tasks that do not fall within any specific job description but are necessary for the operation of a school. Duties are shared among all faculty and staff. Ex: keeping students safe

Peer Mediation programs and Peer helping program are good examples of?

Teaching Conflict resolution

As a counselor, how should you respond when a parent wants to tell you something about her child but does not want the child to know.

Tell the parent that anything she says will be shared with her child.

a group of behavioral traits that are relatively constant over a child's development is referred to as a child's ______________

Temperament

Matt and Steven are both in kindergarten. Matt prefers to play quietly by himself and is fearful of new situations. Steven prefers to be in the middle of a lot of activity and happily tries new things. What best explains the differences between the two boys' behavior?

Temperament - this is a group of behavioral or emotional traits that are relatively constant over a child's development.

___________________ is the foundation for classroom lessons, small groups, and activities of the school counseling program

The 3 domains of the ASAC Student Standards (academic, career, personal/social development)

What does ASCA say about romantic relationships between counselors and students?

The ASCA Ethical Standards for School Counselors say that "any sexual or romantic relationship with students whether illegal in the state of practice is considered a grievous breach of ethics and is prohibited regardless of a student's age" and that the school counselor should "consider the potential for harm before entering into a relationship with former students."

Carl D Perkins Career and Technical Education Act

The Carl D. Perkins Career and Technical Education Act (Perkins) was most recently reauthorized in August 2006. The purpose of Perkins is to provide individuals with the academic and technical skills needed to succeed in a knowlegde- and skills- based economy through career and technical education.

What is the average score on a standardized intelligence test such as the Weschsler Intelligence Scale for Children? a. 100 b. 90 c. 120 d. 110 e. 95

The Correct answer is: 100 The average score on standard IQ tests is 100. Scores between 80 and 120 are said to indicate "normal" intelligence.

How many categories of scores are there in a stanine distribution? a. 3 b. 4 c. 6 d. 9 e. 10

The Correct answer is: 9 A stanine distribution is based on the concept that a normal distribution is divided into nine intervals. Each interval, except the first and last, has a width of 0.5 standard deviations.

Which of the following is associated with the humanist approach to child development? a. Abraham Maslow b. B.F. Skinner c. John B. Watson d. E.L. Thorndike e. Ivan Pavlov

The Correct answer is: Abraham Maslow Maslow is probably the best-known exponent of the humanist theory of child development. Maslow's hierarchy of needs describes the stages of growth in human development.

Validity is an important characteristic of effective testing. In regard to testing, which of the following is the MOST accurate definition of validity? a. statistical significance of test results b. alignment with what students are expected to learn c. similarity in results across all classes d. normal distribution of test scores e. fairness to all students

The Correct answer is: Alignment with what students are expected to learn. A valid test accurately reflects what students are expected to learn. In other words, it is aligned with the subject matter being tested.

Which of the following is rejected by cognitive theory? a. Freudian psychology b. Behaviorism c. Humanistic psychology d. The theory of multiple intelligences e. IQ theory

The Correct answer is: Behaviorism Cognitive theory attempts to explain human behavior by analyzing complex thought processes. It rejects behaviorism because behaviorism reduces human behavior to a mechanical model of cause and effect.

Aptitude tests are intended to measure what? a. Capacity for learning. b. Specific academic abilities. c. Mastery of a subject. d. Class rank. e. Learning readiness.

The Correct answer is: Capacity for learning Aptitude tests are intended to measure a student's capacity for learning. They are different from tests intended to measure a student's academic achievements or mastery of a specific subject, such as reading or writing.

Which of the following is a leading exponent of the theory of emotional intelligence? a. Daniel Goleman b. Jean Piaget c. Erik Erikson d. John Dewey e. Lawrence Kohlberg

The Correct answer is: Daniel Goleman Daniel Goleman has written several books on the topic of emotional intelligence. Goleman describes emotional intelligence as the ability to understand and manage emotions in oneself and others.

Which of the following is mandated by the Individuals with Disabilities Education Act and its amendments? a. Equal employment opportunity for teachers. b. Equal educational opportunities for boys and girls. c. Reduced graduation requirements for students with disabilities. d. Early intervention programs for pre-school children with disabilities. e. A standard curriculum for students with the most common types of disabilities.

The Correct answer is: Early intervention programs for pre-school children with disabilities. The Individuals with Disabilities Education Act, as amended in 2004, mandated early intervention programs for young children, ages birth to two, who have shown some signs of disability. Each state is required to develop an early intervention plan.

The median score on a standardized test with 100 questions is 57. This means what? a. the average score on the test was 57 b. half the students who took the test scored above 57 and half scored below 57 c. the average student's score was in the 57th percentile d. most student failed the test e. most students passed the test

The Correct answer is: Half the students who took the test scored above 57 and half scored below 57

Differentiated instruction is a teaching theory based on what idea? a. The same instructional approaches can be used at all grade levels. b. Different subjects call for different instructional approaches. c. Most instructional approaches can be used with many different types of students. d. It is not practical to adapt instructional approaches to the needs of individual students. e. Instructional approaches should be adapted to individual students.

The Correct answer is: Instructional approaches should be adapted to individual students Differentiated instruction is based on the idea that students learn in different ways. Therefore, instructional approaches should be adapted to individual students.

Isobel's score on a standardized test places her in the 76th percentile of students in her district. What does this mean? a. Isobel answered 76 percent of the questions on the test correctly. b. Isobel achieved a passing grade. c. Isobel's scored lower than 76 percent of the students who took the test. d. Isobel's score was below average. e. Isobel's scored higher than 76 percent of the students who took the test.

The Correct answer is: Isobel's scored higher than 76 percent of the students who took the test This question tests your understanding of percentile rank. A student who scores in the 76th percentile on a test scores higher than 76 percent of the students who took the test. Percentile rank is not related to passing or failing grades, and it is not a direct indication of how many questions a student answered correctly.

Which of the following is associated with the theory of cognitive development? a. Jean Piaget b. Sigmund Freud c. B.F. Skinner d. Edward Thorndike e. Abraham Maslow

The Correct answer is: Jean Piaget Piaget is generally considered the leading proponent of the theory of cognitive development. According to this theory, children cannot learn certain subjects until they have reached a certain point in the maturation process.

A counselor says to a student, "Don't you think it was a mistake to make fun of Diane in front of her friends?" This type of question can best be described as what? a. Exploratory b. Probing c. Open-ended d. Judgmental e. Impartial

The Correct answer is: Judgmental This "question" is really a statement of opinion rather than a question. It makes a judgment about a student's behavior. In a counseling session, questions should be used to gain information, not to make implied judgments about a student.

A school uses a stanine system to report the results of a reading test given to all third-graders. This type of test is called what? a. Criterion-based. b. Norm-referenced. c. Formative. d. Summative. e. Cumulative.

The Correct answer is: Norm-referenced. A stanine is a type of scaled score used in many norm-referenced standardized tests. There are nine stanine units ranging from 9 to 1. Stanine scores are usually described as above average (9, 8, 7), average (6, 5, 4), and below average (3, 2, 1).

The Individuals with Disabilities Education Act does what? a. Requires that handicapped students receive exactly the same services as other students. b. Requires school districts to place special education students in the least restrictive environment possible. c. Requires a school district to spend the same amount per capita on special education students and other students. d. Sets different graduation requirements for special education students. e. Establishes certification exams for special education teachers.

The Correct answer is: Requires school districts to place special education students in the least restrictive environment possible. One of the cornerstones of special education law is that students should be placed in the least restrictive environment possible. The idea behind this approach is that special education students should not be isolated from the rest of the school population.

What is true according to the attribution theory of motivation? a. Students respond best to a consistent system of rewards. b. Students are motivated primarily by external forces. c. Students are strongly motivated by the chance to feel good about themselves. d. Students respond more positively to rewards than punishments. e. Students should be rewarded for the effort they put into an activity, regardless of the outcome.

The Correct answer is: Students are strongly motivated by the chance to feel good about themselves According to the attribution theory of motivation, the need to feel good about oneself can be a powerful motivator. While some of the other choices may be true, they do not reflect the attribution theory of motivation.

Which of the following is the best example of a behavioral objective? a. All eighth grade students will take an introductory course in Spanish. b. At least 60% of high school graduates will go on to college. c. Students will have a greater appreciation of classical music. d. Students will be able to solve basic algebra problems with 75% accuracy. e. All students will take part in some form of community service.

The Correct answer is: Students will be able to solve basic algebra problems with 75% accuracy A behavioral objective is specific and measurable. It describes a behavior that demonstrates a student's mastery of a subject. Taking a course, going on to college, and taking part in some form of community service do not demonstrate mastery of a subject. Having a greater appreciation of classical music is not measurable; therefore, it is not a behavioral objective.

Which of the following is a significant difference between a support group and a therapy group? a. A therapy group is usually large than a support group b. A therapy group meets on a regular basis; a support group does not c. A support group does not include a counselor; a therapy group does d. The counselor takes a more active role in a therapy group than in a support group e. A support group is more appropriate than a therapy group for younger students

The Correct answer is: The counselor takes a more active role in a therapy group than in a support group In a therapy group, the counselor guides the group toward achieving therapeutic objectives. In a support group, the agenda is set more by the members of the group. Support groups are intended mainly to offer support to the members of the group. They are less likely to have therapeutic goals.

Which of the following best describes classroom guidance activities? a. They are usually conducted by teachers using curricula developed by counselors. b. They are useful mainly when a teacher identifies a particular problem that affects most students. c. They should be used only in exceptional cases. d. They are primarily related to disciplinary issues. e. They are an integral part of the curriculum.

The Correct answer is: They are an integral part of the curriculum Classroom guidance programs are usually conducted by counselors, not teachers. They are meant to address a broad range of issues that address student performance. They should be treated as an essential part of the curriculum, not a response to disciplinary issues or other special circumstances.

All of the following are essential elements of an Individualized Education Program (IEP) EXCEPT a. a statement of the present level of educational performance b. a statement of measurable annual goals c. an explanation of progress measurement d. a description of special education services e. a listing of all teachers and counselors who will be involved in implementing the IEP

The Correct answer is: a listing of all teachers and counselors who will be involved in implementing the IEP An IEP is a roadmap that describes the educational path for students with disabilities. It describes a student's current situation, sets goals for development, and defines the educational tools that will help the student reach those goals. It does not include a listing of all teachers and counselors who will be involved in implementing the IEP.

Under the Family Educational Rights and Privacy Act (FERPA), which of the following would NOT be considered directory information? a. a student's date of birth b. a student's dates of attendance c. a student's grade point average d. a student's home address e. a student's honors and awards

The Correct answer is: a student's grade point average Under FERPA, directory information includes all the information in these choices except grade point average. Grade point average is considered confidential information. It may not be released without the consent of parents.

Which of the following best describes what happens in the "norming" stage in Tuckman's model of group development? a. debate b. agreement c. adjournment d. formation e. arbitration

The Correct answer is: agreement During the norming stage of group development, the group comes to an agreement about how it will operate and what it will do. Some members of the group may have to make concessions so that the group can move forward.

In a comprehensive school counseling program, student progress toward meeting competencies is usually evaluated how often? a. weekly b. biweekly c. monthly d. annually e. on no regular schedule

The Correct answer is: annually This type of general evaluation is usually done once a year. More frequent evaluations would be impractical and would probably not produce useful results.

Which of the following must a counselor do under the Individuals with Disabilities Education Act (IDEA)? a. facilitate the IEP meeting b. attend the IEP meeting c. schedule the IEP meeting d. write a report on the IEP meeting e. develop the IEP

The Correct answer is: attend the IEP meeting The Individuals with Disabilities Education Act requires that schools develop Individualized Education Programs (IEP) for all students with disabilities. The only requirement for counselors is that they attend the IEP meeting. In practice, counselors often go beyond this requirement and help to develop and implement the IEP.

All of the following are types of statistical sampling EXCEPT a. convenience b. judgmental c. random d. cluster e. average

The Correct answer is: average Convenience sampling uses an easily available group. For example, all the students in the cafeteria might be used as a sample of the entire student population of a school. Judgmental sampling involves a judgment about who should be included in a sample. Random sampling, as its name implies, aims to be purely random. Cluster sampling uses clusters of units, for example, all fifth grade classes in a state

Specific actions that demonstrate competency in a certain area are called what? a. behavioral anchors b. goals and objectives c. criteria d. inventories e. assessments

The Correct answer is: behavioral anchors A behavioral anchor is a specific example of behavior that demonstrates competency at a certain level. For example, a behavioral anchor for critical thinking might be, "Identify and clarify an issue by reviewing available information."

Which of the following is included in the ASCA Student Standards for personal/social development? a. communication skills b. career development c. academic skills d. time management skills e. test-taking skills

The Correct answer is: communication skills The ASCA Student Standards for personal/social development include communication skills. The other choices are included in academic or career development standards.

A math teacher analyzes how well the results of a new math test correlate with results on previous tests. What type of validity is the teacher evaluating? a. predictive b. content c. discriminant d. concurrent e. temporal

The Correct answer is: concurrent If the results of a new test instrument correlate closely with the results of previous tests, the new test has concurrent validity. In determining concurrent validity, teachers and counselors usually compare a new test to a previous test that is widely used and accepted

Which of the following is NOT a characteristic of a support group? a. confidentiality b. openness c. discussion d. sharing e. trust

The Correct answer is: confidentiality A support group gives a student a chance to share his feelings and discuss problems in an atmosphere of trust. They do not provide confidentiality. Students cannot assume that what they say in a support group will remain confidential.

A teacher wants to create a test that covers the entire range of a subject. What type of validity is the teacher trying to ensure? a. content b. criterion c. predictive d. concurrent e. discriminant

The Correct answer is: content A test has content validity if it covers the entire range of a subject. A final exam would have content validity if it included questions on all the material covered in a class.

what type of validity would be most important in evaluating a school's curriculum? a. criterion b. content c. temporal d. logistical e. cultural

The Correct answer is: content An evaluation of a school's curriculum would be focused on content. Therefore, in conducting such an evaluation, a counselor would be mainly concerned with content validity.

Which of the following problems does the ASCA National Model attempt to address? a. counselors are underpaid b. counselors spend much of their time working with a small percentage of students c. counselors are not treated as professionals d. counselors do not receive proper training e. counselors are the first to be let go when budgets need to be cut

The Correct answer is: counselors spend much of their time working with a small percentage of students The ASCA National Model attempts to create a model for guidance programs. One of its main recommendations is that counselors should spend most of their time in service to all students, not just the small percentage of students whose problems are most obvious.

A school counselor wants to enhance services that facilitate the school-to-work transition. Which of the following would be part of this effort? a. reviewing demographic data for students b. reviewing test results c. developing a program to help students complete job applications d. developing a curriculum intended to enhance job prospects for students e. determining what careers would be appropriate for students

The Correct answer is: develop a program to help students complete job applications Reviewing demographic data or test results is not directly related to the school-to-work transition. Counselors may offer advice on curriculum development, but developing a curriculum intended to enhance job prospects for students is outside their role. Choosing an appropriate career is up to the student, not the counselor. Of these choices, developing a program to help students complete job applications is most closely related to facilitating the school-to-work transition.

A comprehensive guidance program uses a systematic, sequential approach to help students achieve their potential in three areas: academic, career, and personal/social. Which of the following describes this aspect of a comprehensive guidance program? a. results-based b. data-driven c. standards-based d. developmental e. comprehensive

The Correct answer is: developmental A comprehensive program is developmental in nature. It involves a long-term, systematic approach to helping students achieve their potential. In this respect, it is different from an approach that is crisis-driven and reactionary.

The issue most commonly addressed by classroom guidance activities is which of the following? a. time management b. developmental mental health c. student safety d. standardized testing e. course scheduling

The Correct answer is: developmental mental health The most important, and most common, issue addressed in classroom guidance activities is developmental mental health. Classroom guidance activities are typically intended to help students understand and cope with the problems they are experiencing in their lives. These activities can reduce the need for individual counseling by giving students the help they need to cope with their problems.

A student who scores below 80 on a standardized intelligence test is likely to be labeled what? a. average b. developmentally disabled c. gifted d. unteachable e. mentally ill

The Correct answer is: developmentally disabled The average score on a standard IQ test is 100. Scores between 80 and 120 are considered normal. Students who score below 80 are typically labeled developmentally disabled. Students who score above 120 are typically labeled gifted.

Which of the following is considered an at-risk population? a. dysfunctional families b. families of gifted students c. families of students with developmental disabilities d. minority families e. single-parent families

The Correct answer is: dysfunctional families Dysfunctional families are considered an at-risk population because they present serious challenges to children. The other choices represent families that may face problems. However, a family is not considered to be at risk just because it has a single parent or a minority background or a gifted or disabled child.

What general term is used to describe a significant difference in functioning from an established norm? a. exceptionality b. learning disability c. mental retardation d. impairment e. deficiency

The Correct answer is: exceptionality The term exceptionality is used to describe a significant difference in functioning from an established norm. For example, a student who scores below 80 or above 120 on an IQ test is said to have an exceptionality. The term is also used to describe functioning outside the norm in areas such as emotional or physical development.

Going to the zoo to learn about animals by observing them is an example of what? a. experiential learning b. didactic learning c. behavioral learning d. guided learning e. intentional learning

The Correct answer is: experiential learning Experiential learning is the process of making meaning from direct experience. In the example given in the question, a student goes to the zoo, observes animals, and then reflects on these observations.

The term "twice exceptional" is usually applied to which of the following? a. gifted students who have a disability b. students who have more than one disability c. students who are gifted in more than one area d. students who have a disability that makes it imposssible to place them in a regular classroom e. students who have the same disability as their parents

The Correct answer is: gifted students who have a disability Many gifted children have some form of disability that makes it difficult for them to develop their special talents. These disabilities span a broad range, inlcuding dyslexia, obsessive-compulsive disorder, Asperger syndrome, and sensory processing disorder. The combination of giftedness and disability often leads to problems in school, and twice exceptional children are often misdiagnosed or misunderstood.

What theory of development believes learning is viewed as a personal act to fulfill one's potential? a. behaviorism b. humanism c. constructivism d. conditioning e. Gestalt

The Correct answer is: humanism Humanist theories of development hold that it is necessary to study the person as a whole, especially as an individual grows and develops. The humanist approach to education is child-centered and personalized. The teacher is seen as a facilitator.

Stanine scores are reported as what? a. integers b. percentiles c. fractions d. decimals e. letter grades

The Correct answer is: integers In a stanine distribution, all scores are divided into nine groups, which are assigned an integer from 1 to 9. The drawback of this is that two scores in a single stanine are sometimes further apart than two scores in adjacent stanines.

The concept of locus of control involves what dichotomy? a. internal vs. external b. emotional vs. intellectual c. individual vs. group d. change vs. status quo e. introvert vs. extrovert

The Correct answer is: internal vs. external Locus of control refers to an individual's belief about what controls his or her life. On one hand, extreme individuals might believe that they control their own destiny through their actions and decisions. On the other hand, extreme individuals might believe that their lives are controlled by external forces such as fate or chance.

What term is used to describe the counseling approach that involves offering clients new ways to understand their experiences? a. interpreting b. reflecting c. evaluating d. assessing e. guiding

The Correct answer is: interpreting Reflecting, which is often the first step in responding to a client, involves just mirroring a client's message. Interpreting goes a step beyond that by offering a client new information or new ways of understanding experiences.

A school uses a system of grade point averages that runs form 0 to 4.0. What is this type of measurement system called? a. nominal b. ordinal c. interval d. ratio e. weighted

The Correct answer is: interval In an interval measurement system, the distance between items has meaning. The distance between a score of 2.0 and 3.0 is the same as the distance between 3.0 and 4.0. This type of system cannot be used for ratios. A GPA of 4.0 is not twice as good as a GPA of 2.0

Yalom's curative factors of groups involve all of the following EXCEPT a. emotional well-being b. personal development c. social development d. a feeling of usefulness e. leadership skills

The Correct answer is: leadership skills Yalom's curative factors of groups have nothing to do with leadership skills. Rather they are focused on learning from others and helping others learn about themselves. This process promotes emotional well being and personal and social development

Which of the following is a major theory of language development? a. assimilation theory b. nativist theory c. elaboration theory d. stage theory e. cognitive load theory

The Correct answer is: nativist theory The nativist theory of language development, advanced primarily by Noam Chomsky, argues that all children have an inborn understanding of how language works. The other choices are all cognitive learning theories. They are not specifically related to language development.

A school wants to analyze its student population in terms of ethnic background. Which type of measurement system would be appropriate for this task? a. nominal b. ordinal c. interval d. ratio e. weighted

The Correct answer is: nominal A nominal measurement system uses just the "names" of different groups. It does not involve ranking or ratios.

In what type of measurement system are numbers used as ranks but not as quantities? a. nominal b. ordinal c. interval d. ratio e. weighted

The Correct answer is: ordinal An ordinal system assigns ranks to different items (for example, 1 through 5). These numbers are ranks, not quantities. The difference between these numbers does not have meaning.

A system that tracks level of education for adults might use 0 for less than high school, 1 for some high school, 2 for high school degree, 3 for some college, 4 for college degree, and 5 for graduate degree. This type of measurement is called what? a. nominal b. ordinal c. interval d. ratio e. weighted

The Correct answer is: ordinal In an ordinal measurement system, the items are ranked, but the distance between the ranks has no meaning. The numbers assigned to the items are rankings, not quantities. So in this example, a college degree is not two times better than a high school degree

The school counselor is commonly engaged in all of the following EXCEPT a. developing the guidance curriculum b. parent education c. appraisal d. group counseling e. consultation

The Correct answer is: parent education School counselors are usually involved in appraisal, counseling, consultation, and developing the guidance curriculum. They may occasionally be involved in parent education, but this is not one of their common roles.

A counselor finds that a particular test offers a good indication of how well a student will do in an AP Calculus class. What type of validity does this test have? a. content b. convergent c. predictive d. discriminant e. concurrent

The Correct answer is: predictive This test is said to have predictive validity because it offers a good prediction of how well a student will do in a course. The most widely used predictive tests are those that students take when applying to colleges.

What term is used to describe a personality test designed to reveal hidden emotions and internal conflicts by using an individual's response to ambiguous stimuli? a. objective b. projective c. holistic d. artificial e. interpretive

The Correct answer is: projective A projective personality test is intended to provide insights into a subject's subconscious mind by analyzing responses to ambiguous items. The best known projective personality test is the Rorschach inkblot test.

The internal consistency of a test is one measure of its what? a. validity b. reliability c. accuracy d. predictability e. cost

The Correct answer is: reliability Internal consistency is a measure of reliability. Different questions on a given subject should produce similar results. If they do not, the test is not internally consistent.

During the preparation of an Individualized Education Program, a parent has a legal right to do what? a. review a student's complete education file b. observe the student and teacher working together in the classroom c. interview the school principal d. interview the student's guidance counselor e. request a review by an independent observer

The Correct answer is: review a student's complete education file During preparation of an Individualized Education Program, a parent has the right to review the student's education record. This includes things such as grades, report cards, discipline reports, and progress reports.

All of the following are Yalom's curative factors EXCEPT a. altruism b. interpersonal learning c. group cohesiveness d. universality e. self-discipline

The Correct answer is: self-discipline All of these values except self-discipline are included in Yamrom's curative factors. The curative factors describe what can be accomplished in group counseling sessions as opposed to one-on-one counseling. Yamrom's approach relies on sharing feelings and experiences as a way to cope with problems.

What theory focuses on the importance of observation in learning? a. social learning theory b. classical conditioning theory c. behavioral theory d. attribution theory e. cognitive theory

The Correct answer is: social learning theory Social learning theory was developed by Albert Bandura. It stresses the importance of observation and modeling in the learning process.

What term is used to describe a chart of the interrelationships within a group? a. sociogram b. Likert Scale c. stanine d. behavioral anchor e. histogram

The Correct answer is: sociogram A sociogram is a chart of the social interrelationships in a group of students. It can help a teacher identify friendships, isolated students, and personal conflicts. A teacher can create a sociogram by asking some simple questions such as, "Who do you like to sit with in class?"

A comprehensive guidance program is based on a clearly defined body of knowledge and skills. Which of the following describes this aspect of a comprehensive program? a. results-based b. data-driven c. standards-based d. developmental e. comprehensive

The Correct answer is: standards-based A comprehensive program is based on clearly defined standards. These standards define the knowledge and skills that students need to be successful in school and in life.

According to the ASCA National Model, the delivery system for a comprehensive guidance program includes all of the following EXCEPT a. the guidance curriculum b. individual counseling c. responsive services d. system support e. student recordkeeping

The Correct answer is: student student recordkeeping Student recordkeepiing is primarily a clerical task, and is not an essential part of the guidance delivery system. System support, which is an essential part, includes such things as professional development, data analysis and community outreach.

A comprehensive guidance program is results-oriented. This means that the guidance program is focused on what? a. student success b. test scores c. statistics d. evaluations and assessment e. self-awareness

The Correct answer is: student success A results-based program focuses on guaranteeing that all students acquire the competencies to become successful in school and make a successful transition to employment or higher education.

The No Child Left Behind Act requires schools to disaggregate test scores for all of the following groups of students EXCEPT a. economically disadvantaged students b. students from major racial and ethnic groups c. students with disabilities under the Individuals with Disabilities Education Act d. students with limited English proficiency e. students from single-parent households

The Correct answer is: students from single-parent households NCLB does not require schools to disaggregate data for students from single-parent households. The law does require disaggregated data for the other choices to close the achievement

Behavioral approaches to teaching involve all of the following EXCEPT a. breaking down skills and information to be learned into small units b. checking student's work regularly c. providing frequent feedback d. direct "teacher-centered" instruction e. understanding a student's mental processes

The Correct answer is: understanding a student's mental processes Behavioral approaches to teaching make no attempt to understand a student's mental processes. Rather, they focus on observable behavior, such as solving math problems correctly. Behavioral approaches are "teacher-centered" in the sense that teachers decide exactly what students will learn and when they will learn it. Behavioral approaches require teachers to check students' work frequently and provide appropriate feedback.

The Ethical Standards for School Counselors developed by the American School Counselor Association address the responsibilities of counselors to all of the following EXCEPT a. students b. unions c. parents d. the community e. colleagues

The Correct answer is: unions The Ethical Standards for School Counselors do not concern responsibilities to unions. In addition to defining responsibilities, the Ethical Standards also address resources for counselors and maintenance of standards.

Articulation

The aligning of school curriculum in a way that each concept builds on the prior concept. Counselors can make sure curriculum is properly articulated, so that students can properly understand it.

Mr. Andrews approaches the counselor with concerns about one of his students. Mr. Andrews reports that the student is unable to keep up with the rest of the class, is disruptive, and acts out inappropriately. What steps should the counselor take to begin assessing the student?

The assessment would begin with a cumulative review of the student's record, interviewing the student to get a personal perspective, and, because there is a behavioral issue, an observation in the location where the problem is exhibited.

Which of the following components is the LEAST important in a comprehensive professional school counseling program? (A) Student outcomes and competencies (B) Direct delivery of counseling on a demand basis (C) Placement, follow-up, and follow-through activities (D) Accurate counts of the frequency of student-initiated contact with the school's counselors

The best answer is (D). Major features of comprehensive professional school counseling programs include a focus on student outcomes or competencies. Student achievement of these outcomes is the responsibility of the program. Guidance activities designed to assist students in achieving these competencies are organized accordingly. In addition, comprehensive programs provide for placement, follow-up, and follow- through activities to assist students in their next steps. Direct delivery of counseling and other guidance activities on a demand basis are included because there may be a need for direct immediate services to students while they are still in the school building.

A five year old boy continues to attempt to play with children who openly ridicule him and leave him our of games. His persistence with this group of children has recently resulted in a number of fights on the playground. After being referred to your office, you learn that he has an unsupportive home life. His parents were divorced last year. He lives with his mother who works nights, and he does not see his father. He is often left at home alone; however, a neighbor watches him from time to time. According to Pavlov's theory of development, what would you expect to occur in terms of this child's learning and behavior?

The child would become conditioned to the negative response and avoid the situation.

Concurrent Validity is ...

The comparison of two measures done at the same time - the degree to which two tests are in agreement

Who directs the consultation process?

The consultant (often the school counselor)

When is it appropriate for a counselor to disclose information that a student has shared?

The counselor is assisting in the prevention of imminent danger to the student or to others. School counselors have a duty to warn if there is potential harm to the student or to others.

Ilana is a tenth grader who is involved in extracurricular political activities and is outspoken about her political beliefs. A few students have complained to the school counselor that Ilana talks excessively about an upcoming election and tells students who they should vote for. What is the best way for the counselor to handle the situation?

The counselor should remind the students that freedom of speech is a constitutional right. This means that Ilana and the other students are able to talk about what they want. It is appropriate for the counselor to encourage the students to work it out among themselves first. If it cannot be resolved, then the counselor could mediate.

Probing questions

The counselor wants the student to think about the situation and respond in a way that will help both of them understand what is going on

According to Carl Rogers, what is a characteristic of a congruent counselor?

The counselor's actions are not at odds with her feelings and thoughts. Carl Rogers defined "congruence" as an ability to be authentic and genuine when working with a client. When a counselor's thoughts, feelings, and behaviors are not at odds with each other, the counselor is better able to be with the client more fully.

Ordinal scale of measurement

The data is ordered/ranked in some way - but there is not equal distance between the ranks. This indicates the relative standing among subjects/variables Ex. ranking subject scores on a depression inventory or biology test; race results

An elementary school teacher comes to the counselor with a concern that a student often answers questions with irrelevant responses, seems confused when there is a lot of background noise in the classroom, and does not seem to hear classroom instructions clearly. The counselor meets with the student and agrees with the teacher's concerns about the student. What is the most effective course of action for the school counselor to take after meeting with the student?

The first step a school counselor should take in this situation is to verify whether the student has a medical problem.

A school counselor that views the world through a single cultural lens demonstrates:

cultural encapsulation

Cognitive Development

The process of a child's understanding and views of the world changing due to age and life experiences

Nominal scale of measurement

The simplest form. There is no relationship between the categories that could lead to ranking Researchers assign a number to different categories for analysis Ex. gender, teaching method, school

What is cohesion in regard to group counseling?

The stage where students become comfortable with the group.

If a counselor makes a judgement about a student to a teacher and the student finds out, what is the likely outcome?

The student will no longer trust the counselor and the student will intentionally misrepresent herself in counseling in an attempt to be judged positively.

A reliability coefficient of 1.0 on two administrations of the same test means what?

The test scores for the two administrations were identical. The test is reliable. Reliability is not related to validity which is a measurement of how well a test measures what it is supposed to measure.

What is a disadvantage of asking clients open-ended questions during counseling?

The therapeutic process may get off track if a client is verbose - an open-ended question enables the client to choose how to respond. This freedom may be positive if it allows the client to move on to topics that are of concern, but it may also allow the client to move away from the present focus of therapy.

A parent education program conducted by a school counselor best addresses what?

There has been an increase in the number of students diagnosed with learning disorders and attention-deficit/hyperactivity disorder. Parents are having difficulty understanding the educational needs of and available accommodations for students, and are not supporting the children appropriately at home or using appropriate channels to request accommodations. A parent education program could help parents understand students' educational needs and how to support them, as well as how to obtain further assistance if needed.

What is discriminant validity?

There is no relationship between test results and unrelated variables (racial or ethnic background)

How are standard scores expressed?

They are expressed in units of standard deviations from the mean set at 0.

What are the rights of parents to information about their children's counseling?

They are legally the "client" you can ask them to respect the counseling safe space - if necessary you can provide them with a vague summary outline of what was discussed.

What is an "immediacy response"?

They are responses that are focused on the present, right at the time, place, and context of the counseling session

What are fair-share responsibilities? An Example?

They are tasks that do not fall w/in any specific job description but are necessary for school operation. These duties are shared among all school and faculty. Bus duty may be an example of this. It doesn't fall under one person's responsibility but is still crucial for school safety. Thus, teachers and staff rotate performing them as assigned by the principal.

What do aptitudes test measure?

They estimate abilities and predict future performance

The Transforming School Counseling initiative, along with the ASCA National Model, is shifting the role of professional school counselors within the larger education system to meet expanding demands and serve larger populations. What will future counselors need to do to meet these challenges effectively?

They need to become professionally active through state and national counseling associations to ensure the integrity of the school counseling profession. Membership in professional organizations can increase the counselor's breadth and depth of counseling knowledge and provide a collaboration and support network.

What is an advantage of classroom guidance lessons?

They reduce the need for individual interventions.

According to the Social Cognitive Theory, to what does the term "perspective taking" refer?

Thinking cognitively about another person's situation.

Egocentric

Thinking only of one's self without regard for feelings/desires of others.

delivery system is

how the program will be implemented

What is the primary purpose of the NAEP?

To provide assessments of student knowledge and skills in various subjects. Run by the U.S. department of Education, it is the most comprehensive assessment of what American students know and can do

Annual school-wide needs assessments and program evaluations are examples of which component the ASCA National Model for a comprehensive school counseling program?

This is an example of how a counselor demonstrates accountability and effectiveness in measurable terms.

What occurs in the adjouring stage of group counseling?

This is the closure stage where group members separate and learn to support themselves.

What is the goal of consultation?

To indirectly improve the client or situation by empowering and directly developing the knowledge, skills, and abilities of the consultee.

IOSIE model

To look at and reflect on solutions for student related behavioral problems; (I)dentify the problem figure out the (O)bjectives on the intervention (S)olution of the plan (I)mplementation of the plan (E)valuation or reflection of the final results

Destiny, a tenth-grade student, transfers schools. The counselor receives Destiny's assessment data from her previous school and reviews her intelligence, achievement, career, and behavioral assessments. What is the most likely reason for the counselor's review?

To understand Destiny's strengths, interests, weaknesses, and presenting concerns. Corroborating data from a number of sources help create a more thorough understanding of Destiny and her presenting concerns. The school counselor can then interpret these data and formulate hypotheses related to Destiny's strengths and weaknesses.

According to which theory does every person have their own unique pattern of traits made up of everything from interests to personality characteristics?

Trait-Factor Theory

Justice

Treating individuals equitably and fostering fairness and equality.

Behavioral consultation is an example of which of the consultation models?

Triadic-dependent - in the behavioral consultation approach, the counselor actively obtains and evaluates student data that is used to advise other staff members (such as teachers) on how best to help the student. This approach is a triadic consultation (among counselor, teacher, and student), and it is considered dependent because the consultee (teacher) relies on the school counselor to collect and interpret the behavioral data.

_____________ Informs many components of a comprehensive school counseling program such as: annual agreement, calendars, curriculum, small groups and closing-the-gap action plans.

Use-of-time analysis

______________________ assessment helps the counselor determine how much time is spent in each of the components of the ASCA national model. This should be completed twice a year.

Use-of-time assessment

Inferential Statistics

Used for inferring or generalizing from a sample to the population from which the sample was drawn. Descriptive, correlational, and differential statistics can be made inferential if random sampling is used. Examples of a population might include: the citizens of city, high school students, the taxpayers in a state.

Dyadic Counseling

Used in groups counseling; 2 people are paired together and guided by the counselor towards more mature social interactions

Results Based Model

Used to check accountability of counseling programs. Elements include: mission, philosophy, glossary, goals, student competencies, management system, results agreements, needs data, results plans, monitoring system, master calendar, and advisory council.

Correlational Statistics

Used to examine the relationship between two or more variables. At the heart of correlational statistics is the correlational coefficient. Ex: Temperature and chili consumption

Psycho-metric or Test Statistics

Used to explore and describe the psychometric properties of tests, measures, scales, or subscales. such statistics assist in establishing the validity and reliability of measures. Ex: School Vision Inventory

Carl Rogers defined "congruence"

as an ability to be authentic and genuine when working with a client. When a counselor's feelings, thoughts, and behaviors are NOT at odds with each other, the counselor is better to be with the client more fully.

A well-organized counseling program identifies goals and

assigns responsibility to those who will provide services.

Appropriate types of questioning as a counselor

at the beginning of an interview, asking for more information about a topic, asking for more-concrete specifics about a topic, and gathering information for assessment purposes

a form of psychological treatment in which the patient is exposed to a stimulus while simultaneously being subjected to some form of discomfort. This conditioning is intended to cause the patient to associate the stimulus with unpleasant sensations in order to stop the specific behavior.

aversion therapy

Predictive validity

by correlating test scores with scores on a criteriation measure at a later time, aptitude test

___________ system includes the transitions and shifts in one's lifespan.

chronosystem *Ecological Theory

Success results of systemic changes can be measured by?

closing of achievement, opportunity, and attainment gaps. ex. increased grad rates ex.decreased discipline or suspension rates increased attendance

Mentor programs

commonly include tutoring and recreational activities provided by adults in the community.

Examples of responsive services:

counseling, consultation, referrals, education mediation, problem solving, community intervention

theme two; Advocacy

counselors advocate for students' educational needs and work to ensure these needs are addressed at every level of the school experience.

theme one; Leadership

counselors are ingaged in systemwide change to insure students success. they promote succes by closing the existing achivement gap.

theme four: Systemic change

counselors can asses the school for systemic barriers to academic success. Counselors shoudl use data to advocate, for every student, ensuring equitu and acces to rigorous curriculum wich maximizes post-secondary options.

theme three: Collaboration and teaming

counselors work with all stakeholders both inside and outside the school system, to develop and implement responsive educational programs that support the achievement fo the identified goals for every student

The importance of collaborative relationships is most obvious during time of _______________

crisis

interscore reliability (reliability coefficients)

crucial when there may be some variability between different scores

Validity

extent to which the measurement instrument or test accurately measures what it is supposed to measure

brief solution focused counseling

focuses on helping clients create solutions in a straight-forward manner within a limited amount of time. It is based on the assumptions that clients have the necessary strengths and resources to change and that counseling is most effective when focusing on constructing solutions unique to each client.t shifts the focus from problem solving to creating present and future solutions

Career planning is a vital service of

high school counselors.

Construct validity refers to

how accurately a test measures the abstract, psychological characteristics it claims to assess.

accountability answers what questions

how are students diffrent as a result of the program

What are "exceptions" according to solution focused therapy?

identifying times when the problem is not present

reliability

if a test is reliable it is consistent

What do students w/ down syndrome most commonly exhibit?

impairment in cognitive functioning

Transition stage of group description

leader helps group begin to work on concerns - members decide whether to take risks and speak of things they may be holding back

Jesse B. Davis, Frank Goodwin, Anna Reed, Eli Weaver, and Frank Parsons were

leaders of the early guidance movement.

____________ is an essential skill for School counselors as they develop and manage a comprehensive school counseling program

leadership

ASCA incorparates 4 themes

leadership, advocacy, colaboration, and systemic change as part of the frame work of the national model

The consistency with which an instrument measures

reliability

percentile rank

represents the percentage of test takers who scored lower on the test than that person. Therefore, a score at 83rd percentile means that 83 percent of those taking the test scored lower than the student receiving that score.

A limitation of group counseling is that it

requires a high level of leadership skill and energy.

Understanding a student's cultural values

requires exposure to a variety of individuals within the cultural group

Scheduling group services for students

requires planning with teachers and administrators.

What does the term "least restrictive environment" mean?

requires schools to educate students with disabilities with the regular ed. population to the maximum extent possible and appropriate for students with disabilities.

behavioral consultation approach

school counselor actively obtains and evaluates student data, which is used to advise other staff members (such as teachers) how to best help the student. With a behavioral consultation approach, it is a triadic consultation (among counselor, teacher and student), and is considered depended because the counselee (teacher) relies on the school counselor to collect and interpret the behavioral data.

Administrators organize __________________________ teams

school leadership teams

Parental involvement at the high school level

seems to be increasingly important in U.S. society.

Carl Rogers

self actualization; the motivating force of achieving their full potential

Homeostasis

self-awareness *Gestalt

Bandura's social cognitive theory

theory focuses on the fact that people can learn without being directly reinforced (vicarious learning). They can learn by watching someone perform an action.

Cultural identity groups

therapeutic for children experiencing prejudges, strengthening their resilience and coping in response to prejudicial attitudes and behaviors, as well as promoting their positive self-identity within their culture

School counselor evaluation is different from teacher evaluation because

there are so many possible methods of data collection.

trait- factor career development

there is a direct relationship between interests and abilities

What can children do in the concrete operational stage of Jean Piaget's theory of cognitive development?

they can recognize that even though a liquid can change shape, the amount doesn't decrease


Ensembles d'études connexes

GI Pharm Practice Questions, Med Surg Chapter 23: Nursing Management: Patients With Gastric and Duodenal Disorders: PREPU, Chapter 46: Mgt With Gastric and Duodenal Disorders, Med Surg Ch. 46 Upper GI, Chapter 46: Management of Patients With Gastric...

View Set

Ch. 28 Pregnancy & Human Development

View Set

8.5 Pharaoh Hatshepsut: Promoter of Egyptian Trade

View Set

Ch 17 Schizophrenia Spectrum Disorders

View Set

Chapter 14 Chemistry Study Guide

View Set

Chapter 12 Back of the Book Questions

View Set